• Shuffle
    Toggle On
    Toggle Off
  • Alphabetize
    Toggle On
    Toggle Off
  • Front First
    Toggle On
    Toggle Off
  • Both Sides
    Toggle On
    Toggle Off
  • Read
    Toggle On
    Toggle Off
Reading...
Front

Card Range To Study

through

image

Play button

image

Play button

image

Progress

1/164

Click to flip

Use LEFT and RIGHT arrow keys to navigate between flashcards;

Use UP and DOWN arrow keys to flip the card;

H to show hint;

A reads text to speech;

164 Cards in this Set

  • Front
  • Back
A 25-year-old female presents to your office complaining of exertional dyspnea and fatigue. Her past medical history is insignificant. She does not smoke or consume alcohol. Her blood pressure is 110/70 mmHg and heart rate is 90/min. Physical examination reveals pale conjunctiva. Laboratory values are:

ESR 15 mm/hr
Hemoglobin 7.5 g/dL
MCV 70 fl
MCHC 29%
Leukocyte count 7,000/cmm
Segmented Neutrophils 55%
Bands 3%
Eosinophils 3%
Basophils 0%
Lymphocytes 32%
Monocytes 7%

What is the next best step in the management of this patient?


A. Bone marrow sampling
B. Iron studies
C. Serum folate level
D. Schilling test
E. HbA2 measurement
This patient presents with microcytic/hypochromic anemia. The conditions that should be considered in differential diagnosis in this patient include iron-deficiency anemia, thalassemias, anemia of chronic disease, and myelodysplasia/sideroblastic anemia. Iron deficiency is the most common cause of anemia. The other conditions listed above are relatively uncommon. Iron studies help to confirm the diagnosis of iron-deficiency anemia, as well as rule out other causes. Iron studies typically reveal depressed serum iron, increased total iron binding capacity (TIBC), and a decreased serum ferritin level in patients with iron-deficiency anemia. In anemia of a chronic disease, TIBC is below normal and the serum ferritin level is normal or increased. Serum iron values, as well ferritin values, are normal to high in patients with thalassemias or myelodysplasia/sideroblastic anemia. If the diagnosis of iron-deficiency anemia is confirmed, the cause of the problem should be carefully searched.

Bone marrow sampling (Choice A) can be employed to determine the storage iron, but serum ferritin is typically used as an indicator of iron reserve. Bone marrow sampling may be necessary if myelodysplasia is strongly suspected.

A serum folate level (Choice C) and a Schilling test (Choice D) are used in the work-up of patients with megaloblastic anemia.

The HbA2 level (Choice E) is frequently elevated in patients with thalassemia.

Educational Objective:
Iron studies are indicated in patients who present with microcytic/hypochromic anemia. They are useful in confirming the diagnosis of iron-deficiency anemia, the usual cause of microcytic/hypochromic anemia, and ruling out other causes.
A 25-year-old female presents to your office complaining of exertional dyspnea and fatigue. Her past medical history is insignificant. She does not smoke or consume alcohol. Her blood pressure is 110/70 mmHg and heart rate is 90/min. Physical examination reveals pale conjunctiva. Laboratory values are:

ESR 15 mm/hr
Hemoglobin 7.5 g/dL
MCV 70 fl
MCHC 29%
Leukocyte count 7,000/cmm
Segmented Neutrophils 55%
Bands 3%
Eosinophils 3%
Basophils 0%
Lymphocytes 32%
Monocytes 7%

What is the next best step in the management of this patient?


A. Bone marrow sampling
B. Iron studies
C. Serum folate level
D. Schilling test
E. HbA2 measurement
This patient presents with microcytic/hypochromic anemia. The conditions that should be considered in differential diagnosis in this patient include iron-deficiency anemia, thalassemias, anemia of chronic disease, and myelodysplasia/sideroblastic anemia. Iron deficiency is the most common cause of anemia. The other conditions listed above are relatively uncommon. Iron studies help to confirm the diagnosis of iron-deficiency anemia, as well as rule out other causes. Iron studies typically reveal depressed serum iron, increased total iron binding capacity (TIBC), and a decreased serum ferritin level in patients with iron-deficiency anemia. In anemia of a chronic disease, TIBC is below normal and the serum ferritin level is normal or increased. Serum iron values, as well ferritin values, are normal to high in patients with thalassemias or myelodysplasia/sideroblastic anemia. If the diagnosis of iron-deficiency anemia is confirmed, the cause of the problem should be carefully searched.

Bone marrow sampling (Choice A) can be employed to determine the storage iron, but serum ferritin is typically used as an indicator of iron reserve. Bone marrow sampling may be necessary if myelodysplasia is strongly suspected.

A serum folate level (Choice C) and a Schilling test (Choice D) are used in the work-up of patients with megaloblastic anemia.

The HbA2 level (Choice E) is frequently elevated in patients with thalassemia.

Educational Objective:
Iron studies are indicated in patients who present with microcytic/hypochromic anemia. They are useful in confirming the diagnosis of iron-deficiency anemia, the usual cause of microcytic/hypochromic anemia, and ruling out other causes.
A 32-year-old Caucasian female presents to your office for a routine check-up. Her past medical history is significant for generalized seizures controlled with chronic phenytoin therapy. The last seizure was six months ago. She does not smoke or consume alcohol. Physical examination is insignificant, except mild pallor. Laboratory values are:

Hb 10.8 g/dL
MCV 105 fl
Platelet count 180,000/cmm
Leukocyte count 7,500/cmm
Segmented neutrophils 68%
Bands 1%
Eosinophils 1%
Lymphocytes 24%
Monocytes 6%

Which of the following supplementations could have prevented this patient’s anemia?

A. Folic acid
B. Vitamin B12
C. Iron
D. Vitamin B6
E. Vitamin B1
This patient presents with mild megaloblastic anemia, which is most probably caused by chronic phenytoin therapy. Some anti-epileptic drugs including phenytoin, primidone and phenobarbital can cause megaloblastic anemia that is usually mild. The pathophysiology of this condition involves impaired absorption of folic acid in the small intestine. Folic acid supplementation can effectively prevent this condition.

Several other drugs can also cause folic acid deficiency:
1. Trimethoprim: It inhibits dihydrofolate reductase and in high doses can cause megaloblastic pancytopenia.
2. Methotrexate: Also inhibits dihydrofolate reductase. Folinic acid (leucovorin) is indicated to reverse the chemotherapeutic anti-folate effect of methotrexate.

Although vitamin B12 deficiency (choice B) also causes megaloblastic anemia the metabolism of this substance is not impaired by phenytoin.

Vitamin B6 (choice D) is antagonized by some drugs used to treat tuberculosis, especially isoniazid.

Iron deficiency anemia (choice C) manifests as microcytic, not megaloblastic anemia.

Vitamin B1 deficiency (choice E) does not cause anemia.

Educational Objective:
The most common cause of folic acid deficiency is nutritional due to poor diet and/or alcoholism. Folic acid deficiency can be caused by some drugs; these drugs can impair the absorption of folic acid (e.g., phenytoin) or antagonize its physiologic effects (e.g., methotrexate, trimethoprim).

*Extremely high yield question for USMLE!!!
A 45-year-old male is being treated for Burkitt's lymphoma, with combination chemotherapy and allopurinol. On the 3rd day of treatment, he started to have decreased urine output and increased BUN and creatinine. Patient was started on vigorous hydration. On EKG he has a prolonged QT interval. You are suspecting a possibility of tumor lysis syndrome. You order complete metabolic profile. Which of the following sets of results are expected in this patient?

Calcium, Phosphate, Potassium, Uric Acid


A. Increased, Increased, Increased, Increased
B. Decreased, Decreased, Decreased, Decreased
C. Decreased, Decreased, Increased, Increased
D. Decreased, Increased, Increased, Increased
E. Decreased, Increased, Decreased, Decreased
Tumors, which have high cell turnover, are frequently associated with tumor lysis syndrome. Introduction of allopurinol has reduced the acute urate nephropathy to a large extent, but still there is a possibility. The tumors most often associated with this syndrome are the poorly differentiated lymphomas, such as Burkitt's lymphoma, and the leukemias, particularly ALL and less often AML. Hypocalcemia is an important component of Tumor Lysis Syndrome. It is believed to be due to the release of intracellular products by cell lysis. There are a number of metabolic abnormalities associated that include hyperphosphatemia, hypocalcemia, hyperkalemia, and hyperuricemia. Both potassium and phosphate are intracellular ions, and the breakdown of the cells will release excess amounts of these. Released phosphate binds calcium and causes hypocalcemia. Degradation of cellular proteins causes elevation of uric acid. The syndrome needs prompt identification and immediate treatment as it may result in fatal arrhythmias, acute renal failure, and even sudden death.

Educational Objective:
Try to understand the pathophysiology of the tumor lysis syndrome and know how to diagnose and treat. This is a very favorite topic for board exams.
A 46-year-old woman has had worsening fatigue, headache, palpitations and shortness of breath on exertion for 1year. 6 months ago these complaints were attributed to her menopause. She has taken multivitamin and mineral tablets. For the recent 1 month she has had chest pain and nagging cough and her dyspnea has rapidly increased. 3 days ago she coughed up traces of blood. Her vital signs are: PR: 110/min; BP: 170/70mm of Hg; RR: 23/min; Temperature: 36.6C(97.8F). Physical exam is remarkable for very pale skin and 3/6 mid-systolic murmur in second right intercostal space. Lungs are clear to auscultation. Chest x-ray reveals right, upper mediastinal, 5cm mass with clear, lobulated borders.

The laboratory results are:

RBC 2.0
ESR 30
Hb 6.5
Hct 21
MCH 32.3
MCHC 32%
MCV 98
WBC 8,500 with normal differential
Reticulocyte count 0.1% of red cells
Platelet count 250.000

Iron 100 mcg/dL
Total iron-binding capacity 300 mcg/dL
Ferritin 100 ng/mL

ALT 8
AST 12
Bilirubin total//direct 0.8 / 0.1
LDH 70
Protein, Total 6.5
Albumin 3.7
Calcium 9.1 mg/dL

Most likely the mass is:


A. Non-Hodgkin’s lymphoma
B. Tuberculoma
C. Thymoma
D. Plasmacytoma
E. Neuroblastoma
This patient probably has pure red cell aplasia. Everything except RBC, ESR, Hb, Hct, and reticulocyte count are normal. When this disorder is associated with tumor, it most often is Thymoma.

Other secondary anemias are:
1) Autoimmune hemolytic: may be associated with various tumors. Reticulocytes, bilirubin and LDH would be increased.
2) Anemia of chronic disease: Iron and Total iron-binding capacity would be decreased. Ferritin would be increased.
3) Due to infiltration of bone marrow from tumor: All blood cell lines would be decreased. Immature cells would be present in the peripheral blood.
4) Due to Plasmocytoma or Multiple myeloma: Protein and calcium would be increased. Pure red cell aplasia is not a feature.

Educational Objective:
Thymoma is associated with pure red cell aplasia
A 55-year-old male comes to you with complaint of fatigue for the past month. He also complains of occasional heartburn. His past medical history is significant for hepatomegaly, secondary to fatty liver. He has been drinking 3-4 beers per day for the past 30 years. He denies smoking. His physical examination reveals pallor of skin and mucous membranes, and mild hepatomegaly. His laboratory report shows:

Hb 8.5 g/dL
WBC 8,000/cmm
MCV 110 fl
Platelets 150,000/cmm
Blood glucose 118 mg/dL
BUN 16 mg/dL
Serum creatinine 1.0 mg/dL

What is the most likely cause of anemia in this patient?


A. Vitamin B-12 deficiency
B. Folate deficiency
C. Chronic blood loss from peptic ulcer
D. Anemia of chronic disease
E. Thiamine deficiency
Decrease in serum hemoglobin concentration and elevated MCV in this patient is suggestive of megaloblastic anemia. Megaloblastic anemia is most commonly due to folate or vitamin B-12 deficiency. Folate deficiency is the most common cause of megaloblastic anemia in chronic alcoholics.

Alcohol abuse is the most common cause of nutritional folic acid deficiency in the United States, leading to a megaloblastic anemia. Alcohol abuse causes folate deficiency by impairing its enterohepatic cycle and inhibiting its absorption. Alcoholics can develop megaloblastic anemia within 5 to 10 weeks, as body stores of folate are limited.

Other hematological manifestations of alcohol abuse includes iron deficiency from chronic blood loss (microcytic anemia), anemia of chronic disease (normocytic or microcytic anemia), thrombocytopenia, and macrocytosis (may appear even before the development of anemia).

(Choice A) Vitamin B-12 deficiency can also cause megaloblastic anemia, but given the history of chronic alcoholism in this patient, folate deficiency is more likely.

(Choice C) Chronic blood loss would cause iron deficiency or microcytic anemia, which is not the case here.

(Choice D) Anemia of chronic disease usually causes normochromic, normocytic anemia.

(Choice E) Thiamine deficiency is common in alcoholics, however it is not associated with megaloblastic anemia; rather it is associated with Wernicke’s encephalopathy.

Educational Objective:
Alcohol abuse is the most common cause of nutritional folate deficiency in United States and would cause megaloblastic anemia.
A father brings his 7-year-old boy to the office because “his eyes are getting yellow”. The boy has no other complaints. However, he is feeling tired and does not feel like going to the next Steelers game. There is a history of diabetes and lung cancer in the family. His vitals are stable, but he has pallor, jaundice, and splenomegaly. His labs revel:

WBC 10,000cmm
Hemoglobin 10 gm/dL
Hematocrit 28%
RBC count 3.5 million
Platelets 240,000cmm
Reticulocyte count 4%

RBC indices were as follows:

MCV 90 fL
MCHC 38%
MCH 28 pg

Peripheral smear revealed anisocytosis, some spherocytosis, and polychromatophilia. Coomb’s test was negative.

LFTs showed:

Albumin 5.2 g/dL
AST 47 U/L
ALT 52 U/L
Alkaline phosphatase 120 U/L
Total bilirubin 3.0 mg/dL
Direct bilirubin 0.2 mg/dL

What is the next step?


A. Bone marrow biopsy
B. Osmatic fragility test
C. Sugar water test
D. Hemoglobin electrophoresis
E. Serum B12 level
In a patient with increased reticulocyte count, increased bilirubin, negative Coombs test and spherocytes on peripheral blood films, the diagnosis of Hereditary Spherocytosis (HS) is top on the list. HS is an autosomal dominant trait and is the most common hereditary hemolytic anemia in white population. Anemia may manifest itself from early infancy to later age. There is congenital RBC membrane defect in HS resulting in extravascular hemolysis occurring only in the presence of spleen. The range of symptoms varies widely from asymptomatic to severe disease. The patient may be severely anemic with jaundice and splenomegaly on presentation. Peripheral blood smear demonstrate spherocytes. MCV is normal or slightly decreased while MCH is normal but the MCHC is generally greater than 36%. The osmotic fragility test is the next best test for the diagnosis of the patient.

Bone marrow biopsy is not required. The disease can be easily diagnosed by other hematological parameters.

Sugar water test is done for the diagnosis of Paroxysmal Nocturnal Hemoglobinuria (PNH).

Hemoglobin electrophoresis is not useful as HS is not one of the hemoglobinopathies like sickle cell disease or thalassemia.

Serum B12 levels are done for macrocytic anemias, which are characterized by elevated MCV (>100).

Educational Objective:
Know about the classical picture of chronic hemolytic states and hereditary spherocytosis.
Mr. Fernando, a 55-year-old laboratory technician of Hispanic origin, presents with increasing fatigue and generalized weakness, for the last 2 years. He also has chronic pain in the lower back and legs when he walks. He smokes 2-3packs/day for 30 years and drinks only on weekends. His mother has diabetes, while father died of stroke. On exam he has pallor. Neurological examination is completely normal. Complete workup reveals:

Sodium 138 mEq/L
Potassium 4.1 mEq/L
Bicarbonate 24 mEq/L
Blood urea nitrogen 29 mg/dL
Creatinine 1.8 mg/dL
Calcium 11 mg/dL
Glucose 106 mg/dL
WBC 8,600/cmm
Hemoglobin 8.9
Hematocrit 31%,
Platelets 201,000/cmm
ESR 100

What is the next step?


A. Antinuclear antibody, and Anti-smith antibodies
B. Serum immunoelectrophoresis
C. Bone marrow biopsy
D. Kidney biopsy
E. Bone scan
The patient is most probably suffering from Multiple Myeloma (MM). MM presents in old age and back pain is the most common manifestation. Recurrent infections are also common. Complications include renal failure, hypercalcemia, and hyperviscosity syndrome. Laboratory findings include anemia, an increased ESR (Often more than 100), and the presence of Bence Jones proteins in urine. The leukocyte count can be normal, although about one third of patients suffer from leukopenia. MM is a malignant plasma cell disease, which releases monoclonal proteins, resulting in increased plasma volume. Serum immunoelectrophoresis demonstrates a characteristically abnormal M-spike. In majority of the cases this is due to IgG antibody produced by abnormal plasma cells.

ANA, and Anti smith antibodies are done if SLE or other autoimmune disease is suspected. It is very unlikely in an elderly patient. However, it should be considered if the myeloma workup is negative.

In MM bone marrow is typically involved though bone marrow biopsy is not the next best step in this scenario. Invasive procedures should be reserved for clear indications. If this patient has abnormal immunoelectrophoresis then the bone marrow biopsy will be next step.

Renal biopsy may be needed later on if the physician thinks that it is not due to MM.

MM causes lytic lesion in bone but they are not seen on bone scan because there is no associated new bone formation. Skeletal survey would have been more appropriate. Do not order bone scan if you are suspecting MM.

Educational Objective:
Elderly patient with anemia, renal failure, and hypercalcemic should make you think of multiple myeloma.
Mr. Glenn, a 26-year-old man, comes with his girlfriend to ER with a very high fever. He finished his second cycle of BEP (bleomycin, etoposide, and cisplatin) chemotherapy for metastatic seminoma 4 days ago. Other than his fever, the patient has no complaints. He denies any chest pain, cough, diarrhea or any rash. He has stopped smoking since he was diagnosed with the "deadly disease", but drinks alcohol "off and on". His Temperature: 38.9C[102F); BP: 118/70 mm Hg; PR: 102/min; RR: 19/min. On examination he appears pale. He is wearing a wig and does not have eyebrows or eyelashes. Blood tests were ordered and the results are

WBC 690/mm3 with 9% neutrophils
Hemoglobin 8.6
Hematocrit 25%,
Platelets 74,000/mm3

What is the next best step in the management of this patient?


A. Give acetaminophen
B. Admit the patient; obtain blood cultures and give IV cefepime
C. Admit the patient; obtain blood cultures and give vancomycin
D. Order blood cultures and then decide treatment according to the results
E. Give blood, platelet, and G-CSF transfusion
F. Give gentamicin
Fever in a neutropenic patient is a medical emergency. Febrile neutropenia is defined as a single temperature of >38.3ºC (101.3ºF), or a sustained temperature >38ºC (100.4ºF) for more than one hour. However, patients who are on corticosteroids may not develop fever. Neutropenia is defined as an absolute neutrophil count (ANC) <500 cells/mm3. Disruption of the skin and mucosal barrier resulting from the chemotherapy often results in seeding of bacteria into the blood stream (bacteremia) and the most common site of mucositis is in gastrointestinal tract. Chemotherapy also results in impaired immunologic function.

The frequently identified organisms in febrile neutropenic patients are Gram negatives, particularly P. aeruginosa. But there has been a recent increase in Gram positive infections for several reasons. Although GI tract has abundant anaerobic organisms, anaerobic coverage is not indicated in the initial empirical antibiotic regimen. Anaerobic coverage is indicated if there is any evidence of necrotizing mucositis, periodontal abscess, perirectal abscess/cellulitis, intraabdominal or pelvic infection, typhilitis (necrotizing neutropenic colitis), or anaerobic bacteremia.

The empirical antibiotic choice should cover the Gram-negative organisms especially Pseudomonas. Empiric treatment with ceftazidime/cefepime is the first step in these patients. The patient should be admitted and his response monitored. Third-generation cephalosporins are active against Pseudomonas aeruginosa, the Enterobacteriaceae, and many gram-positive organisms so they are used as first choice drugs. Always blood cultures should be obtained prior to initiating antibiotics.

The benefit of colony stimulating factors as an adjuvant to antibiotic therapy in an uncomplicated febrile neutropenic patents is not proven and is not indicated. However, it is considered in patients with predictive poor outcome such as patients with ANC <100/µL, uncontrolled primary disease, pneumonia, hypotension, multiorgan dysfunction, or invasive fungal infection.

Just giving acetaminophen will not be useful in this patient because of serious drop of neutrophils and very high fever.

It is always useful to try to find the cause of fever by ordering blood cultures but the cause is not identified in most of the post chemotherapy neutropenic patients. Many of the organisms responsible for infection are in fact endogenous normal flora.

The patient is anemic and may need some sort of effort to have his hemoglobin restored but this is not the issue right now.

Aminoglycosides are contraindicated in patients receiving cisplatin chemotherapy because of the potential for significant nephrotoxicity. Gentamycin or vancomycin alone doesn’t cover the pseudomonas.

Educational Objective:
Febrile neutropenia is a medical emergency; patient should be admitted in hospital and started on broad-spectrum antibiotics preferably 3rd generation cephalosporins after obtaining blood cultures.
Mr. Jefferson is a 62-year-old African-American librarian, who comes with complaints of increasing fatigue and weakness for the last 4 months. There is also pain in his back and arms, which is dull pain and gets worse with walking. He denies any numbness or paresthesia. On examination, he appears pale. There is tenderness around lumbar spine. CBC is ordered and the results are:

WBC 8,600mm3 with normal differential
Hemoglobin 8.6
Hematocrit 29%,
Platelets 164,000mm3

Morphology was significant for a rouleaux appearance of RBCs. ESR was 55mm/hr. Bence jones protein was identified in the urine. Which of the following is expected on bone marrow examination of the patient?


A. Over proliferation of plasma cells
B. Increased marrow cellularity with megakaryocytic hyperplasia
C. Hypoplastic fat filled marrow with no abnormal cells
D. Normocellular bone marrow
E. Hypocellular and fibrotic bone marrow
The patient has Multiple Myeloma (MM). A patient presenting with the above signs and symptoms with Bence Jones proteins bears all the hallmarks of MM. MM is caused by the proliferation of a single transformed plasma cell usually producing IgG. Following are the classical findings in MM

1.Lytic bone lesions
2.Marrow plasmacytosis
3.Urine and serum monoclonal proteins

MM presents in old age and back pain is the most common manifestation. Recurrent infections are also common. Complications include renal failure, hypercalcemia, and hyperviscosity syndrome. The complete workup consists of CBC with differential and morphology, serum electrolytes, kidney and Liver screening profiles, skeletal survey, and serum electrophoresis, and bone marrow biopsy.

Increased marrow cellularity with megakaryocytic hyperplasia is seen in essential thrombocytopenia.

Aplastic anemia shows hypoplastic fat-filled marrow with no abnormal cells.

Hypocellular and fibrotic bone marrow is seen in myelofibrotic disorders.

Educational Objective:
Multiple myeloma is a plasma cell disorder and often presents with lytic lesions, hypercalcemia, and renal failure.
Mr. Jefferson is a 62-year-old African-American librarian, who comes with complaints of increasing fatigue and weakness for the last 4 months. There is also pain in his back and arms, which is dull pain and gets worse with walking. He denies any numbness or paresthesia. On examination, he appears pale. There is tenderness around lumbar spine. CBC is ordered and the results are:

WBC 8,600mm3 with normal differential
Hemoglobin 8.6
Hematocrit 29%,
Platelets 164,000mm3

Morphology was significant for a rouleaux appearance of RBCs. ESR was 55mm/hr. Bence jones protein was identified in the urine. Which of the following is expected on bone marrow examination of the patient?


A. Over proliferation of plasma cells
B. Increased marrow cellularity with megakaryocytic hyperplasia
C. Hypoplastic fat filled marrow with no abnormal cells
D. Normocellular bone marrow
E. Hypocellular and fibrotic bone marrow
The patient has Multiple Myeloma (MM). A patient presenting with the above signs and symptoms with Bence Jones proteins bears all the hallmarks of MM. MM is caused by the proliferation of a single transformed plasma cell usually producing IgG. Following are the classical findings in MM

1.Lytic bone lesions
2.Marrow plasmacytosis
3.Urine and serum monoclonal proteins

MM presents in old age and back pain is the most common manifestation. Recurrent infections are also common. Complications include renal failure, hypercalcemia, and hyperviscosity syndrome. The complete workup consists of CBC with differential and morphology, serum electrolytes, kidney and Liver screening profiles, skeletal survey, and serum electrophoresis, and bone marrow biopsy.

Increased marrow cellularity with megakaryocytic hyperplasia is seen in essential thrombocytopenia.

Aplastic anemia shows hypoplastic fat-filled marrow with no abnormal cells.

Hypocellular and fibrotic bone marrow is seen in myelofibrotic disorders.

Educational Objective:
Multiple myeloma is a plasma cell disorder and often presents with lytic lesions, hypercalcemia, and renal failure.
Mr. Miranda, a 50-yr-old African-American man is feeling weak, for the past year or so. He gets tired early and has lost his “humor”. He is also finding it difficult to remember things. He is jobless and is living on social security benefits. He denies smoking or alcohol use but admits being charged with driving under intoxication three times. His father and mother died of old age. He shares his room with four of his friends. His vitals are normal, but there is pallor on examination. CBC is ordered and the results are:

WBC 5,500mm3
Hemoglobin 7 mg/dL
Hematocrit 24%,
Platelets 196,000mm3
RBC count 1.7 million
MCV 119
MCH 36
MCHC 28%
Reticulocyte count 0.4%

Peripheral smear showed anisocytosis, poikilocytosis, and basophilic stippling. What is the next step?


A. Iron studies
B. Osmotic fragility test
C. Serum B12 and folate levels
D. Sugar water test
E. Bone marrow biopsy with prussian blue staining
F. Blood lead levels
The patient has macrocytic anemia which is characterized by elevated MCV, 90% of the times elevated MCH, and normal MCHC. Macrocytic anemias are classified in 3 major categories:
1.Vitamin B12 deficiency
2.Folic acid deficiency
3.Miscellaneous like hypothyroidism, severe liver disease, and antimetabolites

The typical findings on the peripheral blood smear are hypersegmented neutrophils with a mean lobe count greater than 4, and macro ovalocyte RBCs. Anisocytosis, poikilocytosis, and basophilic stippling is also seen. The reticulocyte count is decreased although the bone marrow is hypercellular. Neurological deficits are mainly seen with pernicious anemia, which is the most important cause of vitamin B12 deficiency. Supporting laboratory assays of folic acid and vitamin B12 can differentiate between pernicious anemia and folic acid deficiency. This is important because the treatment regimen is different for the two types.

The difference between the two types is tabulated below

Laboratory Tests Pernicious Anemia, Folic Acid Def.
1. Serum B12: Decreased, Normal
2. Serum folic acid: Normal, Decreased
3. Serum LDH: Increased, Normal
4. Achlorhydria: Present, Absent

5. Schilling test: Positive, Negative
6. Methyl malonyl CoA in urine: Present, Absent
7. Neurological signs: Present, Absent

Iron studies are the tests done in microcytic anemia, where the MCV is <80.

The osmotic fragility test is the done to determine the diagnosis of hereditary spherocytosis.

Sugar water test is done for the diagnosis of Paroxysmal Nocturnal Hemoglobinuria (PNH).

Bone marrow biopsy is not of diagnostic significance in this condition. Prussian blue staining is used for sideroblastic anemia.

In lead poisoning you see iron deficiency anemia picture (MCV<80). Basophilic stippling is a distractor here.

Educational Objective:
Patients with high MCV should be tested for vitamin B12 and folic acid levels. Know how to differentiate between various causes of megaloblastic anemia.
Rosy, a 16-year-old girl is brought by her mother because she is having menses for 10-12 days for the past 6 months. She is concerned because her brother has some blood disease and he needed several blood transfusions after a cholecystectomy. The patient is otherwise healthy and denies any smoking or alcohol use. Her vitals are, Temperature: 36.8C(98.3F); BP: 124/78 mm Hg; PR: 89/min; RR: 18/min. There is marked pallor under the mucous membranes. Blood work along with coagulation profile is ordered and results are:

WBC 8,600cmm
Hemoglobin 9.0gm/dL
Hematocrit 26%,
Platelets 96,000mm3
Bleeding Time 8 min
PT 12.1s
APTT 30s

Giant platelets are seen on the peripheral smear. Looking at the bleeding time the physician asks qualitative platelets tests, which showed

Platelet aggregation studies with ADP Normal
Platelet aggregation studies with ristocetin Subnormal
VWF Level Normal

What is the most probable diagnosis?


A. Thrombotic thrombocytopenic purpura
B. Von Willebrand disease
C. Idiopathic thrombocytopenic purpura
D. Glazmann’s Thrombasthenia
E. Chédiak - Higashi Syndrome
F. Bernard-Soulier syndrome
The patient is suffering from Bernard –Soulier syndrome. The hallmark of this disease is the Giant platelets. There is mild thrombocytopenia but the major defect is of membrane glycoprotein Ib. This defective platelet membrane lacks the receptor site for vWF so the platelets can’t adhere to vascular subendothelium. Typically greater than expected bleeding is noted for the degree of thrombocytopenia. Platelets from these patients do not aggregate in the presence of normal vWF and ristocetin because of the decrease or abnormality in the GP Ib. This may result in increased bleeding and surgical interventions need to be supported by platelet concentrates.

Thrombotic thrombocytopenic purpura is a fulminant disorder and results in marked decrease in platelet count.

vWF Level is decreased in Von Willebrand disease.

Idiopathic thrombocytopenic purpura is an autoimmune disease when antibodies are formed against platelets resulting in decreased platelet count.

Chédiak - Higashi Syndrome is a storage granulocyte abnormality resulting in hepatosplenomegaly, lymphadenopathy, anemia, thrombocytopenia, roentgenological changes of bones, lungs and heart, skin and psychomotor abnormalities, and susceptibility to infection, usually resulting in death in childhood.

Glazmann’s Thrombasthenia is a qualitative platelet defect with normal platelet count but prolonged bleeding time and abnormal aggregation test with ADP. They have normal platelet count. This patient has thrombocytopenia.

Educational Objective:
Know rare bleeding disorders. They are not common in real life but frequently gate crashes into the exams.
Stupor Junior is an 8-month-old African-American child referred by a nurse practitioner because she observed that mucous membranes are pale. She also noted that the child was irritable and listless. Stool examination is negative for occult blood, ova and parasites. She also ordered some basic tests and the results are:

WBC 6,600/cmm
Hemoglobin 5.8
Hematocrit 26%,
RBC cont 3.5 million
Platelets 116,000/cmm

The child appears to be pale which is consistent with his blood work. Further blood tests are done which reveal:

MCV 68 fL
MCH 16.5 pg
MCHC 25%
Reticulocyte count 0.6%
Serum Iron 40µg/dl
TIBC 460µg/dl
Percent saturation of transferrin 8.7%
Total serum bilirubin 0.9mg/dl

The peripheral blood smear shows marked anisocytosis, microcytosis, hypochromia, and poikilocytosis. What is the diagnosis?


A. Iron deficiency anemia
B. Sideroblastic anemia
C. Dimorphic anemia
D. Megaloblastic anemia
E. Pernicious anemia
The child has hypochromic, microcytic erythrocytes in a peripheral blood film suggestive of an iron deficiency anemia. RBC indices reveal a decreased MCV and MCH. On serum iron studies, a decreased serum iron and percent saturation (Serum Iron /TIBC) along with an increased TIBC is seen, which is consistent with iron deficiency anemia. The serum bilirubin level is normal ruling out hemolysis. However, no evidence of bleeding and parasitic infections was detected. So, the most probable cause of this iron deficiency is inadequate diet. Small children are most frequent victim of this unfortunate disorder. With a diet consisting largely of milk, and unsupplemented with fortified food products during the early years of development, iron deficiency anemia is a frequent by product.

Serum iron is increased while TIBC may be normal in sideroblastic anemia.

In dimorphic anemia two distinct forms of red cells are circulating.

Megaloblastic anemia results in elevated MCV, elevated MCH, and normal MCHC.

Pernicious anemia is a cause of megaloblastic anemia due to absence of intrinsic factor secondary to gastric atrophy.

Educational Objective:
Know the laboratory findings in various types of microcytic anemias.
A 25-year-old male presents to his physician with dyspnea and fatigue for the last few weeks. He is a non-smoker and there is no family history of asthma or any blood disorder. He does not take any medications. Examination shows pallor, scleral icterus and splenomegaly.
Lab tests show

Hematocrit 20%
WBC-count 10,000/micro-L
Platelet count 180,000/miro-L
Total Bilirubin 7 mg/dL
Direct bilirubin 0.6 mg/dL
BUN 10mg/dL
Serum creatinine 0.7 mg/dL
Serum LDH 400 U/L(normal value is 80-280 U/L)
Serum haptoglobin 160mg/dL (normal value is 30-220 mg/dL)
Reticulocyte count 8 %
Peripheral blood smear shows spherocytes with central pallor.
Osmotic fragility testing and direct Coomb’s test are positive.

Based on these findings, what is the most likely diagnosis?


A. Hereditary spherocytosis
B. Autoimmune hemolytic anemia
C. Paroxysmal nocturnal hemoglobinuria
D. Sickle cell anemia
E. G6PD deficiency
Findings of pallor and jaundice, elevations in the levels of indirect bilirubin, reticulocyte count and serum LDH are all indicative of hemolytic anemia. Hemolysis may be either intra-vascular or extra-vascular. Splenomegaly is associated with extra-vascular hemolysis while decreased levels of serum haptoglobin are caused by intra-vascular hemolysis. This patient is most likely suffering from extra-vascular hemolysis.

Hereditary spherocytosis is an important cause of extra-vascular hemolysis. It is an autosomal dominant condition so that there will be a positive family history. Peripheral blood picture shows spherocytes with loss of central pallor. MCHC is elevated. Osmotic fragility test is positive but it is also positive when spherocytosis is caused by acquired causes like autoimmune hemolytic anemias.

Autoimmune hemolytic anemia is an acquired disorder and may produce spherocytes on peripheral blood smear. Usually direct Coomb’s test is positive. But direct Comb’s test is not always positive in all cases of autoimmune hemolytic anemias and in such cases micro-Coomb’s test is performed to confirm the diagnosis.

Paroxysmal nocturnal hemoglobinuria causes intravascular hemolysis with decreased haptoglobin levels.

Sickle cell anemia is a hereditary disorder and there will be a positive family history. Peripheral blood smear shows sickle-shaped cells and there will be autosplenectomy by this age.

G6PD deficiency is another hereditary cause of hemolytic anemia. It is an X-linked condition and hemolytic episodes are triggered by drug ingestion. Peripheral blood smear may show cells with Heinz bodies.

Educational Objective:
Spherocytes on peripheral blood smear may also be seen with autoimmune hemolytic anemias and negative family histroy and positive coomb's test distinguish this disorder from hereditary spherocytosis.
A 34-year-old male, who recently immigrated from Asia, presented with a two months history of exertional shortness of breath and easy fatigability. On examination, he has severe pallor. Peripheral smear showed macrocytosis with hypersegmented polymorphonuclear neutrophils. Because of religious beliefs, the patient was completely converted in to a vegetarian diet for the last one year. He has been taking isoniazid and rifampicin for his tuberculosis, which was diagnosed four months ago. His WBC and platelet counts are with in normal limits. This patient’s most likely problem is due to?


A. B12 deficiency because of vegetarian diet.
B. B12 deficiency because of pernicious anemia.
C. Drug induced B12 deficiency.
D. Myeloproliferative disorders.
E. Chronic myeloid leukemia.
Strict vegetarian diet can cause B12 deficiency. Usually the total body vitamin B12 stores are available for three to four years. This patient has been on a vegetarian diet only since one year so the diet-related B12 deficiency is very unlikely in this patient. Antituberculous drugs like INH can cause B6, that is, pyridoxine deficiency, not B12 deficiency. This patient’s most likely diagnosis could be due to pernicious anemia, which is an autoimmune diseases resulting from destruction of parietal cells. The next step should be measuring intrinsic factor antibody and B 12 levels.

Normal WBC and platelet count is unlikely with myelodysplastic syndromes and chronic myeloid leukemia. CML usually have elevated WBC count, splenomegaly and systemic symptoms. Myelodysplastic syndromes usually present with pancytopenia.

Educational Objective:
Recognize the pathology of B12 deficiency anemia. B12 stores are available for 3-4 years.
A 42-year-old white female, who has a long history of dysfunctional uterine bleeding, presented with exertional shortness of breath. On examination, she appears very pale and there is a pulmonic flow murmur heard over the second intercostal space. Her vitals are: BP: 130/80mm of Hg; HR: 80/min and regular; RR: 16/min; Temperature 36.7C(98F). Initial evaluation reveals hemoglobin of 8.2 gm/dl. WBC count is within normal limits. You are suspecting an iron deficiency anemia secondary to excessive bleeding. Which one of the following studies is most definite for the diagnosis of iron deficiency anemia?


A. A low serum ferritin level.
B. Increased total iron binding capacity.
C. A low serum iron concentration.
D. Bone marrow iron staining.
E. Total iron content of the gastric epithelial cells.
A low serum iron concentration is really nonspecific; it is low in iron deficiency anemia but it is also usually low in patients with anemia of chronic disease. The total iron binding capacity is usually elevated in iron deficiency anemia. However, many iron deficient patients may have normal TIBC. A low serum ferritin level almost always reflects iron deficiency anemia. However, serum ferritin can be normal or even elevated in patients with iron deficiency anemia as an acute phase reactant, but the bone marrow iron stain is the most definite way to diagnose an iron deficiency. But it is usually not required to make the diagnosis in real clinical practice. Since iron stores are depleted when iron deficiency anemia is present, and are usually normal or elevated in patients with microcytic anemia due to other causes like thalassemia.

Educational Objective:
Bone marrow iron stain is the most definite way to diagnose iron deficiency anemia.
A 44-year-old, African American, woman comes with complaints of loss of weight and blood in stools, for the last 1 year. Her mother is on chemotherapy for colon carcinoma. Her maternal uncle also had colon cancer, as did her first cousin who died of colon cancer at the age of 46. She is very worried that she might have the same cancer. According to her history, she falls within the criteria for Lynch syndrome. (HNPCC: Hereditary Nonpolyposis Colorectal Cancer). Apart from complete work-up for colon cancer, the patient should be evaluated for which of the following condition?


A. Pancreatic carcinoma
B. Hepatic carcinoma
C. Pseudomembranous colitis
D. Diverticulitis
E. Endometrial carcinoma
The patient has a strong family history of colon cancer and falls within the criteria for Lynch syndrome (HNPCC). The International Collaborative Group on Hereditary Nonpolyposis Colorectal Cancer established research criteria for the diagnosis of HNPCC (known as the Amsterdam Criteria I):

1. At least three relatives with colorectal cancer, one of whom must be a first degree relative of the other two
2. Involvement of two or more generations
3. At least one case diagnosed before age 50
4. Familial adenomatous polyposis has been excluded

HNPCC was classically divided into two subgroups on clinical grounds:
1. Hereditary site specific colon cancer or Lynch syndrome I
2. Cancer family syndrome or Lynch syndrome II

Lynch syndrome II is differentiated by a high risk of extra colonic tumors, the most common of which is endometrial carcinoma, which develops in up to 43 % of females in affected families.

Educational Objective:
Lynch syndrome II is differentiated by a high risk of extra colonic tumors, the most common of which is endometrial carcinoma, which develops in up to 43 % of females in affected families.
A 51-year-old man comes to you for annual examination. He is healthy and does not have any complaints. His diet is normal and he exercises regularly. He has some stress at work but he is strong to fight it out. His father died of heart attack 4 years ago while mother has Alzheimer’s disease. His brother is recently diagnosed with colonic polyps. His vitals are stable and hematology and chemistries are within normal limits of his age. You perform fecal occult blood test (FOBT) and you get a positive sample. What is the next best step in the management of this patient?


A. Perform screening sigmoidoscopy
B. Perform double contrast barium enema
C. Do colonoscopy
D. Advise him to change his diet and examine him next week
E. Order abdomen radiograph for air under the diaphragm
Expert groups recommend four options and their combinations: FOBT, sigmoidoscopy, FOBT + sigmoidoscopy, double contrast barium enema, and colonoscopy for colorectal cancer screening. In practice, FOBT is the most commonly used screening test followed by sigmoidoscopy, although screening colonoscopy is gaining popularity. Offer fecal occult blood screening each year after 50yr, using a guaiac-based test with dietary restriction or an immunochemical test without dietary restriction. Two samples from each of three consecutive stools should be examined. Patients with a positive test on any specimen should be followed up with colonoscopy.

Sigmoidoscopy and double contrast barium enema are also useful tools in screening for colorectal cancer, once you get FOBT positive. However, the best way to go is to perform colonoscopy. Colonoscopy has the added benefit that the lesions can be removed during the same procedure. Furthermore, colonoscopy finds a significant number of proximal lesions that would otherwise be missed by performing sigmoidoscopy.

Changing the diet or abdominal radiograph is not the next best step in this patient especially with a family history of clonic polyps.

Educational Objective:
Fecal occult blood testing (FOBT) is the most commonly used screening test for colon cancer. Patients with positive FOBT test should be followed up with colonoscopy.
A 56-year-old white male patient is being evaluated for isolated Ig G elevation with a serum level of 2400 (Normal is700 to 1500). Other paraproteins are in normal limits. Bone marrow biopsy revealed 2% of plasma lesions. His serum Ca is 8mg/dl, BUN is 24mg/dl and creatinine is 1.2mg/dl. He was admitted for pneumonia. Pneumonia resolved completely with levofloxacin. What is the most appropriate next step in the management of this patient?


A. Reassurance and regular follow up
B. Melphalan and prednisolone
C. Interferon
D. Combination chemotherapy
E. Autologus bone marrow transplantation
This patient is having a Monoclonal gammopathy of undetermined significance (MGUS). MGUS should be differentiated from multiple myeloma. MGUS is characterized by the absence of symptoms; M component (IgA, or IgG, or IgM) <3000mg/dL; fewer than 10 % plasma cells in the bone marrow, and no lytic lesions, anemia, hypercalcemia, or renal insufficiency. MGUS patients doesn’t require any treatment. MGUS patients should be told that the risk of development of a serious disease is only 25 % and usually constitutes a late event. However, they should also know that the evolution from MGUS to multiple myeloma, which occurs at a rate of 1 %/yr, may be abrupt. So all patients should be advised to obtain medical evaluation promptly if clinical symptoms occur.

All other options are used for multiple myeloma patients.

Educational Objective:
Patients with Monoclonal gammopathy of undetermined significance (MGUS) are best treated with reassurance and regular followup.
A 56-year-old woman comes to you because she has noticed an ulcer on her left elbow. She had an accident when she was 13-year-old. She burned her left elbow badly, while learning how to cook pasta. The ulcer is a persistent, scaly red patch with irregular borders that sometimes crusts or bleeds. She works as a public relations officer in a large marketing company. She is only a social smoker and doesn't like drinking alcohol. On examination she has a big scar on her left elbow with a firm, nontender, reddened, non-healing indolent, 3cm ulcer in the center. What is the next best step in the management of this patient?


A. Surgical excision
B. Punch biopsy
C. Observation for 3 weeks with antibiotics
D. Chemotherapy
E. Local radiation
This is Squamous cell carcinoma, which first has to be diagnosed by punch biopsy. Any patient who has long history of scar, and develops a non-healing, painless, bleeding ulcer should be considered for carcinoma most likely squamous cell, until proven otherwise.

Squamous cell carcinoma involves cancerous changes of the cells of the middle portion of the epidermal skin layer. It is usually painless initially, but may become painful with the development of ulcers that do not heal. This cancer can affect both the skin and mucous membranes. It often occurs in sun-exposed or burned areas. Tar derivatives, such as those in tobacco smoke or other carcinogens and chronic radiation exposure, are predisposing factors. Immunosuppression in transplant patients makes them at greater risk of developing these lesions. The lesions appear as rough scaly nodules that can ulcerate and metastasize. The cancerous keratinocyte cells often extend down through the basement membrane into the dermis. After the appearance of the skin lesion has indicated a squamous cell carcinoma, a biopsy and examination of the lesion confirm the diagnosis. Surgical removal of the tumor, which may include removal of the skin around the tumor (wide excision), is the treatment of choice. The tumor may be reduced in size by radiation treatment. Chemotherapy can be used if surgery and radiation fail, but it is usually minimally effective.

Educational Objective:
Punch biopsy is used for the diagnosis of squamous cell carcinoma of skin.
A 64-year-old, African-American, male who rarely sees the doctor and doesn’t believe in screening procedures, comes to office with the complaint of severe back pain. He has not felt comfortable for the past 4 months and is having severe pain in the back. He has found it difficult to sleep. He has hypertension, for which he takes hydrochlorothiazides regularly. He thinks that he has lost some weight. On examination, his BP is 165/90mm of Hg, while rest of vitals are stable. On rectal examination you find a rock hard 2 cm nodule in prostate. Biopsy shows poorly differentiated adenocarcinoma. Bone scans shows wide spread metastasis all over his skeleton. What is the most appropriate treatment for his metastatic disease?


A. Leuprolide
B. Diethylstilbestrol (DES)
C. Flutamide
D. Radicle prostatectomy
E. Complete androgen blockade with Leuprolide and Flutamide
The patient has prostate cancer that has metastasized to bones. Prostate cancer is one of the leading causes of cancer deaths among men in USA. At the time when the cancer is in late stages, surgery is not performed unless needed for palliation.

Palliative radiation, along with anti-androgen therapy, is the treatment of choice. Anti-androgen therapy consists of Leuprolide (LHRH analogue). Flutamide is considered inferior to LHRH analogues, though some studies have shown added benefit of combining the two. In various studies the results with anti-androgen therapy are comparable to orchiectomy; however, long-term survival in orchiectomy is not yet completely studied.

Complete androgen blockade (CAB) using combination of LHRH analogue with an antiandrogen has been tested in various trials. Most studies reported a higher incidence of adverse events with CAB, because of the addition of an antiandrogen, and even the benefit is very modest. So far, there is no clear consensus on the advantage of CAB over monotherapy as first-line therapy for metastatic prostate cancer.

DES is associated with increased cardiovascular disease and is not used anymore.

Educational Objective:
Palliative radiation, along with anti-androgen therapy, is the treatment of choice for metastatic prostate cancer. Anti-androgen therapy consists of Leuprolide (LHRH analogue). Flutamide is considered inferior to LHRH analogues.
A 68-year-old man is brought to the ER by hospice nurse because he had a seizures 2 hours ago. The man is unconscious and does not respond to painful stimulus. His vitals are, Temperature: 37.9C(100.2F); RR: 23/min; PR: 96/min; BP: 140/85mm of Hg. The hospice staff has secured his airway and he is on 2L oxygen and his pulse oximetry is 91%. He is in hospice for terminal care for stage IV esophageal carcinoma. A rectal exam reveals hard, irregular surface of an enlarged prostate. His recent prostate specific antigen level is 40. He also had a part of finger removed for skin cancer 1.5 years ago. An MRI is ordered and it shows an intracranial lesion with bleeding inside, consistent with metastasis. What is the most probable diagnosis?


A. Metastatic prostatic carcinoma
B. Glioblastoma multiforme
C. Metastatic melanoma
D. Metastatic squamous cell carcinoma of the skin
E. Metastatic esophageal carcinoma
F. Metastatic basal cell carcinoma
The patient has history of surgery for a skin cancer, which was most probably malignant melanoma. Malignant melanoma is called "the fascinating disease". It can go to most unimaginable places and can even lay dormant for 15-25 years, and come back in a big way even if the primary tumor was resected. This is the case with this patient. His primary tumor was removed and now he is having explosion of metastasis in brain. Malignant melanoma has spread in many cases to all sorts of places like muscle of heart or in bile duct. Malignant melanomas are notorious to cause to bleed in side the mass in brain.

There are four tumors in the body that are not reported (almost never metastasize) to have metastasized to brain
1. Non-melanomatous skin cancer
2. Oropharyngeal cancer
3. Esophageal carcinoma
4. Prostate cancer

The patient is having esophageal cancer and may have prostate cancer but they are rarely found to be going in brain.

Metastatic brain tumors are more common than primary brain tumors.

Educational Objective:
Melanoma should be on the top of the list if a metastatic mass bleeds inside brain.
A 75-year-old Caucasian male who was diagnosed with carcinoma of the prostate presents to ER with worsening back pain for the past 2-3 days. He never had this pain before. He denies any weakness of the legs. On examination, his muscle power in the lower extremities is 4.5/5 and has brisk reflexes. Rectal sphincter tone is weak. He has point tenderness over the L5 and S1 region. His vital signs are, BP: 122/80 mm Hg, PR: 80/min, RR: 16/min and Temperature: 37C(98F). What is the most appropriate next step in management of this patient?


A. Radiotherapy.
B. Intravenous dexamethasone.
C. Immediate MRI of spine.
D. Decompression surgery of spine.
E. CT myelogram.
The patient in this vignette has clinical presentation of acute cord compression with cauda equina, most likely secondary to metastasis of prostrate cancer.

Spinal cord compression or cauda equina due to metastasis is a medical emergency and needs prompt diagnosis and treatment.

First symptom of cord compression is back pain associated with tenderness at the site of metastasis. Progressive or sudden cord compression with neurological deficit can ensue if back pain is not evaluated further.

The algorithm for management of metastatic cord compression is as follows:

Cancer patients with new or progressive back pain should have detailed neurological examination for signs of myelopathy, which if present warrants intravenous administration of steroids (dexamethasone usually) as first step to prevent any further damage. Next step would be an MRI of spine. If MRI spine is not available or contraindicated (patients with pacemaker), CT myelogram should be done. Once the diagnosis of metastatic cord compression is confirmed radiotherapy should be considered.

Educational Objective:
Acute cord compression in cancer patient is a medical emergency and needs prompt administration of intravenous steroids as the first step.
*Extremely high yield topic!!!
An 18-year-old boy comes to clinic to discuss an embarrassing problem. For the last 6 weeks he has noticed a painless mass in his right testis. This is not bothering him at all, but it is becoming larger. He is afraid that his girlfriend may notice it and find out about his other partners. He has multiple partners, and does not use condoms. He has no fever and no other medical complaints. His cousin has SLE but otherwise family history is unremarkable. On ultrasound you suspect testicular cancer and order a few tests. Results are as follows:

Alpha-fetoprotein (AFP) Increased
Beta subunit of the human chorionic gonadotropin (hCG) Normal
Placental Alkaline phosphate (PLAP) Normal
Carcinoembryonic antigen (CEA) Slightly Increased
Lactate Dehydrogenase (LDH) Increased

What could be the most likely diagnosis?


A. Seminoma
B. Embryonal carcinoma
C. Choriocarcinoma
D. Teratoma
E. Sertoli cell tumor
AFP, beta-hCG, and PLAP are tumor markers useful in diagnosis, staging, and monitoring patients with testicular cancers. AFP is never produced in seminoma. It usually indicates embryonal cell carcinoma. This is an important differentiation, as the treatment is different for seminoma and non-seminoma. AFP is not specific; it may also elevate in cancers of GIT and lungs. Following is the list of useful tumor markers for testicular carcinomas:

Seminoma: PLAP
Embryonal Carcinoma: AFP + beta-hCG (50%)
Choriocarcinoma: beta-HCG

LDH and CEA have some utility but trials have shown them to be highly non-specific, so they are rarely used.

Educational Objective:
Alpha-fetoprotein is a tumor marker of embryonal cell carcinoma.

28% of people answered this question correctly
An 18-year-old boy comes to clinic to discuss an embarrassing problem. For the last 6 weeks he has noticed a painless mass in his right testis. This is not bothering him at all, but it is becoming larger. He is afraid that his girlfriend may notice it and find out about his other partners. He has multiple partners, and does not use condoms. He has no fever and no other medical complaints. His cousin has SLE but otherwise family history is unremarkable. On ultrasound you suspect testicular cancer and order a few tests. Results are as follows:

Alpha-fetoprotein (AFP) Increased
Beta subunit of the human chorionic gonadotropin (hCG) Normal
Placental Alkaline phosphate (PLAP) Normal
Carcinoembryonic antigen (CEA) Slightly Increased
Lactate Dehydrogenase (LDH) Increased

What could be the most likely diagnosis?


A. Seminoma
B. Embryonal carcinoma
C. Choriocarcinoma
D. Teratoma
E. Sertoli cell tumor
AFP, beta-hCG, and PLAP are tumor markers useful in diagnosis, staging, and monitoring patients with testicular cancers. AFP is never produced in seminoma. It usually indicates embryonal cell carcinoma. This is an important differentiation, as the treatment is different for seminoma and non-seminoma. AFP is not specific; it may also elevate in cancers of GIT and lungs. Following is the list of useful tumor markers for testicular carcinomas:

Seminoma: PLAP
Embryonal Carcinoma: AFP + beta-hCG (50%)
Choriocarcinoma: beta-HCG

LDH and CEA have some utility but trials have shown them to be highly non-specific, so they are rarely used.

Educational Objective:
Alpha-fetoprotein is a tumor marker of embryonal cell carcinoma.
An 80-year-old man has advanced prostate cancer with bony metastasis. Now, he presents with a complaint of severe back pain that has been progressively worsening for the past two weeks. His pain is so severe that it restricts him from playing golf. Eight months ago, he underwent orchiectomy, after which he was free of bone pain until now. Physical examination reveals tenderness at two sites in the lumbar region. Radionuclide bone scan shows increased uptake in these areas. Which of the following is the most appropriate next step in the management of this patient?


A. Cervical cordotomy
B. Etidronate disodium therapy
C. Flutamide therapy
D. Hypophysectomy
E. Radiation therapy
Radiation therapy would be most appropriate for management of progressive pain in a patient with prostate cancer metastatic to bone after androgen ablation (orchiectomy). Focal external beam therapy will be an excellent choice in this patient, as metastasis is localized to few sites.

(Choice A) Cervical cordotomy is not effective for controlling axial pain or pain in the upper extremities.

(Choice B) Etidronate disodium and other bisphosphonates may be useful in patients with bony metastasis as it reduces bone resorption. It is useful in controlling chronic pain from bony metastasis, but, its onset of action is not as rapid as radiation therapy in alleviating acute bone pain.

(Choice C) Flutamide is a non-steroidal anti-androgen and it provides androgen blockade by competitively binding to dihydrotestosterone receptors. This drug, when combined with a LHRH agonist, prolongs the survival of patients with limited disease. However, there were no demonstrable benefits in patients who underwent orchiectomy, as in this patient. Estramustine, a combination of estrogen and nitrogen mustard, has a 40% response rate in castrated men, but is not well tolerated in individuals with cardiovascular or hematopoietic disorders.

(Choice D) Hypophysectomy is not effective in delaying progression or controlling pain.

Educational Objective:
Radiation therapy is useful in managing bone pain in patients with prostate cancer who have undergone orchiectomy.
Mr. Divine, a 40-year-old male, comes to the office because of progressive knee and shoulder pain. The pain is 5/10 and started 6 months ago. He has no other medical problems, except newly diagnosed diabetes. He admits to "chain smoking" for "all his life" and drinks 1-2 beer a night. He is faithful to his wife. His mother died of “blood cancer” when he was 3 year old. His father also has diabetes mellitus. His vitals are: Temperature: 37C(98.6 F); PR: 86/min; BP: 134/86mm Hg: RR: 16/min. On exam, both knee joints are slightly swollen and tender. Mild hepatomegaly is present on abdominal examination. What one of the following is an appropriate next investigation in this patient?


A. HbA1C level
B. Serum iron studies
C. Liver biopsy
D. X-ray of the shoulder and knee
E. Steroid injection of the joint
The patient has newly diagnosed diabetes, hepatomegaly, and arthropathy. This makes him a likely candidate of the syndrome of hemochromatosis. It is an autosomal recessive disorder in which intestinal iron absorption is abnormally increased leading to iron deposition in the tissue. This damages the organs especially liver, pancreas, heart, and pituitary. Clinically it presents with hepatomegaly, hyperpigmentation, diabetes (bronze diabetes), arthropathy, restrictive heart failure, and hypogonadism. It may cause cirrhosis, which has an increased risk of hepatoma. The first logical step in the diagnosis of this disease is serum iron studies. Increased serum iron, ferritin, and transferrin saturation correlates with hemochromatosis. Liver biopsy confirms the diagnosis.

HbA1C level is done to detect long term glucose control in a diabetic patient taking various medications.

X-ray of shoulder and knee may be needed but symptomatology of the patient is highly suggestive of hemochromatosis and needs early attention.

Steroid injection of the joint is not at all indicated at this stage.

Educational Objective:
New onset diabetes, arthropathy, and hepatomegaly should alert you about the possibility of hemochromatosis.
Mr. Divine, a 40-year-old male, comes to the office because of progressive knee and shoulder pain. The pain is 5/10 and started 6 months ago. He has no other medical problems, except newly diagnosed diabetes. He admits to "chain smoking" for "all his life" and drinks 1-2 beer a night. He is faithful to his wife. His mother died of “blood cancer” when he was 3 year old. His father also has diabetes mellitus. His vitals are: Temperature: 37C(98.6 F); PR: 86/min; BP: 134/86mm Hg: RR: 16/min. On exam, both knee joints are slightly swollen and tender. Mild hepatomegaly is present on abdominal examination. What one of the following is an appropriate next investigation in this patient?


A. HbA1C level
B. Serum iron studies
C. Liver biopsy
D. X-ray of the shoulder and knee
E. Steroid injection of the joint
The patient has newly diagnosed diabetes, hepatomegaly, and arthropathy. This makes him a likely candidate of the syndrome of hemochromatosis. It is an autosomal recessive disorder in which intestinal iron absorption is abnormally increased leading to iron deposition in the tissue. This damages the organs especially liver, pancreas, heart, and pituitary. Clinically it presents with hepatomegaly, hyperpigmentation, diabetes (bronze diabetes), arthropathy, restrictive heart failure, and hypogonadism. It may cause cirrhosis, which has an increased risk of hepatoma. The first logical step in the diagnosis of this disease is serum iron studies. Increased serum iron, ferritin, and transferrin saturation correlates with hemochromatosis. Liver biopsy confirms the diagnosis.

HbA1C level is done to detect long term glucose control in a diabetic patient taking various medications.

X-ray of shoulder and knee may be needed but symptomatology of the patient is highly suggestive of hemochromatosis and needs early attention.

Steroid injection of the joint is not at all indicated at this stage.

Educational Objective:
New onset diabetes, arthropathy, and hepatomegaly should alert you about the possibility of hemochromatosis.
Mr.Tyson, a 57-year-old man, comes with long-standing heartburn and chest pain. The pain is burning in nature and is unrelated to eating. The patient, for the past week or so, had decreased interest in things and has been more tired than usual. He thinks that he has lost almost 40 lb(18kg) in the last 6 months. He used to smoke 6-10 cigarettes/day but quitted 4 years ago. He drinks alcohol, once in a while. His father died at the age of 67 with lung cancer. He takes ranitidine for his heartburn. His vitals are stable. No other abnormality is noted. Portable chest x-ray is within normal limits. What is the next best step?


A. Bronchoscopy
B. CT scan chest with and without contrast
C. Barium swallow followed by endoscopy
D. Give omeprazole and return visit in 2 months
E. Test and eradicate helicobacter pylori infection
Patient’s symptom of heartburn, along with so much weight loss, has to be investigated thoroughly. He is probably having esophageal cancer. Esophageal cancer is relatively uncommon in the United States, and occurs most often in men over 50 years old. There are two main types of esophageal cancer, distinguished by the way they look under the microscope: squamous cell carcinoma and adenocarcinoma. Squamous cell cancer is associated with smoking and alcohol consumption. The incidence of this disease in the United States has remained relatively constant, while the incidence of adenocarcinoma of the esophagus has risen dramatically. Barrett's esophagus, a complication of gastroesophageal reflux disease (GERD), is a risk factor for the development of adenocarcinoma of the esophagus. Symptoms included are difficulty swallowing, regurgitation of food, heartburn, weight loss, vomiting blood and chest pain unrelated to eating.

The investigations include Barium swallow, EGD (esophagogastroduodenoscopy) and biopsy, and PET scan (sometimes useful to determine the stage of disease and whether surgery is possible). When esophageal cancer is localized to the esophagus and has not spread elsewhere (metastasized), surgery is the treatment of choice. The goal of surgery, in most cases, is to cure the patient. In some circumstances chemotherapy, radiation, or a combination of the two will be used to make surgery easier to perform. In patients who cannot tolerate surgery, or in situations where the cancer has spread to other organs (metastatic disease), chemotherapy or radiation may be used to help alleviate symptoms (palliative therapy). In such circumstances, however, the disease is usually not curable.

CT chest is considered if the upper GI endoscopy is negative. Bronchoscopy is indicted if there is anything on CT or the patient have high index of suspicion (hemoptysis) for endobronchial lesion. CT is not the first step in a patient with normal CXR, and other system symptoms.

H. Pylori eradication should be considered if this patient is proven to have mucosa associated lymphoid tissue lymphoma on endoscopy and biopsy.

Even though switching to omeprazole is an option, to give him some symptomatic benefit, his weight loss should be addressed seriously and return visit in 2 months, is not an acceptable choice here.

Educational Objective:
Barium swallow followed by endoscopy is usually done when a patient is suspected of having an esophageal cancer.
Ms. Nancy is very worried about her 8-month-old child. She says that she has come to you because she's not satisfied with her doctor. She is worried because her child does not have left testicle. Her doctor is always telling her to wait, but she wants something done now because she has read on internet that boys with "undescended testes always get cancer". The child is healthy and has attained all his milestones. The examination is unremarkable except for absent left testicle. Palpation of the left inguinal canal does not reveal a mass. What is the next best step?


A. Tell mother that her doctor is waiting unnecessarily and perform orchiopexy
B. Tell mother that her doctor is right in observing for one year and the child has no increased risk of cancer later in life.
C. Tell mother that her doctor is right in observing for one year and the child has increased risk of cancer later in life.
D. Refer the mother to a psychiatrist suspecting Munchausen by proxy.
E. Refer him to urologist for orchiectomy, as the patient will definitely develop testicular cancer in his teens.
The baby has Cryptorchidism (crypto=hidden, orchid= testicle) where the testis fails to descend into its normal position in the scrotum, by age 12 months. If the baby was premature, you wait until they would have been 12 month old, e.g., if 3 month premature, wait until they are 15 month before labeling the testicle as really undescended. Treatment is delayed until 12 month of age to give the testicle a chance to come down on its own. The procedure is called orchiopexy, and frees up the bands of tissue that kept the testicle from coming down. It is 99% effective and is done on an outpatient basis under general anesthesia. Boys with cryptorchidism have a lifetime risk of testicular cancer that is nearly four times higher than the risk in the general population. So, the best advice to mother is to wait until 1 year and also that her child has increased risk of testicular carcinoma even if it is fixed by surgery.

Educational Objective:
Treatment of undescended testes is delayed until 12 month of age to give the testicle a chance to come down on its own. These patients are at high risk for the development of testicular malignancy later in life.
Ms. Pascal, a 49-year-old woman, comes with a “strange itchy” lesion on her left areola, for the last 1 month. She has tried various lotions and creams but, it hasn’t gone away. She has no other skin conditions though she had poison-ivy, 2 months ago. She does not take any medications and feels “goofy” when she takes penicillin. On examination there is an eczemoid lesion on the left nipple. No abnormality was visible on mammogram. Biospy was done of the lesion and it reported large cells, surrounded by halo-like area, invading the epidermis. What is the diagnosis?


A. Adenoma of nipple.
B. Paget’s disease
C. Scirrhous carcinoma
D. Phyllodes tumor
E. Lymphocytic carcinoma of breast
Paget’s disease presents as a persistent dermatitis of the nipple. There is a red, oozing, crusted lesion, which is often unresponsive to topical steroid and antibiotics. The alert physician may biopsy the nipple, revealing the characteristic changes. Under the microscope, there is a proliferation of malignant epithelial cells scattered throughout the epidermis. The cells have abundant pale staining cytoplasm surrounding a hyper-chromatic nucleus, with prominent nucleoli. This disorder is a breast cancer and in almost every case, there is an underlying breast carcinoma present. Most commonly, this is an infiltrating ductal carcinoma but occasionally a ductal carcinoma in situ (DCIS) may be present. Overall, this cancer is rare, comprising 1-2% of all patients with breast carcinoma.

Educational Objective:
Paget’s disease presents as a persistent dermatitis of the nipple. There is a red, oozing, crusted lesion, which is often unresponsive to topical steroid and antibiotics. Biopsy shows malignant cells that invade the epidermis and have abundant pale staining cytoplasm surrounding a hyper-chromatic nucleus, with prominent nucleoli.
Ms. Stern is a 57-year-old female diagnosed with stage II Hodgkin’s lymphoma 4 weeks ago. She is put on combination chemotherapy of ABVD regimen (Doxorubicin, Bleomycin, Prednisone, and Procarbazine). She complains of severe nausea and vomiting. She does not want to continue with her medicine, as they are making her life even more miserable. She is also having some general malaise, and then had a bit of a sore throat. She has lost 30 lb(12.6kg) in a month. Her vitals are, Temperature: 38.5C(101.3F); PR: 88/min; RR: 18/min; BP: 109/68mm of Hg. The patient is on 92% pulse oximetry on room air. On examination the mouth is dry and she looks significantly cachectic with enlarged cervical lymph nodes and hepatomegaly. How can her vomiting be best managed?


A. Stop the chemotherapy drugs
B. Ondansetron
C. Metoclopramide
D. Tell patient that her nausea will get better with time
E. Prochlorperazine
Nausea and vomiting is one of the major undesirable effects of chemotherapy. It is the major reason for non-compliance and failure of chemotherapy regimes. Patients should be thoroughly educated about nausea and vomiting before the chemotherapy begins. As far as pharmacological treatment is concerned, a 5-HT3 receptor antagonist Ondansetron is very promising in reducing chemotherapy-induced emesis and is available in a tablet form also. The other steps that should be taken in such patients include small frequent meals, avoiding bland foods, appropriate food presentation, and giving patient what he wants to eat.

Nausea and vomiting is a major hindrance to patient's compliance to chemotherapy so telling him to bear it is not the best option.

Metoclopramide and prochlorperazine are the classical agents that can be give to such patients either alone or in combination with ondansetron though they are not the best drugs for chemotherapy-induced emesis.

Expected side effects like nausea and vomiting are not the indications to stop chemotherapy.

Educational Objective:
Ondansetron is the drug of choice for chemotherapy-induced emesis.
You have been asked to collect epidemiological data on cancers in North America. After going through volumes of patient data, you discover that only one cancer has significantly declined in incidence universally without any improvements in treatment. The cancer is:


A. Lung cancer
B. Stomach cancer
C. Melanoma
D. Colon cancer
E. Breast cancer
Stomach cancer is usually an adenocarcinoma arising in the stomach and includes tumors arising at or below the esophagogastric junction. The incidence of stomach cancer has dramatically declined universally, the reasons for which are still not known. This is the only cancer, which has universally declined in incidence, without any significant advances in treatment.

Choice A: Lung cancer is still increasing because of the increasing number of smokers. Once this cancer was seen in males only, but with more women smoking, the prevelance of this cancer in women is rising fast.

Choice C: Melanoma is still on the rise in the southern hemisphere. The cancer is being increasingly diagnosed in the North American population.

Choice D: Colon cancer has not declined despite numerous attempts at screening and preventive measures. It is still a major cause of death in the North American population.

Choice E: Breast cancer still is the most common cancer in females. The cause remains unknown and despite screening, no impact has been made on its prognosis.

Educational objective:
The only malignancy to have decreased universally over last several decades is stomach cancer. The reason for decrease in stomach cancer is not known and is not related to advances in medicine.
You receive a page from a Hospice nurse that wants you to see a patient. You find an 80-year-old woman with stage IV breast cancer. It has metastasized to liver and she has refused chemotherapy because of the side effects. Her abdomen is distended and on examination you find the fluid thrill positive with some irregular masses in liver. The woman does not want any invasive procedure done on her but is willing to take pills to "have her belly settle down." What is the drug of first choice in this patient?


A. Dexamethasone
B. Acetazolamide
C. Furosemide
D. Hydrochlorothiazide
E. Spironolactone
The patient has malignant ascites and the method of choice for acute relief is paracentesis, which the lady has refused. The drug of first choice is the aldosterone antagonist spironolactone. Spironolactone is a potassium-sparing diuretic also used for ascites associated with liver cirrhosis. Malignant ascites is seen most commonly in patients with ovarian, endometrial, breast, colon, gastric, and pancreatic cancer. The management of malignant ascites may include systemic chemotherapy, instillation of radioisotopes or chemotherapy drugs into the peritoneal fluid, and peritoneal-venous shunting procedures.

Other diuretics can be added to increase the effect of spironolactone (Choices B, C, D).

Dexamethasone has no role in the management of malignant ascites. (Choice A)

Educational Objective:
Spironolactone is the diuretic of choice for managements of ascites.
A 25-year-old, G3P0 married, white female comes to the antenatal clinic. She has a history of two second-trimester abortions, and she is 8 week pregnant now. She is taking folic acid and iron supplementation. She does not smoke but drinks alcohol only on social occasions. She is in monogamous relationship with her husband and has never been diagnosed or treated for any STDs. Her VDRL is positive with negative FTA-ABS test. CBC shows hematocrit of 30%, WBC count of 7,000/mico-L and platelet count of 70,000/micro-L. Her PT is 10 sec (normal value is 8.2-10.3seconds) and APTT is 37 sec (normal value is 21-32seconds). What will be the most appropriate next step in the management of this patient?


A. Reassurance
B. Repeat FTA-ABS
C. Repeat VDRL
D. Single dose of benzathine penicillin
E. Three doses of benzathine penicillin
F. Give heparin and aspirin.
Antiphospholipid antibody syndrome is characterized by recurrent arterial or venous thrombosis or recurrent fetal losses in the presence of antiphospholipid antibodies. Antiphospholipid antibodies are of three types. The first is responsible for false-positive syphilis serology. The second is lupus anticoagulant, which falsely elevates APTT. The third is anticardiolipin antibody. Antiphospholipid antibody syndrome is either primary or may be associated with other autoimmune disorders like SLE.

If a person is having arterial or venous thrombosis or fetal losses due to Antiphospholipid antibody syndrome, he or she must be started on anticoagulant therapy. Acute thrombosis is treated with heparin and factor anti-X-a activity is measured, as APTT is not reliable in such cases. Subsequently such patient is anticoagulated with warfarin.

Women with fetal losses due to Antiphospholipid antibody syndrome are managed during pregnancy with heparin and aspirin. Warfarin is not used due to its teratogenicity.

Reassurance alone is not going to help this patient as antiphospholipid syndrome puts her at an increased risk of first or second trimester abortion and use of heparin and aspirin reduces this risk.

Positive non-treponemal tests like VDRL can occur in a variety of conditions other than syphilis, therefore it must be followed by more specific treponemal tests like FTA-ABS. When FTA-ABS is negative, syphilis is highly unlikely and there is no need to treat with benzathine-penicillin.

Repeating VDRL or FTA-ABS are not appropriate choices here.

Educational Objective:
Antiphospholipid syndrome puts a pregnant female at an increased risk of first or second trimester abortion and use of heparin and aspirin reduces this risk.
A 10-year-old white male comes with his foster parents for evaluation of severe pain in his knees for the last few days. He says that his left knee has been hurting so bad that he is not able to play with his peers. The pain is continuous, not moving anywhere, and is associated with swollen knee. There is no history of any trauma to the knee. His family history is not known. His vitals are stable and he is afebrile. On examination he appears pale, his left knee is tender and there is obvious erythema and swelling. Blood tests show

WBC 12,600mm3
Hemoglobin 9.3
Hematocrit 32%,
Platelets 201,000mm3
ESR 40

X-ray is ordered and you notice that lower end of femur a central lytic lesion, onion skinning and a moth eaten appearance with some extension into the soft tissue . What is the most probable diagnosis?


A. Osteomylitis
B. Ewings sarcoma
C. Chondrosarcoma
D. Glomus tumor
E. Fibrosarcoma
Ewing’s sarcoma is a highly malignant tumor that is found in the lower extremity more than the upper extremity. The most common sites are the metaphysis and diaphysis of the femur followed by the tibia and humerus. Ewing's sarcoma is seen in the first and second decade and is common in white males. The tumor is very aggressive and metastasizes early to lungs and lymph nodes. The clinical presentation of Ewing's sarcoma includes pain and swelling for weeks or month’s duration. Erythema and warmth of the local area are sometimes seen. Osteomyelitis is often the initial diagnosis based on intermittent fevers, leukocytosis, anemia, and an increased ESR. Radiologically, Ewing's sarcoma is associated with a lamellated or "onion skin" periosteal reaction. The lesion is usually lytic and central. Endosteal scalloping is often present. The "onion-skin" appearance is often followed with a "moth-eaten" or mottled appearance and extension into soft tissue. Treatment for Ewing's sarcoma includes surgery, radiation, and multi-drug chemotherapy. Radiation or chemotherapy is used preoperatively.

Osteomyelitis (Choice A) is usually mistaken in such a scenario as it also presents with fever, malaise, local pain in joint, and swelling. X-ray findings in chronic osteomyelitis show a central lytic bone defect with surrounding sclerosis termed as Brodie’s abscess.

Chondrosarcoma (Choice C) as a primary tumor is not very common and it occurs in fifth or sixth decade of life. People with disorders like multiple enchondromas and Hemangiomas are at a higher risk than the general population. On X-ray there is a fusiform defect with scalloping.

Glomus tumor (Choice D) is a common vascular benign tumor, the most common site is subungual in 70% of the cases, and other sites are the palm and wrist. These are usually found in the fourth decade and effects females more than the males. There is a triad of severe intermittent pain, tenderness, and sensitivity to touch.

Fibrosarcoma (Choice E) is a malignant spindle cell neoplasm found mostly in 30-60 years of age. Clinical presentation is usually a painful mass. Radiological picture is that of an osteolytic lesion whose margins can be well defined or ragged and moth eaten.

Educational Objective:
Ewing’s sarcoma is often confused with osteomyelitis; however characteristic X-ray finding points to the diagnosis.
A 10-year-old white male comes with his foster parents for evaluation of severe pain in his knees for the last few days. He says that his left knee has been hurting so bad that he is not able to play with his peers. The pain is continuous, not moving anywhere, and is associated with swollen knee. There is no history of any trauma to the knee. His family history is not known. His vitals are stable and he is afebrile. On examination he appears pale, his left knee is tender and there is obvious erythema and swelling. Blood tests show

WBC 12,600mm3
Hemoglobin 9.3
Hematocrit 32%,
Platelets 201,000mm3
ESR 40

X-ray is ordered and you notice that lower end of femur a central lytic lesion, onion skinning and a moth eaten appearance with some extension into the soft tissue . What is the most probable diagnosis?


A. Osteomylitis
B. Ewings sarcoma
C. Chondrosarcoma
D. Glomus tumor
E. Fibrosarcoma
Ewing’s sarcoma is a highly malignant tumor that is found in the lower extremity more than the upper extremity. The most common sites are the metaphysis and diaphysis of the femur followed by the tibia and humerus. Ewing's sarcoma is seen in the first and second decade and is common in white males. The tumor is very aggressive and metastasizes early to lungs and lymph nodes. The clinical presentation of Ewing's sarcoma includes pain and swelling for weeks or month’s duration. Erythema and warmth of the local area are sometimes seen. Osteomyelitis is often the initial diagnosis based on intermittent fevers, leukocytosis, anemia, and an increased ESR. Radiologically, Ewing's sarcoma is associated with a lamellated or "onion skin" periosteal reaction. The lesion is usually lytic and central. Endosteal scalloping is often present. The "onion-skin" appearance is often followed with a "moth-eaten" or mottled appearance and extension into soft tissue. Treatment for Ewing's sarcoma includes surgery, radiation, and multi-drug chemotherapy. Radiation or chemotherapy is used preoperatively.

Osteomyelitis (Choice A) is usually mistaken in such a scenario as it also presents with fever, malaise, local pain in joint, and swelling. X-ray findings in chronic osteomyelitis show a central lytic bone defect with surrounding sclerosis termed as Brodie’s abscess.

Chondrosarcoma (Choice C) as a primary tumor is not very common and it occurs in fifth or sixth decade of life. People with disorders like multiple enchondromas and Hemangiomas are at a higher risk than the general population. On X-ray there is a fusiform defect with scalloping.

Glomus tumor (Choice D) is a common vascular benign tumor, the most common site is subungual in 70% of the cases, and other sites are the palm and wrist. These are usually found in the fourth decade and effects females more than the males. There is a triad of severe intermittent pain, tenderness, and sensitivity to touch.

Fibrosarcoma (Choice E) is a malignant spindle cell neoplasm found mostly in 30-60 years of age. Clinical presentation is usually a painful mass. Radiological picture is that of an osteolytic lesion whose margins can be well defined or ragged and moth eaten.

Educational Objective:
Ewing’s sarcoma is often confused with osteomyelitis; however characteristic X-ray finding points to the diagnosis.
A 16-year-old boy comes to the clinic with worsening chronic left lower leg pain. The pain is dull, was initially mild and intermittent but now is increasing in intensity and persistence. He is taking aspirin for this pain and it makes pain much better. There is no history of trauma. On examination, you see there is also a 1.5" swelling at his elbow. The patient says that it is there for quite some time and is growing slowly, but not bothering him at all. It is soft, discrete, and freely movable round mass right under the skin. You order an x-ray of the leg and biopsy of the mass and you get the following reports:

Radiologist: “Left Tibia: Centrally located oval radio-lucent area, measuring less than 1cm in diameter, surrounded by a zone of uniform bone sclerosis.”

Biopsy: “ Mass in elbow: Circumscribed collection of mature adipose tissue”

What is you additional concern in this patient?


A. Colonic polyp
B. Testicular cancer
C. Child abuse
D. Medulloblastoma
E. Rickets
The patient has Gardner's syndrome. Gardner syndrome refers to an association between colonic polyps and the presence of prominent extra-intestinal lesions. These include desmoid tumors, sebaceous or epidermoid cysts, lipomas, osteomas (especially of the mandible), supernumerary teeth, gastric polyps, and juvenile nasopharyngeal angiofibromas. The patient has osteoma and lipoma so colonic polyps is an additional concern. So, he needs complete work up for this too in addition to the complaints he has.

There is no association of testicular cancers and rickets with the complaints patient has.

The scenario does not fit in with child abuse. Medulloblastoma is part of Turcot’s syndrome, not Gardner.

Educational Objective:
Gardner syndrome refers to an association between colonic polyps and the presence of prominent extra-intestinal lesions. These include desmoid tumors, sebaceous or epidermoid cysts, lipomas, osteomas (especially of the mandible), supernumerary teeth, gastric polyps, and juvenile nasopharyngeal angiofibromas.
A 16-year-old boy is brought by his basketball coach, due to his inability to workout with other boys. The boy is complaining of pain in his left knee for the past several weeks. The pain is dull and becomes worse when he is running or comes on the court. He is sexually active with many partners, and does not use condoms. He does not smoke or drink alcohol. He thinks he has lost some weight, also. His vitals are stable, with no fever. On examination, his left knee is swollen and tender. X-ray reveals a sclerotic lesion in distal femur with periosteum lifted and a "sun burst appearance". What is the most likely diagnosis?


A. Ewings sarcoma
B. Growing pain
C. Osteosarcoma
D. Septic arthritis
E. Giant cell tumor
The characteristic radiographic findings and the history are highly suggestive of osteogenic sarcoma. Osteosarcoma is the most common malignant bone tumor in youth. The average age at diagnosis is 15 years. Boys and girls have a similar incidence of this tumor, until late adolescence, at which time boys are more commonly affected. It is a metaphyseal tumor. Osteosarcoma tends to occur in the bones of the thigh (distal end), upper arm (proximal end), and shin (proximal end). This cancer occurs most commonly in larger bones and in the area of bone with the fastest growth rate. Osteosarcoma can occur in any bone, however. The most common symptom initially is pain. As the tumor grows, there may be visible swelling and limitation of motion. Tumors in the legs cause limping, while those in the arms cause pain on lifting. Swelling over the tumor may be warm and slightly reddened. Treatment is surgery. Limb-saving surgery is usually possible, but more radical surgery (such as amputation) may be necessary for a permanent cure. Codman’ s triangle and sunburst appearance are the 2 classic radiologic features.

Ewings sarcoma is a very malignant tumor but it has characteristic ‘onion skinning’ on the radiograph. It is usually a diaphyseal tumor.

Growing pains are very common in this age group but it is a diagnosis of exclusion. The diagnosis should not be made in the presence of characteristic x-ray changes.

The patient is not having safe sexual practices and is at increased risk of gonococcal arthritis but absence of fever and other such signs rules this out.

Giant cell tumor has characteristic "soap bubble appearance" on the radiograph.

Educational Objective:
Know how to diagnose osteosarcoma. Codman’ s triangle and sunburst appearance are the two classic radiologic features of this entity.
A 16-year-old boy is brought by his basketball coach, due to his inability to workout with other boys. The boy is complaining of pain in his left knee for the past several weeks. The pain is dull and becomes worse when he is running or comes on the court. He is sexually active with many partners, and does not use condoms. He does not smoke or drink alcohol. He thinks he has lost some weight, also. His vitals are stable, with no fever. On examination, his left knee is swollen and tender. X-ray reveals a sclerotic lesion in distal femur with periosteum lifted and a "sun burst appearance". What is the most likely diagnosis?


A. Ewings sarcoma
B. Growing pain
C. Osteosarcoma
D. Septic arthritis
E. Giant cell tumor
The characteristic radiographic findings and the history are highly suggestive of osteogenic sarcoma. Osteosarcoma is the most common malignant bone tumor in youth. The average age at diagnosis is 15 years. Boys and girls have a similar incidence of this tumor, until late adolescence, at which time boys are more commonly affected. It is a metaphyseal tumor. Osteosarcoma tends to occur in the bones of the thigh (distal end), upper arm (proximal end), and shin (proximal end). This cancer occurs most commonly in larger bones and in the area of bone with the fastest growth rate. Osteosarcoma can occur in any bone, however. The most common symptom initially is pain. As the tumor grows, there may be visible swelling and limitation of motion. Tumors in the legs cause limping, while those in the arms cause pain on lifting. Swelling over the tumor may be warm and slightly reddened. Treatment is surgery. Limb-saving surgery is usually possible, but more radical surgery (such as amputation) may be necessary for a permanent cure. Codman’ s triangle and sunburst appearance are the 2 classic radiologic features.

Ewings sarcoma is a very malignant tumor but it has characteristic ‘onion skinning’ on the radiograph. It is usually a diaphyseal tumor.

Growing pains are very common in this age group but it is a diagnosis of exclusion. The diagnosis should not be made in the presence of characteristic x-ray changes.

The patient is not having safe sexual practices and is at increased risk of gonococcal arthritis but absence of fever and other such signs rules this out.

Giant cell tumor has characteristic "soap bubble appearance" on the radiograph.

Educational Objective:
Know how to diagnose osteosarcoma. Codman’ s triangle and sunburst appearance are the two classic radiologic features of this entity.
A 22-year-old African American male suffering from sickle cell anemia presents in the emergency room with sustained painful erection of his penis for the last 4 hours. His vitals are, PR: 76/min; RR: 16/min; BP: 115/76mm of Hg; Temperature: 37C(98.6F). Previously, he had one episode of acute chest syndrome and it was treated with oxygenation, hydration and blood transfusion. Examination of all his systems is unrevealing. What will be the best next step in the management of this patient?


A. Observation
B. Oxygen therapy
C. Exchange transfusion
D. Surgical intervention
E. Hydroxyurea
Sickle cell anemia may cause acute painful episodes. Precipitating factors need to be found and treated to relieve acute painful episode. Adequate hydration and oxygen therapy are also important to terminate such episodes.

Acute vasoocclusive crisis is an important complication of sickle cell anemia and may cause stroke, priapism or intractable pain. Exchange transfusion is indicated whenever acute vasoocclusive crisis occurs.

Surgical intervention to treat priapism is not an appropriate choice here.

Hydroxyurea is a cytotoxic agent and is used in a patient of sickle cell anemia when his life is disrupted by frequent painful episodes. Hydroxyurea produces its beneficial effect by elevating HbF levels.

Educational Objective:
Acute vasoocclusive crisis is an important complication of sickle cell anemia and may cause stroke, priapism or intractable pain. Exchange transfusion is indicated whenever acute vasoocclusive crisis occurs.
A 22-year-old white male presented to the emergency room (ER) with the sudden onset of acute right upper quadrant pain. The ultrasound showed cholelithiasis. Initial evaluation revealed hemoglobin of 9 gm/dl with an MCV of 90 fl and a total reticulocyte count of 1000 cells per microliter. Peripheral smear revealed polychromatophilia and spherocytes. Liver function tests revealed an elevated indirect bilirubin and normal hepatic enzyme levels. Physical examination is consistent with pallor. The patient’s parents were killed in an accident when he was 8-year-old, and the patient does not know anything about the family history. Which of the following is the most correct statement about this patient’s condition?


A. His condition is classically transmitted as autosomal recessive disorder.
B. He will probably depend on transfusions.
C. This patient should be placed on folic acid supplementation.
D. Vaccination against parvovirus has shown to decrease morbidity and mortality.
E. This patient’s mean corpuscular hemoglobin concentration (MCHC) is likely to be very low.
This patient has classic hereditary spherocytosis, which is typically transmitted as an autosomal dominant disorder characterized by a defect in erythrocyte membrane usually spectrin. Patients usually presents with anemia, spherocytosis, splenomegaly, and an increased erythrocyte osmotic fragility. It is usually first suspected in the neonatal period because of hyperbilirubinemia, although this may be mild and may go unrecognized until adulthood. By the third decade of life, up to 40% of the patients may develop cholelithiasis and present with cholecystitis. Chronic leg ulcers may complicate this disease. These patients are often complicated by episodes of aplastic crisis in which usually erythropoiesis is suppressed and the hemolytic process continues. These episodes are life threatening and usually develop because parvovirus infection and are often associated leukopenia and thrombocytopenia. Sometimes severe anemia may also occur when the intake of folic acid is too low to meet the demands of increased erythropoiesis.Therefore, all patients should be prescribed folic acid supplementation.

Peripheral smears typically show spherocytosis and polychromatophilia. Usually the MCV is normal but the mean corpuscular hemoglobin concentration is elevated.

The osmotic fragility is usually the diagnostic test even though it may also be increased in any condition in which spherocytes are present like warm autoimmune hemolytic anemia or ABO incompatibility in the newborn.

These patients rarely require transfusions unless they have episodes of aplastic crisis. Splenectomy is usually curative.

Splenectomy is usually indicated for patients who have moderate to severe disease in order to prevent the complications of the disorder such as aplastic crisis, cholelithiasis and chronic leg ulcers. Usually splenectomy should be delayed four to five years to prevent post splenectomy sepsis. All patients prior to splenectomy should receive vaccinations against encapsulated organisms such as Haemophilus, Pneumococci, and meningococci.

There is no evidence that the use of vaccination against parvovirus will decrease the morbidity or mortality.

Educational Objective:
Folic acid supplementation is encouraged in patients with high risk for developing aplastic crisis.
A 24-month-old African-American child, who lives in poor neighborhood, is brought to the office by his mother. The mother has noted that he has been acting bizarre and lethargic. She thinks that this is due to “some spell”. He has a habit of eating wooden, painted toys. On exam he has marked pallor. CBC is ordered which reveals

WBC 8,600mm3
Hemoglobin 7.1
Hematocrit 25%,
Platelets 146,000mm3

His blood lead levels are elevated. Which of the following is the most likely picture in peripheral blood smear?


A. Megaloblastic anemia, and basophilic stippling
B. Tear drop RBC, and hypochromic microcytic anemia
C. Loss of concavity of the RBC, and basophilic stippling
D. Basophilic stippling, microcytic hypochromic anemia
E. Normochromic normocytic anemia, and basophilic stippling
F. Dimorphic anemia, and basophilic stippling
Lead Poisoning is seen in children who are exposed to old houses and can ingest paint from them. Elevated blood lead levels can interfere with hemoglobin synthesis resulting in microcytic hypochromic anemia. Signs and symptoms depend on the blood lead levels. They range from being asymptomatic to anorexia, behavior changes, clumsiness, nausea, and vomiting. Screening is done by finger stick blood lead test in high-risk children; a positive test is followed by venous blood lead levels. Any finding that is greater than 10µg/dL is abnormal. X-rays may show lead lines at the metaphysis of the long bones. The peripheral blood smear picture is significant and shows anisocytosis, microcytosis, hypochromia and basophilic stippling. Microcytic anemia and basophilic stippling are 2 important clues to make the physician should think about lead poisoning in this kind of patient population.

Megaloblastic anemia and basophilic stippling are seen in either Vitamin B12 or Folic acid deficiency.

Tear drop RBCs are seen in all diseases that infiltrate the bone marrow causing myelofibrosis.

Loss of concavity of the RBCs is seen in diseases like hereditary spherocytosis (HS).

Basophilic stippling can also be seen in dimorphic anemias.

Educational Objective:
Microcytic anemia and basophilic stippling in a child should make you think about lead poisoning.
A 25-year-old African American male comes to your office with sudden onset of back pain, dark urine and fatigue since one day. His past medical history is unremarkable except that two days back, he was started on trimethoprim-sulphamethoxazole for sinusitis. He has significant pallor on exam. His laboratory report shows Hb: 7.5 g/dL, Total bilirubin: 3.5 mg/dL, Direct bilirubin: 0.8 mg/dL, decreased haptoglobin levels and increased LDH levels. His peripheral smear shows ‘bite’ cells. You order his G-6-P-D levels, which comes as normal. Which of the following is the most likely cause of his hemolysis?


A. G-6-P-D deficiency.
B. Pyruvate kinase deficiency.
C. Galactokinase deficiency.
D. Sickle cell disease.
This patient is having acute hemolysis as suggested by low hemoglobin, increased indirect bilirubin, increased LDH levels and decreased haptoglobin levels.

He is an African American and his symptoms were precipitated after ingestion of trimethoprim-sulfamethoxazole, all of which suggest G-6-P-D deficiency as the cause of hemolysis in this patient. G-6-P-D levels are often normal during the attack of hemolysis as in this patient (Choice A). Bite cells on peripheral smear is also characteristic of G-6-P-D deficiency.

Glucose 6-phosphate dehydrogenase (G6PD) deficiency is an X-linked disorder and is the most common enzymatic disorder of red blood cells in humans; it is most commonly seen in African American, Asian and people from Mediterranean origin.

G6PD A- (moderate enzyme deficiency) and G6PD Mediterranean (severe enzyme deficiency) are the variants of G6PD deficiency, where hemolysis is precipitated by infection, exposure to drugs having a high redox potential (primaquine and sulfa drugs like sulphamethoxazole as in this case) or to fava beans, or metabolic abnormalities.

Choice B: Pyruvate kinase deficiency can also lead to similar clinical picture of hemolytic anemia, however in such cases hemolysis is not precipitated by sulfa drugs and also it would not explain the presence of bite cells in peripheral smear.

Choice C: Galactokinase deficiency would not cause hemolytic anemia; rather the only manifestation would be development of cataract.

Choice D: Sickle cells crises are generally painful crisis and would be characterized by presence of sickle cells or Howell Jolly bodies on peripheral smear.

Educational Objective:
Always consider G6PD deficiency in patients who develop acute hemolysis following ingestion of primaquine or sulfa drugs. Also, G6PD levels are often normal during the hemolytic episode.
A 25-year-old female, with previous history of easy bruising, presents with menorrhagia. Previously, she was investigated for endocrine disorders but the only abnormality found was low platelet count. A diagnosis of idiopathic thrombocytopenia was made. She has not been taking any medications. She smokes 1pack/day cigarettes for the last 5 years and drinks alcohol, only on social occasions. She is in a monogamous relation with her husband. She does not have a family history of any bleeding (hematological) disorder. Her vitals are, PR: 76/min; RR: 16/min; BP: 115/76 mm of Hg; Temperature: 37C(98.6F). Petechiae are the only abnormality found on her examination. There is no lymphadenopathy or hepatosplenomegaly.
Lab tests show:

Hematocrit 40%
WBC count 11,000/micro-L
Platelet count 19,000/miro-L.

Peripheral blood smear shows large platelets. Prothrombin time and partial thromboplastin times are normal while bleeding time is increased. Bone marrow biopsy is performed which shows increased number of megakaryocytes. Which of the following additional testing should be considered as part of work-up in this patient?


A. Antinuclear antibody testing
B. Liver function tests
C. Chest X-ray
D. ELISA testing for HIV
E. Factor VIII assay
F. Monospot test
Idiopathic thrombocytopenic purpura (ITP) is an autoimmune disorder characterized by: isolated thrombocytopenia, absence of splenomegaly, absence of fever or other systemic features, normal bone marrow with normal or increased number of megakaryocytes. ITP is acute and self-limiting in children but it usually becomes a chronic disorder in adults. It presents with skin or mucosal bleeding. Coagulation studies are normal. Autoimmune destruction of platelets may be primary or it may be secondary to SLE or infection with agents like CMV, toxoplasma or HIV.

Whenever a person is having chronic ITP, the bone marrow examination must be performed to rule out primary hematological disorder. Patient must also be screened for SLE by antinuclear anti-body testing, as isolated thrombocytopenia may be a presenting feature of SLE especially if the patient is a young female.

If a patient, with diagnosis of chronic ITP, shows abnormal findings like hepatosplenomegaly, lymphadenopathy or atypical lymphocytosis, secondary causes of autoimmune thrombocytopenia like infection with CMV, HIV, hepatitis viruses and toxoplasma should be considered. The patient presented in this vignette does not have any such abnormality; therefore, she does not need to be screened for HIV by ELISA or EBV by Monospot test.

This patient is highly unlikely to be suffering from Hemophilia, as her coagulation studies are normal. There is no family history and bleeding pattern of skin and mucosal involvement indicate platelet or vessel wall abnormality. Therefore, factor VIII assay is highly inappropriate.

Coagulopathy of liver disease is unlikely with normal coagulation tests, and with absence of other stigmata of liver disease. Therefore, LFTs are not appropriate choice here.

Cigarette smoking is a risk factor for lung cancer but chest x-ray is not used for its screening.

Educational Objective:
Patient with chronic idiopathic thrombocytopenia must be screened for systemic lupus erythematosus by antinuclear anti-body testing.
A 25-year-old male presents to his physician with the complaint of petechiae on his extremities. He also sates that he has had several episodes of nasal bleed in the last 6 months, each of which was self-limiting. He has no other complaints. He does not take any medication and is a non-smoker and non-alcoholic. His family history is not significant for any similar disease. His vitals are, PR: 76/min; RR: 16/min; BP: 115/76mm of Hg; Temperature: 37C(98.6F). Systemic examination shows petechiae as the only abnormal finding.
Lab studies show:

Hematocrit 35%
WBC count 9,000/micro-L
Platelet count 16,000/miro-L

Coagulation studies and bone marrow examination are normal. Serological testing shows negative antinuclear antibody testing. A diagnosis of chronic idiopathic thrombocytopenic purpura is made and he is started on high-dose prednisolone therapy. He is still having petechiae and his platelet count is 17,000 after 3 weeks of treatment with high-dose prednisolone. What is the most appropriate next step in the management of this patient?


A. Continue high-dose prednisolone therapy
B. Add cyclophosphamide
C. Start treatment with danazol
D. Perform splenectomy
E. Start IV immunoglobulins
F. Give platelet transfusion
Idiopathic thrombocytopenic purpura (ITP) is an autoimmune disorder characterized by isolated thrombocytopenia, absence of splenomegaly, absence of fever or other systemic features, normal bone marrow with normal or increased number of megakaryocytes. ITP is acute and self-limiting in children but it usually becomes a chronic disorder in adults. It presents with skin or mucosal bleeding. Coagulation studies are normal.

Most adults respond to steroids within 2 weeks, with most responding within the first week. When a patient of ITP fails to respond to high-dose prednisolone therapy (2 weeks after diagnosis), splenectomy is indicated. About 80 % of patients improve after removal of spleen.

When a patient of ITP fails to respond to glucocorticoids and splenectomy, he is then treated with danazol or cytotoxic agents like cyclophosphamide.

IVIG is employed in those cases of ITP when there is a bleeding emergency or when platelet counts are to be raised in a severely thrombocytopenic patient before surgery.

Platelet transfusions are needed only in cases of life-threatening bleed.

Educational Objective:
When a patient of chronic idiopathic thrombocytopenic purpura fails to respond to high-dose prednisolone therapy, splenectomy is indicated.
A 25-year-old white female presents with 5-day history of sore throat, extreme fatigue, and headaches. She smokes 2-3 cigarettes per day and occasionally drinks alcohol. She has returned from a spring break in Jamaica where she had “time of her life”. Her vitals are stable and she is afebrile. On examination, she has cervical lymphadenopathy, mild splenomegaly, and exudative pharyngitis. CBC showed:

WBC 16,000cmm with 55% lymphocytes
Hemoglobin 13.5gm/dL
Hematocrit 47%,
Platelets 216,000cmm

Many variant forms of lymphocytes were seen including cells with convoluted nuclei and highly vacuolated cytoplasm. Rapid streptococcal throat test, urinalysis, and heterophilic antibody test are all negative. What is the most likely diagnosis?


A. Acute myeloid leukemia
B. Chronic myeloid leukemia
C. Acute lymphoblastic leukemia
D. Chronic lymphocytic leukemia
E. Infectious mononucleosis
F. Hodgkin’s lymphoma
This is a classic presentation of Infectious Mononucleosis (IM). It is an acute, benign and self-limiting lymphoproliferative condition caused by Epstein-Bar Virus (EBV). EBV is transmitted primarily by close contact with infectious oro-pharyngeal secretions. Clinical manifestations include extreme fatigue, malaise, close throat, fever, and lymphadenopathy. Splenomegaly is also common and contact sports should be avoided to prevent the chances of splenic rupture. When rupture occurs the mortality is significant. Hematological studies reveal leukocytosis with variant lymphocytes. Heterophile antibodies are very specific and may be demonstrable at the onset of illness or may appear later in the course of the illness. So, a negative antibody test doesn’t exclude the diagnosis of IM.

AML and ALL are commonly seen in younger patients and are characterized by increased blasts in the peripheral smear.

CLL is the leukemia of old age and mature small lymphocytes are the predominant cells in this case.

CML is one of the myeloproliferative syndromes seen normally in adults and consists of leukocytosis with basophilia and marked splenomegaly. Lymphadenopathy is not a feature of CML.

Hodgkin’s lymphoma usually have normal blood smear, and they have Reid Sternberg cells in the biopsy of the lymph nodes.

Educational Objective:
Know about the typical presentation of infectios mononucleosis; heterophile antibodies are very specific and may be demonstrable at the onset of illness or may appear later in the course of the illness. So, a negative antibody test doesn’t exclude the diagnosis of IM.
A 25-year-old white female presents with 5-day history of sore throat, extreme fatigue, and headaches. She smokes 2-3 cigarettes per day and occasionally drinks alcohol. She has returned from a spring break in Jamaica where she had “time of her life”. Her vitals are stable and she is afebrile. On examination, she has cervical lymphadenopathy, mild splenomegaly, and exudative pharyngitis. CBC showed:

WBC 16,000cmm with 55% lymphocytes
Hemoglobin 13.5gm/dL
Hematocrit 47%,
Platelets 216,000cmm

Many variant forms of lymphocytes were seen including cells with convoluted nuclei and highly vacuolated cytoplasm. Rapid streptococcal throat test, urinalysis, and heterophilic antibody test are all negative. What is the most likely diagnosis?


A. Acute myeloid leukemia
B. Chronic myeloid leukemia
C. Acute lymphoblastic leukemia
D. Chronic lymphocytic leukemia
E. Infectious mononucleosis
F. Hodgkin’s lymphoma
This is a classic presentation of Infectious Mononucleosis (IM). It is an acute, benign and self-limiting lymphoproliferative condition caused by Epstein-Bar Virus (EBV). EBV is transmitted primarily by close contact with infectious oro-pharyngeal secretions. Clinical manifestations include extreme fatigue, malaise, close throat, fever, and lymphadenopathy. Splenomegaly is also common and contact sports should be avoided to prevent the chances of splenic rupture. When rupture occurs the mortality is significant. Hematological studies reveal leukocytosis with variant lymphocytes. Heterophile antibodies are very specific and may be demonstrable at the onset of illness or may appear later in the course of the illness. So, a negative antibody test doesn’t exclude the diagnosis of IM.

AML and ALL are commonly seen in younger patients and are characterized by increased blasts in the peripheral smear.

CLL is the leukemia of old age and mature small lymphocytes are the predominant cells in this case.

CML is one of the myeloproliferative syndromes seen normally in adults and consists of leukocytosis with basophilia and marked splenomegaly. Lymphadenopathy is not a feature of CML.

Hodgkin’s lymphoma usually have normal blood smear, and they have Reid Sternberg cells in the biopsy of the lymph nodes.

Educational Objective:
Know about the typical presentation of infectios mononucleosis; heterophile antibodies are very specific and may be demonstrable at the onset of illness or may appear later in the course of the illness. So, a negative antibody test doesn’t exclude the diagnosis of IM.
A 27-year-old, African American, male presents with dyspnea and fatigue. He does not have family history of asthma, heart disease or any blood disorder. His vitals are stable and afebrile. The only significant findings on his examination are pallor and splenomegaly.
Lab studies show

Hematocrit 20%
WBC count 4,000/micro-L
Platelet count 25,000/miro-L
Bilirubin 7 mg/dL
Direct bilirubin 0.6 mg/dL.
Serum LDH 500 U/L (normal value is 80-280 U/L)
Serum haptoglobin 20mg/dL (normal value is 30-220 mg/dL).

Peripheral blood smear shows microcytic hypochromic cells. Serum ferritin is low while total iron binding capacity (TIBC) is elevated. Reticulocyte count is 4 %. Urine dipstick testing is positive for hematuria and microscopy of urine shows 1 RBC/HPF. Repeated G6PD assays are normal; Coomb’s and micro-Coomb’s tests are all negative. Bone marrow examination shows hypocellular marrow. Based on these findings, what is the most likely diagnosis?


A. Aplastic anemia
B. G6PD deficiency
C. Hereditary spherocytosis
D. Autoimmune hemolytic anemia
E. Paroxysmal nocturnal hemoglobinuria
F. Anemia of chronic disease
Bone marrow in a patient of aplastic anemia is hypocellular and peripheral blood smear does not show any abnormal morphology of cells. RBCs are normocytic or macrocytic. There is neither hemolysis of RBCs nor splenomegaly.

G6PD deficiency is an X-linked condition, so there will be a positive family history. There is no chronic hemolytic anemia and episodes of hemolysis occur only due to oxidative stress from infection or drugs. Peripheral blood smear is normal, there is no splenomegaly and platelet counts and WBC counts are normal. G6PD levels are low between hemolytic episodes.

Hereditary spherocytosis is an autosomal dominant condition, thus there will be a positive family history. Peripheral smear shows spherocytes with loss of central pallor and hemolysis patterns is of extra-vascular type with normal serum LDH and serum haptoglobin levels.

In autoimmune hemolytic anemia, hemolysis is of extra-vascular type and peripheral smear may show spherocytes. Coomb’s or micro-Coomb’s test is positive in such cases. Platelet and WBC counts are normal.
In anemia of chronic disease, RBCs may be microcytic but serum ferritin levels are high with low TIBC. Platelet and WBC counts are normal.

Paroxysmal nocturnal hemoglobinuria (PNH) is an acquired disorder of the hematopoietic cells and may cause pancytopenia. PNH should be considered in all confusing cases of hemolytic anemias or pancytopenias. Hemolysis is intra-vascular with low serum haptoglobin and elevated serum LDH. Loss of iron in the urine may result in iron deficiency anemia. Congestive splenomegaly is one of the complications of this disorder. Bone marrow may be hypocellular. Ham’s test is not a reliable test for PNH. Flow cytometry is a better choice and confirms the diagnosis by showing absence of CD59.

Educational Objective:
Paroxysmal nocturnal hemoglobinuria (PNH) should be considered in all confusing cases of hemolytic anemia or pancytopenia. Flow cytometry confirms the diagnosis of PNH by showing absence of CD59.
A 29-year-old African-American woman comes to the office of a primary care physician complaining of pain in her right knee for the last 3 months. The pain is getting progressively worse and is not moving anywhere. She has tried ibuprofen but found little comfort. She is a housewife and unable to do household work because of this pain. She does not smoke nor drink and has hypertension well controlled by hydrocholorothiazide. Her vitals are stable and she is afebrile. On examination her right knee is a little swollen and tender with reduced range of motion. On chest x-ray, there is soap-bubble appearance in the epiphysis of distal femur. What is the next best step in the management of this patient?


A. Refer to an orthopedic surgeon
B. Perform biopsy in the office
C. Order bone scan
D. Start antibiotics
E. Do rheumatoid factor and anti-nuclear antibody
F. Give her painkiller and reassure her
Key here is “soap bubble appearance” in the epiphyseal end of long bone. This is giant cell tumor of bone. The classical patient is 20-40yr old female with knee pain and some mass. The tumor cells are oval or spindle shaped intermingled with numerous multinuclear giant cells in fibrous stroma. It is benign but locally aggressive. It frequently recurs even after local curettage. The best way to handle this tumor is to recognize it and leave it for experts.

Biopsy of a bone tumor is a highly specialized field and should not be tried in office.

Her radiographic findings are not indicative of Rheumatoid arthritis so RF + ANA is not indicated.

Giving her painkillers or reassurance should not be done in case of Giant cell tumors.

She has no signs of infection; she is afebrile and there is no redness or warmth and her X-ray has classic appearance for tumor.

Bone scan is not useful in these patients. It is useful for detecting solid tumor metastasis.

Educational Objective:
Know how to recognize bone tumors and refer them to appropriate experts.
A 32-year-old Caucasian female presents to your office for a routine check-up. Her past medical history is significant for generalized seizures controlled with chronic phenytoin therapy. The last seizure was six months ago. She does not smoke or consume alcohol. Physical examination is insignificant, except mild pallor. Laboratory values are:

Hb 10.8 g/dL
MCV 105 fl
Platelet count 180,000/cmm
Leukocyte count 7,500/cmm
Segmented neutrophils 68%
Bands 1%
Eosinophils 1%
Lymphocytes 24%
Monocytes 6%

Which of the following supplementations could have prevented this patient’s anemia?


A. Folic acid
B. Vitamin B12
C. Iron
D. Vitamin B6
E. Vitamin B1
This patient presents with mild megaloblastic anemia, which is most probably caused by chronic phenytoin therapy. Some anti-epileptic drugs including phenytoin, primidone and phenobarbital can cause megaloblastic anemia that is usually mild. The pathophysiology of this condition involves impaired absorption of folic acid in the small intestine. Folic acid supplementation can effectively prevent this condition.

Several other drugs can also cause folic acid deficiency:
1. Trimethoprim: It inhibits dihydrofolate reductase and in high doses can cause megaloblastic pancytopenia.
2. Methotrexate: Also inhibits dihydrofolate reductase. Folinic acid (leucovorin) is indicated to reverse the chemotherapeutic anti-folate effect of methotrexate.

Although vitamin B12 deficiency (choice B) also causes megaloblastic anemia the metabolism of this substance is not impaired by phenytoin.

Vitamin B6 (choice D) is antagonized by some drugs used to treat tuberculosis, especially isoniazid.

Iron deficiency anemia (choice C) manifests as microcytic, not megaloblastic anemia.

Vitamin B1 deficiency (choice E) does not cause anemia.

Educational Objective:
The most common cause of folic acid deficiency is nutritional due to poor diet and/or alcoholism. Folic acid deficiency can be caused by some drugs; these drugs can impair the absorption of folic acid (e.g., phenytoin) or antagonize its physiologic effects (e.g., methotrexate, trimethoprim).

*Extremely high yield question for USMLE!!!
A 35-year-old white female slipped and fell on her side 2 days ago while she was going down the stairs from her house. Since that time, she has been having pain in her right shoulder. She describes this as an ache, which has been about the same over the last couple of days. She has tried ibuprofen, with only slight relief. She denies smoking and alcohol use. Examination suggests a shoulder sprain. You order an x-ray and see a normal shoulder but incidentally a 1.5cm coin-shaped lesion, in the outer side of right lung with well-aerated surrounding lung. She denies any respiratory complaints. What is the next best step regarding her lung lesion?


A. Ask for an old X-ray
B. CT scan chest
C. Fine needle aspiration
D. Bronchoscopy
E. Open lung biopsy
F. PET Scan
This patient has a "solitary pulmonary nodule" defined as a 2cm or less coin-shaped lesion in the middle to lateral one third of the lung surrounded by normal parenchyma. Most of them are benign. Calcification of the nodules favors a benign lesion. "Popcorn" calcification is a hamartoma while "bulls eye" is a granuloma.

In low risk patients (i.e. < 40 years and non smokers) a solitary pulmonary nodule is not a sign of immediate alarm. The best approach is to ask for an old x-ray. If there is no change in it for the last 12 months it is considered benign. It is followed by a CXR every 3 months, for the next 12 months, and if there is no growth or no symptoms it is left as such.

High-risk patients i.e. smokers, require a full investigation work up. It starts with a CT scan chest just to get better picture of the lesion and its extent, followed by fine needle aspiration. If the lesion is missed, an open-lung biopsy is an option.

Bronchoscopy is not good at reaching peripheral lesions.

PET scan may also be a tool in the hands of the pulmonologist, to sort out benign from malignant lesions in a high-risk patient. PET scans are expensive and considered only in certain patients especially for metastatic workup.

Educational Objective:
In low risk patients (i.e. < 40 years and non smokers) a solitary pulmonary nodule is not a sign of immediate alarm. The best approach is to ask for an old x-ray.
A 35-year-old white female slipped and fell on her side 2 days ago while she was going down the stairs from her house. Since that time, she has been having pain in her right shoulder. She describes this as an ache, which has been about the same over the last couple of days. She has tried ibuprofen, with only slight relief. She denies smoking and alcohol use. Examination suggests a shoulder sprain. You order an x-ray and see a normal shoulder but incidentally a 1.5cm coin-shaped lesion, in the outer side of right lung with well-aerated surrounding lung. She denies any respiratory complaints. What is the next best step regarding her lung lesion?


A. Ask for an old X-ray
B. CT scan chest
C. Fine needle aspiration
D. Bronchoscopy
E. Open lung biopsy
F. PET Scan
This patient has a "solitary pulmonary nodule" defined as a 2cm or less coin-shaped lesion in the middle to lateral one third of the lung surrounded by normal parenchyma. Most of them are benign. Calcification of the nodules favors a benign lesion. "Popcorn" calcification is a hamartoma while "bulls eye" is a granuloma.

In low risk patients (i.e. < 40 years and non smokers) a solitary pulmonary nodule is not a sign of immediate alarm. The best approach is to ask for an old x-ray. If there is no change in it for the last 12 months it is considered benign. It is followed by a CXR every 3 months, for the next 12 months, and if there is no growth or no symptoms it is left as such.

High-risk patients i.e. smokers, require a full investigation work up. It starts with a CT scan chest just to get better picture of the lesion and its extent, followed by fine needle aspiration. If the lesion is missed, an open-lung biopsy is an option.

Bronchoscopy is not good at reaching peripheral lesions.

PET scan may also be a tool in the hands of the pulmonologist, to sort out benign from malignant lesions in a high-risk patient. PET scans are expensive and considered only in certain patients especially for metastatic workup.

Educational Objective:
In low risk patients (i.e. < 40 years and non smokers) a solitary pulmonary nodule is not a sign of immediate alarm. The best approach is to ask for an old x-ray.
A 36-year-old male comes to your office complaining of progressive fatigability. His past medical history is significant for infiltrative pulmonary tuberculosis diagnosed two months ago. His current treatment includes isoniazid, rifampin, ethambutol, and pyrazinamide. Laboratory values are:

Hemoglobin 8 g/dL
MCV 77 fL
MCHC 30%
ESR 17 mm/hr
Serum iron 170 µg/dL (N 50-150 µg/dL)
Total iron binding capacity (TIBS) 400 µg/dL (N 300-360 µg/dL)

Microscopy reveals two populations of red blood cells (RBC) - hypochromic and normochromic. What is the next best step in the management of this patient?

A. Bone marrow biopsy
B. Iron preparations
C. Folic acid
D. Folic acid and vitamin B12
E. Pyridoxine
The most probable diagnosis in this patient is acquired sideroblastic anemia. This condition results from defective heme synthesis, most commonly due to pyridoxine-dependent impairment in early steps of protoporphyrin synthesis. This patient is taking isoniazid, a well-known pyridoxine antagonist that can be responsible for this condition. Acquired sideroblastic anemia frequently manifests as microcytic hypochromic anemia simulating iron-deficiency anemia. Usually two groups of RBC can be demonstrated on microscopy - hypochromic and normochromic ("dimorphic" RBC population). Besides that, iron studies typically reveal increased serum iron concentration and increased total iron binding capacity (TIBS), which helps to differentiate sideroblastic anemia from iron-deficiency anemia (Choice B). In this patient, with an easily identifiable cause of vitamin B6 deficiency, administration of pyridoxine is the most reasonable next step. Bone marrow sampling can confirm the diagnosis by demonstrating ‘ringed sideroblasts,’ (a specific finding in this type of anemia), but it is not necessary in this patient (Choice A).

Folic acid or the combination of folic acid with vitamin B12 (Choice D) is employed in patients with megaloblastic anemia.

Educational Objective:
Sideroblastic anemia usually manifests as microcytic/hypochromic anemia simulating iron-deficiency anemia, but iron studies reveal elevated serum iron level and increased TIBS. In patients with an identifiable cause of vitamin B6 deficiency (alcoholism, drugs), the administration of pyridoxine can easily correct the problem.

*Extremely high yield question for USMLE!!!
A 41-year-old woman comes to the office for an evaluation of breast lump. She states that she has no medical problems, and feels generally well. She takes a multi-vitamin and Tums daily. She has not smoked a cigarette after she got married 21 year ago and drinks wine, only on weekends. Her only surgery was a bilateral tubal ligation last year, after her fourth child. On examination, you feel two axillary nodes on the left side and a 4 cm lump at 11 O’ clock position, on the left breast. Mammogram shows irregular micro-calcifications. FNA reveals it to be infiltrating ductal carcinoma. Which of the following is considered single most important prognostic factor in this patient?


A. TNM staging
B. Estrogen receptor and progesterone receptor status
C. Her-2/neu oncogene expression
D. Histological grade of the tumor
E. Irregular micro calcifications in mammogram
Tumor burden, based on TNM staging, is considered the single most important prognostic consideration in treating patient with breast cancer. The patients who have stage IV disease have the worst prognosis. The following are the other important factors in the prognosis of breast cancer, in the order of significance.

ER + and PR + are good prognostic features. (Choice E)
Over expression of Her-2/new oncogene is related to worse prognosis. (Choice C)
Histological grade also tends to reflect outcome. Poorly differentiated tumors have the worst prognosis (Choice B).
The data is inconclusive regarding mammogram findings and its relation to prognosis.

Educational Objective:
Tumor burden, based on TNM staging, is considered the single most important prognostic consideration in treating patients with breast cancer.
A 42-year-old male presents to your office complaining of fatigue. His past medical history is insignificant. He does not smoke or consume alcohol. His blood pressure is 120/70 mmHg and heart rate is 85/min. Physical examination is insignificant. Laboratory values are:

Hemoglobin


7.7 g/dL

MCV


72 fL

MCHC


28%

Leukocyte count


8,000/cmm

ESR


15 mm/hr

Serum iron and ferritin levels are decreased. What is the next best step in the management of this patient?


A. Iron supplementation
B. Dietary modifications
C. Test for occult blood in the stool
D. Work-up for malabsorption
E. Bone marrow sampling
This patient presents with hypochromic/microcytic anemia accompanied with depressed serum iron and ferritin levels. Such a clinical picture is typical for iron-deficiency anemia. The most common cause of iron-deficiency anemia is chronic blood loss. Dietary iron deficiency (Choice B) and malabsorption (Choice D) are rare causes of iron-deficiency anemia. Discovery of iron-deficiency anemia in an adult male or a post-menopausal woman should prompt the physician to look for gastrointestinal causes of blood loss. Tests for occult blood in the stool should be done first. It is not unusual for right-sided colon cancer to manifest as iron-deficiency anemia. Other gastrointestinal causes of chronic blood loss may include peptic ulcer disease, angiodysplasia, colonic diverticula, etc.

Iron supplementation (Choice A) helps to restore iron reserves, but the identification of the cause of the problem is the foremost issue.

Bone marrow sampling (Choice E) is not indicated in this patient.

Educational Objective:
Chronic gastrointestinal blood loss is the most common cause of iron-deficiency anemia in an adult male or a post-menopausal woman.
A 42-year-old white female, who has a long history of dysfunctional uterine bleeding, presented with exertional shortness of breath. On examination, she appears very pale and there is a pulmonic flow murmur heard over the second intercostal space. Her vitals are: BP: 130/80mm of Hg; HR: 80/min and regular; RR: 16/min; Temperature 36.7C(98F). Initial evaluation reveals hemoglobin of 8.2 gm/dl. WBC count is within normal limits. You are suspecting an iron deficiency anemia secondary to excessive bleeding. Which one of the following studies is most definite for the diagnosis of iron deficiency anemia?


A. A low serum ferritin level.
B. Increased total iron binding capacity.
C. A low serum iron concentration.
D. Bone marrow iron staining.
E. Total iron content of the gastric epithelial cells.
A low serum iron concentration is really nonspecific; it is low in iron deficiency anemia but it is also usually low in patients with anemia of chronic disease. The total iron binding capacity is usually elevated in iron deficiency anemia. However, many iron deficient patients may have normal TIBC. A low serum ferritin level almost always reflects iron deficiency anemia. However, serum ferritin can be normal or even elevated in patients with iron deficiency anemia as an acute phase reactant, but the bone marrow iron stain is the most definite way to diagnose an iron deficiency. But it is usually not required to make the diagnosis in real clinical practice. Since iron stores are depleted when iron deficiency anemia is present, and are usually normal or elevated in patients with microcytic anemia due to other causes like thalassemia.

Educational Objective:
Bone marrow iron stain is the most definite way to diagnose iron deficiency anemia.
A 44-year-old white male comes to office with persistent upper abdominal pain for the past 2 months. His roommate has peptic ulcer. He thought that he is also having an ulcer "down there" so he started taking his friend's medicines. They helped a bit initially, but soon he proved to be " too tough for them." He hardly wants to eat anything now and wonders if it is because of some reaction to those medicines. There is mild tenderness in the epigastric region. On esophagogastroduodenoscopy there is a mass in pylorus. Upon biopsy, it turns out to be an infiltrating low-grade gastric mucosa-associated lymphoid tissue (MALT) lymphoma. After further work-up, it is concluded that there are no regional lymph nodes involved. What is the next best step in management?


A. Radical gastrectomy with en bloc resection of spleen and regional nodes.
B. Give a combination of omeprazole, clarithromycin, and amoxicillin.
C. Give combination chemotherapy.
D. Radiotherapy.
E. Observation and strict surveillance.
Various studies have suggested an important role for Helicobacter pylori infection in the pathogenesis of low-grade gastric mucosa-associated lymphoid tissue (MALT) lymphoma. These lymphomas may regress after eradication of H. pylori by antibiotics. Though some recent works have shown certain pitfalls in this strategy, it still remains the most accepted and recommended next best step in the management of gastric mucosa-associated lymphoid tissue lymphoma without any metastasis. In the past, radical gastrectomy with en bloc resection of spleen and regional nodes was performed after the diagnosis but studies have shown it to be associative with high operative mortality.

Chemotherapy has a role if they fail to respond to eradication of H. Pylori. Therapeutic regimens usually include CHOP (cyclophosphamide, Adriamycin, vincristine, and prednisone) or CHOP-Bleomycin.

The role of Radiotherapy in the management of gastric lymphoma is controversial and it is mostly adjuvant to other regimes.

Observation is certainly not recommended in case of gastric lymphoma.

Educational Objective:
Eradication of H. pylori by antibiotics is the most accepted and recommended next best step in the management of gastric mucosa-associated lymphoid tissue lymphoma without any metastasis.
A 44-year-old white male comes to office with persistent upper abdominal pain for the past 2 months. His roommate has peptic ulcer. He thought that he is also having an ulcer "down there" so he started taking his friend's medicines. They helped a bit initially, but soon he proved to be " too tough for them." He hardly wants to eat anything now and wonders if it is because of some reaction to those medicines. There is mild tenderness in the epigastric region. On esophagogastroduodenoscopy there is a mass in pylorus. Upon biopsy, it turns out to be an infiltrating low-grade gastric mucosa-associated lymphoid tissue (MALT) lymphoma. After further work-up, it is concluded that there are no regional lymph nodes involved. What is the next best step in management?


A. Radical gastrectomy with en bloc resection of spleen and regional nodes.
B. Give a combination of omeprazole, clarithromycin, and amoxicillin.
C. Give combination chemotherapy.
D. Radiotherapy.
E. Observation and strict surveillance.
Various studies have suggested an important role for Helicobacter pylori infection in the pathogenesis of low-grade gastric mucosa-associated lymphoid tissue (MALT) lymphoma. These lymphomas may regress after eradication of H. pylori by antibiotics. Though some recent works have shown certain pitfalls in this strategy, it still remains the most accepted and recommended next best step in the management of gastric mucosa-associated lymphoid tissue lymphoma without any metastasis. In the past, radical gastrectomy with en bloc resection of spleen and regional nodes was performed after the diagnosis but studies have shown it to be associative with high operative mortality.

Chemotherapy has a role if they fail to respond to eradication of H. Pylori. Therapeutic regimens usually include CHOP (cyclophosphamide, Adriamycin, vincristine, and prednisone) or CHOP-Bleomycin.

The role of Radiotherapy in the management of gastric lymphoma is controversial and it is mostly adjuvant to other regimes.

Observation is certainly not recommended in case of gastric lymphoma.

Educational Objective:
Eradication of H. pylori by antibiotics is the most accepted and recommended next best step in the management of gastric mucosa-associated lymphoid tissue lymphoma without any metastasis.
A 44-year-old, African American, woman comes with complaints of loss of weight and blood in stools, for the last 1 year. Her mother is on chemotherapy for colon carcinoma. Her maternal uncle also had colon cancer, as did her first cousin who died of colon cancer at the age of 46. She is very worried that she might have the same cancer. According to her history, she falls within the criteria for Lynch syndrome. (HNPCC: Hereditary Nonpolyposis Colorectal Cancer). Apart from complete work-up for colon cancer, the patient should be evaluated for which of the following condition?


A. Pancreatic carcinoma
B. Hepatic carcinoma
C. Pseudomembranous colitis
D. Diverticulitis
E. Endometrial carcinoma
The patient has a strong family history of colon cancer and falls within the criteria for Lynch syndrome (HNPCC). The International Collaborative Group on Hereditary Nonpolyposis Colorectal Cancer established research criteria for the diagnosis of HNPCC (known as the Amsterdam Criteria I):

1. At least three relatives with colorectal cancer, one of whom must be a first degree relative of the other two
2. Involvement of two or more generations
3. At least one case diagnosed before age 50
4. Familial adenomatous polyposis has been excluded

HNPCC was classically divided into two subgroups on clinical grounds:
1. Hereditary site specific colon cancer or Lynch syndrome I
2. Cancer family syndrome or Lynch syndrome II

Lynch syndrome II is differentiated by a high risk of extra colonic tumors, the most common of which is endometrial carcinoma, which develops in up to 43 % of females in affected families.

Educational Objective:
Lynch syndrome II is differentiated by a high risk of extra colonic tumors, the most common of which is endometrial carcinoma, which develops in up to 43 % of females in affected families.
A 45-year-old white male comes to the emergency room with "all sorts of things going wrong with him", for the last few months. He has a severe headache, chest and abdominal pain. He is sweating profusely. He has lost weight recently, has diarrhea and palpitations. He is feeling hot all the time. Vital signs reveal BP: 190/100mm of Hg; PR: 124/min; RR 18/min; Temperature: 37.7 C(99.8F). On physical examination he has enlarged cervical lymph nodes. Examination of the thyroid reveals multiple thyroid nodules. FNA biopsy reveals thyroid C-cell hyperplasia. Which of the following can also be found on laboratory results?


A. Decreased serum calcitonin
B. Increased serum phosphorus
C. Decreased serum alkaline phosphatase
D. Decreased urine metanephrine
E. Increased serum calcium
The patient has the classical signs and symptoms of pheochromocytoma. This includes sudden attack of palpitations, headache, profuse sweating, and apprehension. Pain in chest or abdomen may be associated with nausea. In addition to that he has thyroid C-cell hyperplasia, which becomes medullary thyroid carcinoma. This makes him highly likely for Sipple syndrome (MEN Type IIa) which is also associated with hyperplasia (overgrowth) of the parathyroid gland. These following tests help confirm the diagnosis:
1. Increased serum calcium
2. Elevated calcitonin
3. Decreased serum phosphorus
4. Elevated urine metanephrine
5. Increased serum alkaline phosphatase
6. Elevated urine catecholamines
7. MRI of abdomen showing adrenal mass
8. Thyroid biopsy showing medullary carcinoma cells
9. Adrenal biopsy showing pheochromocytoma
10. Parathyroid biopsy showing tumor or hyperplasia
11. Radioimmune assay of parathyroid hormone showing increased level

Educational Objective:
MEN type 2a is a syndrome of pheochromocytoma, medullary carcinoma of thyroid and hyperparathyroidism.
A 45-year-old white male, who is a pure vegetarian and a known alcoholic, presented to primary care physician for easy fatigability. His examination is significant for pallor. His hemoglobin is 10.8 gm/dl; serum iron studies were within normal limits. A physician has placed him on folic acid 1mg daily. His hemoglobin went from 10.9 to 13 gm/dl over a period of several months. The patient was continued on folic acid for the last two years. For the last 6 months, the patient has gradually been losing his memory and has been having difficulty with maintaining balance. Which of the following is the most likely thing to consider at this point?


A. He has been treated with sub therapeutic doses of folic acid
B. He should be referred for CT of the abdomen with and without contrast immediately
C. The physician should have checked his vitamin B12 levels
D. Order FDA-ABS to rule out syphilis
E. Patients should have been started on pyridoxine
The folic acid replacement in patients with vitamin B12 deficiency may sometimes raise the hemoglobin level. However, the neurologic complications associated with vitamin B12 deficiency may continue or even worsen for a period of time. This patient does not require any CT scan or MRI, as this patient does not have any focal signs of disease. The FDA-ABS is not indicated as a screening test for syphilis and this patient does not have any risk factors and it is not indicated at this point in time. Vitamin B6 deficiency is not associated with memory problems or balance; however, it may be associated with peripheral neuropathy, which is usually seen in patients who are on INH therapy for tuberculosis. Isolated vitamin B6 deficiency is extremely rare. The patient is a pure vegetarian and this can cause vitamin B12 deficiency; so the patient’s physician should have checked his vitamin B12 levels.

Educational Objective:
Supplementation with folic acid may correct anemia but will not correct neurological complications in patients with vitamin B12 deficiency.
A 45-year-old white male, who is a pure vegetarian and a known alcoholic, presented to primary care physician for easy fatigability. His examination is significant for pallor. His hemoglobin is 10.8 gm/dl; serum iron studies were within normal limits. A physician has placed him on folic acid 1mg daily. His hemoglobin went from 10.9 to 13 gm/dl over a period of several months. The patient was continued on folic acid for the last two years. For the last 6 months, the patient has gradually been losing his memory and has been having difficulty with maintaining balance. Which of the following is the most likely thing to consider at this point?


A. He has been treated with sub therapeutic doses of folic acid
B. He should be referred for CT of the abdomen with and without contrast immediately
C. The physician should have checked his vitamin B12 levels
D. Order FDA-ABS to rule out syphilis
E. Patients should have been started on pyridoxine
The folic acid replacement in patients with vitamin B12 deficiency may sometimes raise the hemoglobin level. However, the neurologic complications associated with vitamin B12 deficiency may continue or even worsen for a period of time. This patient does not require any CT scan or MRI, as this patient does not have any focal signs of disease. The FDA-ABS is not indicated as a screening test for syphilis and this patient does not have any risk factors and it is not indicated at this point in time. Vitamin B6 deficiency is not associated with memory problems or balance; however, it may be associated with peripheral neuropathy, which is usually seen in patients who are on INH therapy for tuberculosis. Isolated vitamin B6 deficiency is extremely rare. The patient is a pure vegetarian and this can cause vitamin B12 deficiency; so the patient’s physician should have checked his vitamin B12 levels.

Educational Objective:
Supplementation with folic acid may correct anemia but will not correct neurological complications in patients with vitamin B12 deficiency.
A 46-year-old woman comes to you because of losing "many pounds" in the last 2 months, although she is eating much more than before. "It seems everything in me is vanishing", she says about her condition. She is drinking lemonade all the time, and has to rush to the bathroom to let it out. On examination, she is lean and her Temperature: 37.C (98.6F); BP: 120/84 mm Hg; PR: 78/min; RR: 14/min. She has a rash on her face, which is scaly. She also shows you another rash on her buttocks that is filled with clear fluid. Labs are ordered and results are

WBC 6,400mm3
Hemoglobin 8.7
Hematocrit 29%,
Platelets 193,000mm3
Sodium 144 mEq/L
Potassium 3.6 mEq/L
Bicarbonate 21 mEq/L
Blood urea nitrogen 16 mg/dL
Creatinine 0.6 mg/dL
Glucose 245 mg/dL

Which of the following is expected in this patient?


A. Increased thyroxine
B. Decreased insulin
C. Increased glucagon
D. Increased gastrin
E. Increased serotonin
The patient presents with classical triad of hyperglycemia, necrotizing dermatitis, and weight loss. This is the signature of glucagonoma. Glucagonoma is a tumor of the islet cells of the pancreas, which secrete the hormone glucagon. Glucagonoma is usually malignant and metastasis to the liver may occur. The excess glucagon causes symptoms such as glucose intolerance and hyperglycemia. It also causes a distinctive skin lesion called ‘necrolytic migratory erythema’. The diagnosis is confirmed by elevated fasting glucose, elevated serum glucagon, and pancreatic tumor revealed by CT scan. Surgical removal of the tumor is the preferred treatment. The tumor does not respond to chemotherapy.

Though there are some symptoms common to hyperthyroidism but the persistent hyperglycemia and skin lesion is more consistent with increased glucagon levels.

Due to hyperglycemia, insulin level is higher than normal, not low in glucagonoma.

Gastrin level is increased in gastrinomas while serotonin is increased in carcinoid syndrome.

Educational Objective:
Hyperglycemia, necrolytic migratory erythema and diarrhea should make you think about glucagonoma.
A 46-year-old woman has had worsening fatigue, headache, palpitations and shortness of breath on exertion for 1year. 6 months ago these complaints were attributed to her menopause. She has taken multivitamin and mineral tablets. For the recent 1 month she has had chest pain and nagging cough and her dyspnea has rapidly increased. 3 days ago she coughed up traces of blood. Her vital signs are: PR: 110/min; BP: 170/70mm of Hg; RR: 23/min; Temperature: 36.6C(97.8F). Physical exam is remarkable for very pale skin and 3/6 mid-systolic murmur in second right intercostal space. Lungs are clear to auscultation. Chest x-ray reveals right, upper mediastinal, 5cm mass with clear, lobulated borders.

The laboratory results are:

RBC 2.0
ESR 30
Hb 6.5
Hct 21
MCH 32.3
MCHC 32%
MCV 98
WBC 8,500 with normal differential
Reticulocyte count 0.1% of red cells
Platelet count 250.000

Iron 100 mcg/dL
Total iron-binding capacity 300 mcg/dL
Ferritin 100 ng/mL

ALT 8
AST 12
Bilirubin total//direct 0.8 / 0.1
LDH 70
Protein, Total 6.5
Albumin 3.7
Calcium 9.1 mg/dL

Most likely the mass is:


A. Non-Hodgkin’s lymphoma
B. Tuberculoma
C. Thymoma
D. Plasmacytoma
E. Neuroblastoma
This patient probably has pure red cell aplasia. Everything except RBC, ESR, Hb, Hct, and reticulocyte count are normal. When this disorder is associated with tumor, it most often is Thymoma.

Other secondary anemias are:
1) Autoimmune hemolytic: may be associated with various tumors. Reticulocytes, bilirubin and LDH would be increased.
2) Anemia of chronic disease: Iron and Total iron-binding capacity would be decreased. Ferritin would be increased.
3) Due to infiltration of bone marrow from tumor: All blood cell lines would be decreased. Immature cells would be present in the peripheral blood.
4) Due to Plasmocytoma or Multiple myeloma: Protein and calcium would be increased. Pure red cell aplasia is not a feature.

Educational Objective:
Thymoma is associated with pure red cell aplasia.
A 48-year-old woman had irregular densities discovered on a regular screening mammogram. Radiographic guided needle biopsy was performed and it turned out to be invasive lobular carcinoma. The tumor was hormone receptor (HR +). On further investigation, she had her axillary lymph nodes involved but the other breast seemed to be spared. No distant metastasis was detected. She underwent modified radical mastectomy, along with radiotherapy. Now she is being started on adjuvant chemotherapy and tamoxifen for 5 years. Which of the following cancers has its risk increased with use of tamoxifen?


A. Ovarian cancer
B. Endometrial cancer
C. Lung cancer
D. Pancreatic cancer
E. Cervical cancer
Tamoxifen is an anti-estrogen drug used to treat breast cancer. Research has shown that when tamoxifen is used as adjuvant therapy for early stage breast cancer, it reduces the risk of recurrence of the original cancer and also reduces the risk of developing new cancer in the other breast. However, there are risks associated with tamoxifen. Tamoxifen increases the risk of two types of cancer that can develop in the uterus: endometrial cancer, which arises in the lining of the uterus, and uterine sarcoma, which arises in the muscular wall of the uterus. Like all cancers, endometrial cancer and uterine sarcoma is potentially life threatening. Women who have had a hysterectomy (surgery to remove the uterus) and are taking tamoxifen are not at increased risk for these cancers. In the Breast Caner Prevention Trial (BCPT), women who took tamoxifen had more than twice the chance of developing endometrial cancer, compared with women who took a placebo (an inactive substance that looks the same as, and is administered in the same way as, tamoxifen). Most of the endometrial cancers that have occurred in women taking tamoxifen have been found in the early stages, and treatment has usually been effective. However, for some breast cancer patients who developed endometrial cancer while taking tamoxifen, the disease was life threatening.

Educational Objective:
Tamoxifen increases the risk of two types of cancer that can develop in the uterus: endometrial cancer, which arises in the lining of the uterus, and uterine sarcoma, which arises in the muscular wall of the uterus.
A 50-year-old man wants to talk to you about something, "absolutely confidential". After you assure him, he admits, "He is unable to get an erection and just can’t have sex." He wants to figure it out quickly because "he simply can't live like this." He has never been diagnosed with diabetes and denies other complaints. He has a 2 pack/day history of smoking for 30 years. On examination, his BP: 158/90mm of Hg; Temperature: 37.1C(98.8F); RR: 14/min; PR: 82/min. There is upper body obesity, rounded face, increased fat around the neck, and thinning of arms and legs. You find his skin to be bruised, fragile and thin. Laboratory reveals the following results.

Serum
Glucose 186 mg/dL
Sodium 142 mEq/L
Potassium 2.5 mEq/L
Chloride 94 mEq/L
Bicarbonate 38 mEq/L
Chest-X ray shows a large mass in left bronchus.

What is the most likely cause of patient’s condition?


A. Pituitary adenoma
B. Adrenal tumors
C. Ectopic ACTH syndrome
D. Familial cushing's syndrome
E. Exogenous steroid intake
The patient has all the features of Cushing's syndrome: including upper body obesity, rounded face, increased fat around the neck, and thinning arms and legs. Other symptoms appear in the skin, which becomes fragile and thin. It bruises easily and heals poorly. Purplish pink stretch marks may appear on the abdomen, thighs, buttocks, arms and breasts. The bones are weakened, and routine activities, i.e. bending, lifting, or rising from a chair may lead to backaches, rib and spinal column fractures. Most people have severe fatigue, muscle weakness, high blood pressure, and high blood sugar. Irritability, anxiety, and depression are also common. Sometimes sexual dysfunction can be the presenting complaint. The patients also have hypokalemic alkalosis.

Cushing's syndrome may be caused by a number of causes: including Adrenal neoplasia, Pituitary adenoma (Cushing's disease), Exogenous steroid intake, or some tumors that produces ACTH like Lung cancers. This condition is known as Ectopic ACTH syndrome. With long history of smoking and radiographic findings this should be high on the differential diagnosis in this case.

Educational Objective:
Ectopic ACTH production by lung cancer is an important cause of cushing's syndrome.
A 50-year-old man wants to talk to you about something, "absolutely confidential". After you assure him, he admits, "He is unable to get an erection and just can’t have sex." He wants to figure it out quickly because "he simply can't live like this." He has never been diagnosed with diabetes and denies other complaints. He has a 2 pack/day history of smoking for 30 years. On examination, his BP: 158/90mm of Hg; Temperature: 37.1C(98.8F); RR: 14/min; PR: 82/min. There is upper body obesity, rounded face, increased fat around the neck, and thinning of arms and legs. You find his skin to be bruised, fragile and thin. Laboratory reveals the following results.

Serum
Glucose 186 mg/dL
Sodium 142 mEq/L
Potassium 2.5 mEq/L
Chloride 94 mEq/L
Bicarbonate 38 mEq/L
Chest-X ray shows a large mass in left bronchus.

What is the most likely cause of patient’s condition?


A. Pituitary adenoma
B. Adrenal tumors
C. Ectopic ACTH syndrome
D. Familial cushing's syndrome
E. Exogenous steroid intake
The patient has all the features of Cushing's syndrome: including upper body obesity, rounded face, increased fat around the neck, and thinning arms and legs. Other symptoms appear in the skin, which becomes fragile and thin. It bruises easily and heals poorly. Purplish pink stretch marks may appear on the abdomen, thighs, buttocks, arms and breasts. The bones are weakened, and routine activities, i.e. bending, lifting, or rising from a chair may lead to backaches, rib and spinal column fractures. Most people have severe fatigue, muscle weakness, high blood pressure, and high blood sugar. Irritability, anxiety, and depression are also common. Sometimes sexual dysfunction can be the presenting complaint. The patients also have hypokalemic alkalosis.

Cushing's syndrome may be caused by a number of causes: including Adrenal neoplasia, Pituitary adenoma (Cushing's disease), Exogenous steroid intake, or some tumors that produces ACTH like Lung cancers. This condition is known as Ectopic ACTH syndrome. With long history of smoking and radiographic findings this should be high on the differential diagnosis in this case.

Educational Objective:
Ectopic ACTH production by lung cancer is an important cause of cushing's syndrome.
A 51-year-old African-American comes to you complaining of increased itching, especially after bathing. He also has occasional headaches and dizziness. There is no history of smoking or alcohol use. His sister has psychiatric problems and both parents died in a car accident, many years ago. His vitals are; Temperature: 36.7C(98.2F); BP: 148/90 mm Hg; PR: 77/min; RR: 12/min. On examination, his spleen is enlarged and he appears plethoric. CBC was ordered and came back as:

WBC 14,500mm3
Hemoglobin 21.5
Hematocrit 64%,
Platelets 521,000mm3
RBC count 7.6 million
MCH 30
MCHC 36
MCV 92
RDW 15.1 (n=10.3-14.1)

Which of the following is expected on further work up?


A. Absence of measurable erythropoetin in urine
B. Elevated ESR
C. Markedly elevated serum cortisol level
D. Elevated brain natriureteric peptide level
E. Hyponatremia and hyperkalemia
The patient has Polycythemia Vera (PCV). It is distinguished from the other kind of Myeloproliferative Disease (MPD) by the remarkable increase in RBC mass and total blood volume. A typical patient is an old plethoric male who may complain of pruritus after bathing. Symptoms such as headache, dizziness, and paresthesias are frequently related to hyperviscosity. Both thrombosis and bleeding can occur due to elevated platelet count and impaired platelet function Reversible moderate hypertension frequently occurs as a result of expanded blood volume. They also have granulocytosis and thrombocytosis on peripheral blood smear, and splenomegaly. Bone marrow is virtually always hypercellular. There is an elevated leukocyte alkaline phosphatase, normal oxygen saturation, and low erythropoetin level. A urinary erythropoetin assay reveals the absence of measurable erythropoetin in the urine.

Elevated ESR is seen in many autoimmune and hematological conditions like Multiple Myeloma. In polycythemia ESR will be very low or close to ‘0’

Markedly elevated serum cortisol level is seen in Cushing’s disease. His blood picture and splenomegaly doesn’t explain the Cushing’s syndrome.

Hyponatremia and Hyperkalemia are associated with mineralocorticoid abnormalities that do not explain the blood picture in this patient.

Educational Objective:
Know the criteria and laboratory findings for the diagnosis of Polycythemia Vera.
A 54-year-old male comes to the ER with complaints of cough and bloody sputum. He used to smoke 2 packs/day for 22 years but quit last month. He is having weight loss, anorexia, constipation, increased thirst, and easy fatigability, which he thinks is due to his depression. He has lost his wife 4 months ago and “life never felt same after that”. He is sure that he “cannot have cancer as he is not smoking anymore”. On examination, he is a slim man who appears pale. The vitals are stable though he appears a little irritable and short of breath. You order a complete blood work and chest x-ray and the results are as under:

Sodium 144 mEq/dL
Potassium 4.3 mEq/dL
Chloride 98 mEq/dL
Bicarbonate 21 mEq/dL
Calcium 14.5 mg/dL
BUN 48 mg/dL
Creatinine 2.0 mg/dL
Chest X Ray Hilar mass in the left lung

While the patient is in ER he starts getting very sick, somnolent and vomits twice. Your first step should be?


A. IV furosemide
B. CT head
C. IV normal saline
D. Emergency hemodialysis
E. Start IV pamidronate
The patient is going into hypercalcemic crisis. The leading symptoms that characterize the crisis are vomiting, oliguria, and anuria as well as somnolence and coma.

The management of acute hypercalcemia is vigorous hydration with normal saline (Choice C) followed by furosemide (Choice A) to maintain urine output at 200cc/hr.

We can also give bisphosphonate pamidronate 60-90mg in normal saline given over 3-24hr (Choice E). But IV normal saline corrects not only hypercalcemia but also replaces intravascular volume thus prevents prerenal azotemia, which this patient has already developed.

If the patient does not show improvement we should go for hemodialysis against a calcium-free dialysate (Choice D).

There is no role of doing CT without treating the obvious problem (Choice B).

Educational Objective:
IV normal saline is the first step in managing acute hypercalcemic crisis.
A 54-year-old male comes to the ER with complaints of cough and bloody sputum. He used to smoke 2 packs/day for 22 years but quit last month. He is having weight loss, anorexia, constipation, increased thirst, and easy fatigability, which he thinks is due to his depression. He has lost his wife 4 months ago and “life never felt same after that”. He is sure that he “cannot have cancer as he is not smoking anymore”. On examination, he is a slim man who appears pale. The vitals are stable though he appears a little irritable and short of breath. You order a complete blood work and chest x-ray and the results are as under:

Sodium 144 mEq/dL
Potassium 4.3 mEq/dL
Chloride 98 mEq/dL
Bicarbonate 21 mEq/dL
Calcium 14.5 mg/dL
BUN 48 mg/dL
Creatinine 2.0 mg/dL
Chest X Ray Hilar mass in the left lung

While the patient is in ER he starts getting very sick, somnolent and vomits twice. Your first step should be?


A. IV furosemide
B. CT head
C. IV normal saline
D. Emergency hemodialysis
E. Start IV pamidronate
The patient is going into hypercalcemic crisis. The leading symptoms that characterize the crisis are vomiting, oliguria, and anuria as well as somnolence and coma.

The management of acute hypercalcemia is vigorous hydration with normal saline (Choice C) followed by furosemide (Choice A) to maintain urine output at 200cc/hr.

We can also give bisphosphonate pamidronate 60-90mg in normal saline given over 3-24hr (Choice E). But IV normal saline corrects not only hypercalcemia but also replaces intravascular volume thus prevents prerenal azotemia, which this patient has already developed.

If the patient does not show improvement we should go for hemodialysis against a calcium-free dialysate (Choice D).

There is no role of doing CT without treating the obvious problem (Choice B).

Educational Objective:
IV normal saline is the first step in managing acute hypercalcemic crisis.
A 54-year-old male comes to the ER with complaints of cough and bloody sputum. He used to smoke 2packs/day for 22 years, but quit last month. He is having weight loss, anorexia, constipation, increased thirst, and easy fatigability, which he thinks is due to his depression. His wife died 4 months ago and “life never felt same after that”. On examination, he is a slim man who appears pale. The vitals are stable though he appears a little irritable and short of breath. You order a complete blood work and chest x-ray and the results are:

Sodium 144 mEq/dL
Potassium 4.3 mEq/dL
Chloride 98 mEq/dL
Bicarbonate 21 mEq/dL
Calcium 11.5 mg/dL
BUN 28 mg/dL
Creatinine 0.8 mg/dL
Chest X Ray Hilar mass in the left lung

What is the most probable diagnosis?


A. Tuberculosis
B. Squamous cell carcinoma of lung
C. Oat cell carcinoma of lung
D. Sarcoidosis
E. Adenocarcinoma of the lung
This patient’s symptoms are suggestive of hypercalcemia, which is confirmed by the laboratory results. Hypercalcemia is usually associated with Squamous Cell Carcinoma of Lungs (Remember sCa++mous) (Choice B) not Oat Cell Carcinoma (Choice C). Hypercalcemia usually develops as a late complication of cancer, and its appearance is very serious. Even though hypercalcemia is frequently due to metastatic involvement of the bone, it is also because of parathormone-related protein (PTHrP). PTHrP is similar to PTH in the receptor binding area. Binding to PTH results in increased calcium resorption from the bones and increased renal calcium resorption in the distal tubule.

The most common symptoms of hypercalcemia are feeling tired, difficulty thinking clearly, lack of appetite, pain, frequent urination, increased thirst, constipation, nausea, and vomiting. Most patients do not experience all of the symptoms of hypercalcemia, and some patients may not have any symptoms at all.

Sarcoidosis (Choice D) does cause hypercalcemia but usually patients have bilateral hilar adenopathy, evidence of erythema nodosum. However, hilar mass in a smoker is most likely a cancer, than anything else.

Hypercalcemia is not normally associated with Tuberculosis (Choice A). TB is very unlikely in USA unless patient has some kind of immunosuppression.

Adenocarcinoma is usually a peripheral lesion and usually not associated with hypercalcemia.

Educational Objective:
Hypercalcemia and hilar mass in a smoker of more than 45 years is most likely from a squamous cell carcinoma of the lungs.
A 54-year-old male comes to the ER with complaints of cough and bloody sputum. He used to smoke 2packs/day for 22 years, but quit last month. He is having weight loss, anorexia, constipation, increased thirst, and easy fatigability, which he thinks is due to his depression. His wife died 4 months ago and “life never felt same after that”. On examination, he is a slim man who appears pale. The vitals are stable though he appears a little irritable and short of breath. You order a complete blood work and chest x-ray and the results are:

Sodium 144 mEq/dL
Potassium 4.3 mEq/dL
Chloride 98 mEq/dL
Bicarbonate 21 mEq/dL
Calcium 11.5 mg/dL
BUN 28 mg/dL
Creatinine 0.8 mg/dL
Chest X Ray Hilar mass in the left lung

What is the most probable diagnosis?


A. Tuberculosis
B. Squamous cell carcinoma of lung
C. Oat cell carcinoma of lung
D. Sarcoidosis
E. Adenocarcinoma of the lung
Explanation:

This patient’s symptoms are suggestive of hypercalcemia, which is confirmed by the laboratory results. Hypercalcemia is usually associated with Squamous Cell Carcinoma of Lungs (Remember sCa++mous) (Choice B) not Oat Cell Carcinoma (Choice C). Hypercalcemia usually develops as a late complication of cancer, and its appearance is very serious. Even though hypercalcemia is frequently due to metastatic involvement of the bone, it is also because of parathormone-related protein (PTHrP). PTHrP is similar to PTH in the receptor binding area. Binding to PTH results in increased calcium resorption from the bones and increased renal calcium resorption in the distal tubule.

The most common symptoms of hypercalcemia are feeling tired, difficulty thinking clearly, lack of appetite, pain, frequent urination, increased thirst, constipation, nausea, and vomiting. Most patients do not experience all of the symptoms of hypercalcemia, and some patients may not have any symptoms at all.

Sarcoidosis (Choice D) does cause hypercalcemia but usually patients have bilateral hilar adenopathy, evidence of erythema nodosum. However, hilar mass in a smoker is most likely a cancer, than anything else.

Hypercalcemia is not normally associated with Tuberculosis (Choice A). TB is very unlikely in USA unless patient has some kind of immunosuppression.

Adenocarcinoma is usually a peripheral lesion and usually not associated with hypercalcemia.

Educational Objective:
Hypercalcemia and hilar mass in a smoker of more than 45 years is most likely from a squamous cell carcinoma of the lungs.
A 54-year-old male had undergone an emergency colonic resection following an extensive ischemic colitis. The surgery was uneventful. The patient has been on peperacillin and tazobactam (Zosyn) for the past five days. He was NPO (nothing by mouth) for the past five days. He has a significant alcoholic history. On postoperative day six, he developed bleeding from the venipuncture site. His temperature is 36.7 C (98 F), blood pressure is 120/76 mmHg, pulse is 80/min, and respirations are 16/min. Lab results are:

Hb 11.5g/dL
MCV 88 fl
Platelet count 160,000/cmm
Leukocyte count 7,500/cmm
Segmented neutrophils 68%
Bands 1%
Eosinophils 1%
Lymphocytes 24%
Monocytes 6%
Prothrombin time 20 sec (INR=1.9)
Partial thromboplastin time 45 sec

His family history is insignificant. Which of the following is the most likely cause of his condition?

A. Thrombotic thrombocytopenic purpura
B. Hemolytic uremic syndrome
C. Idiopathic thrombocytopenia purpura
D. Vitamin deficiency
E. Medication affect
F. Disseminated intravascular coagulation
G. Factor VIII deficiency
H. Factor IX deficiency
I. Von Willebrand’s disease
The clinical presentation of this patient is suggestive of an acquired bleeding disorder most likely due to vitamin-K deficiency. Vitamin K is a fat-soluble vitamin and plays an important role in hemostasis, by serving as a cofactor in the enzymatic carboxylation of glutamic acid residues on prothrombin complex proteins. The body gets vitamin K from two sources: exogenous from the food, which is absorbed in the small intestine, and endogenous from the bacterial production of vitamin K in the intestine. The patient in the given vignette was postoperatively NPO and was taking a broad spectrum antibiotic; thus, both the sources of vitamin K in him were compromised. Inadequate dietary intake, intestinal malabsorption, and loss of storage sites due to hepatocellular disease are the three major causes of vitamin-K deficiency. A 30-day store of vitamin K is stored in a normal liver; however, an acutely sick person can become vitamin-K deficient in 7-10 days.

Vitamin-K deficiency leads to a fall in plasma levels of all the prothrombin complex proteins (factor 2, 7, 9, 10, and protein C and S). This would lead to prolonged PT followed by prolonged PTT and PT >> PTT. Parenteral administration of vitamin K rapidly restores the stores in 8-10 hours. Fresh frozen plasma (FFP) is indicated for the management of an acute hemorrhage.

(Choice A) Thrombotic thrombocytopenic purpura (TTP) is a rare form of consumptive thrombocytopenia caused by widespread platelet thrombi that form in the microcirculation. Thrombocytopenia, microangiopathic hemolytic anemia, fever, and neurological signs characterize it.

(Choice B) Hemolytic uremic syndrome (HUS) often follows diarrhea. HUS and TTP belong to a similar spectrum. HUS is also manifested by thrombocytopenia, microangiopathic hemolytic anemia, and fever; but, instead of more neurologic signs, patients will have renal failure.

(Choice F) Disseminated intravascular coagulation is a potentially life threatening coagulation disorder characterized by initial thrombotic phase followed by a phase of procoagulant consumption and secondary fibrinolysis. Labs would show thrombocytopenia, prolonged PT, PTT, and thrombin time. Reduced fibrinogen level from depletion of coagulation proteins and elevated fibrin degradation products (FDP) from intense secondary fibrinolysis is also seen.

(Choice E) Heparin-induced thrombocytopenia (HIT) is a frequent cause of drug-induced thrombocytopenia in hospitalized patients. It can cause bleeding and, rarely, paradoxical thrombosis.

(Choices G and H) Factor VIII and IX deficiency are an X-linked inherited plasma coagulation disorders and would cause prolongation of PTT. Also, it would manifest early in life. It is impossible to clinically distinguish between factor VIII and IX deficiency and a detailed laboratory work up is required.

(Choice I) Von Willebrand’s disease is the most frequently diagnosed inherited bleeding disorder in adults. Labs would show an increase in bleeding time with an increase in PTT. PT will be normal.

Educational Objective:
The typical vitamin-K deficiency is seen in a patient who has been kept NPO for a prolonged period of time and receiving broad-spectrum antibiotics. Labs would show prolongation of PT followed by prolonged PTT; PT is elevated more than PTT.

*You may get a similar presentation of vitamin-K deficiency in a newborn who had not received prophylactic vitamin K (home born) for the prevention of hemorrhagic disease of the newborn.
A 54-year-old male had undergone an emergency colonic resection following an extensive ischemic colitis. The surgery was uneventful. The patient has been on peperacillin and tazobactam (Zosyn) for the past five days. He was NPO (nothing by mouth) for the past five days. He has a significant alcoholic history. On postoperative day six, he developed bleeding from the venipuncture site. His temperature is 36.7 C (98 F), blood pressure is 120/76 mmHg, pulse is 80/min, and respirations are 16/min. Lab results are:

Hb 11.5g/dL
MCV 88 fl
Platelet count 160,000/cmm
Leukocyte count 7,500/cmm
Segmented neutrophils 68%
Bands 1%
Eosinophils 1%
Lymphocytes 24%
Monocytes 6%

Prothrombin time 20 sec (INR=1.9)
Partial thromboplastin time 45 sec

His family history is insignificant. Which of the following is the most likely cause of his condition?


A. Thrombotic thrombocytopenic purpura
B. Hemolytic uremic syndrome
C. Idiopathic thrombocytopenia purpura
D. Vitamin deficiency
E. Medication affect
F. Disseminated intravascular coagulation
G. Factor VIII deficiency
H. Factor IX deficiency
I. Von Willebrand’s disease
The clinical presentation of this patient is suggestive of an acquired bleeding disorder most likely due to vitamin-K deficiency. Vitamin K is a fat-soluble vitamin and plays an important role in hemostasis, by serving as a cofactor in the enzymatic carboxylation of glutamic acid residues on prothrombin complex proteins. The body gets vitamin K from two sources: exogenous from the food, which is absorbed in the small intestine, and endogenous from the bacterial production of vitamin K in the intestine. The patient in the given vignette was postoperatively NPO and was taking a broad spectrum antibiotic; thus, both the sources of vitamin K in him were compromised. Inadequate dietary intake, intestinal malabsorption, and loss of storage sites due to hepatocellular disease are the three major causes of vitamin-K deficiency. A 30-day store of vitamin K is stored in a normal liver; however, an acutely sick person can become vitamin-K deficient in 7-10 days.

Vitamin-K deficiency leads to a fall in plasma levels of all the prothrombin complex proteins (factor 2, 7, 9, 10, and protein C and S). This would lead to prolonged PT followed by prolonged PTT and PT >> PTT. Parenteral administration of vitamin K rapidly restores the stores in 8-10 hours. Fresh frozen plasma (FFP) is indicated for the management of an acute hemorrhage.

(Choice A) Thrombotic thrombocytopenic purpura (TTP) is a rare form of consumptive thrombocytopenia caused by widespread platelet thrombi that form in the microcirculation. Thrombocytopenia, microangiopathic hemolytic anemia, fever, and neurological signs characterize it.

(Choice B) Hemolytic uremic syndrome (HUS) often follows diarrhea. HUS and TTP belong to a similar spectrum. HUS is also manifested by thrombocytopenia, microangiopathic hemolytic anemia, and fever; but, instead of more neurologic signs, patients will have renal failure.

(Choice F) Disseminated intravascular coagulation is a potentially life threatening coagulation disorder characterized by initial thrombotic phase followed by a phase of procoagulant consumption and secondary fibrinolysis. Labs would show thrombocytopenia, prolonged PT, PTT, and thrombin time. Reduced fibrinogen level from depletion of coagulation proteins and elevated fibrin degradation products (FDP) from intense secondary fibrinolysis is also seen.

(Choice E) Heparin-induced thrombocytopenia (HIT) is a frequent cause of drug-induced thrombocytopenia in hospitalized patients. It can cause bleeding and, rarely, paradoxical thrombosis.

(Choices G and H) Factor VIII and IX deficiency are an X-linked inherited plasma coagulation disorders and would cause prolongation of PTT. Also, it would manifest early in life. It is impossible to clinically distinguish between factor VIII and IX deficiency and a detailed laboratory work up is required.

(Choice I) Von Willebrand’s disease is the most frequently diagnosed inherited bleeding disorder in adults. Labs would show an increase in bleeding time with an increase in PTT. PT will be normal.

Educational Objective:
The typical vitamin-K deficiency is seen in a patient who has been kept NPO for a prolonged period of time and receiving broad-spectrum antibiotics. Labs would show prolongation of PT followed by prolonged PTT; PT is elevated more than PTT.

*You may get a similar presentation of vitamin-K deficiency in a newborn who had not received prophylactic vitamin K (home born) for the prevention of hemorrhagic disease of the newborn.
A 54-year-old male was brought to the ER because of severe pain in the abdomen, and diarrhea for the past 24 hours. He was confused and was crying of pain. On examination there was extensive abdominal distention and fresh blood in stools. His vitals were, Temperature: 38.6C(101.6F); BP: 82/58 mm Hg; PR: 118/min; RR: 24/min. A stat CBC and serum electrolytes were ordered. An IV saline solution was started after the blood had been drawn. The results of work up came back as:

Sodium 136 mEq/L
Potassium 3.6 mEq/L
Bicarbonate 18 mEq/L
Blood urea nitrogen 22 mg/dL
Creatinine 1.3 mg/dL
Glucose 86 mg/dL
WBC 35,000mm3
Hemoglobin 13
Hematocrit 38%,
Platelets 460,000mm3

The leukocyte distribution is described as:

Promyelocyte 2%
Myelocytes 7%
Metamyelocytes 18%
Bands 32%
Segmented neutrophils 24%
Lymphocytes 15%

The patient temperature continued to remain elevated during the night of admission. A broad-spectrum antibiotic was added to IV infusion. A repeat CBC was ordered next morning, which showed WBC count of 118,000/mm3 with essentially the same differential distribution of leukocytes. His leukocyte alkaline phosphatase score was elevated. What is the most probable diagnosis for his blood abnormality?


A. Chronic myeloid leukemia with blast crisis
B. Chronic lymphocytic leukemia
C. Leukemoid reaction
D. Secondary polycythemia
E. Myelodysplastic syndrome
F. Drug reaction
A leukemoid reaction is a marked increase in leucocytes by a severe infection or inflammation. Chronic myeloid leukemia (CML) and a leukemoid reaction are indistinguishable on a peripheral blood film. The sudden elevation in the total leukocyte count and marked increase (to the left) of granulocyte precursors suggests a serious infection with or without an underlying leukemic state. The Leukocyte Alkaline Phosphatase (LAP) test is of diagnostic importance in distinguishing between CML and a leukemoid reaction. In CML, LAP score is decreased. However, an increased LAP score may be encountered in CML because of subsequent secondary infections. So, the presence of Philadelphia chromosome is more diagnostic for CML and should be obtained in suspected cases.

Lymphocytes are the predominant cells in CLL.

Primary Polycythemia Vera is characterized by increased RBC mass, mild granulocytosis and low erythropoietin level. LAP may be increased in PCV. In secondary polycythemia usually only RBC mass is elevated and the rest of the cell lines are usually normal.

Myelodysplastic syndromes (MDS) are clonal stem cell disorders, which may progress to acute leukemias. They are characterized by pancytopenia.

Educational Objective:
Know that leukocyte alkaline phosphatase score is high in leukemoid reactions and usually low in chronic myeloid leukemia.
A 55-year-old Caucasian male is hospitalized after a massive hemorrhage from a duodenal ulcer. The hemorrhage was stopped with endoscopic manipulation. Two liters of Ringer lactate was infused, and two units of packed red blood cells (RBC) ordered. Ten minutes after the transfusion of packed RBC had been started, the patient was complaining of tremor and feeling ‘chilly.’ His temperature is 39.4 C (103 F), blood pressure is 120/76 mmHg, pulse is 100/min, and respirations are 24/min. The transfusion was stopped, and acetaminophen was given. Direct antiglobulin test and plasma free hemoglobin level are negative. Urinalysis is normal. The reaction abated three hours after the transfusion had started. Which of the following actions could have prevented the reaction described?


A. Warming the blood
B. Using whole blood
C. Careful cross matching of the blood
D. Infusing calcium gluconate
E. Washing cells
The clinical scenario described is most consistent with febrile transfusion reaction. It is characterized by fever and chills that usually respond to NSAIDs and acetaminophen. Unlike more serious reactions, like hemolytic reaction or bacterial contamination of the transfused blood, no hemodynamic abnormalities and renal dysfunction are usually present. The febrile reaction is believed to be caused by antibodies in the patient’s plasma reacting with the donor’s leukocytes, and the antibodies frequently have HLA specificity. Therefore, leukocyte depletion techniques (although not commonly employed due to high cost) can reduce the probability of febrile transfusion reaction. These techniques are cell washing, use of frozen deglycerolized red cells, use of leukocyte depletion RBC filters, etc.

Transfusion of whole blood (Choice B) carries the increased risk of febrile reaction, because centrifugation removes up to 70% of leucocytes.

Careful cross matching of the blood (Choice C) prevents hemolytic reactions. Acute hemolytic transfusion reactions (AHTRs) also present in a similar way. The most common complaints are fever and chills. The classic triad of fever, back pain, and red or pink color urine is rarely seen. So, this has to be ruled out by stopping the transfusion and obtaining a sample for a direct antiglobulin test (Coombs) and for plasma free hemoglobin. Urinanalysis will also show the presence of hemoglobin.

Calcium gluconate infusion (Choice D) is employed in rare cases of severe hypocalcaemia following massive blood transfusion.

Warming the blood (Choice A) is recommended only during rapid massive transfusion to prevent hypothermia.

Educational Objective:
Febrile reaction is a common transfusion reaction that is caused by antibodies in the patient’s plasma reacting with the donor’s leukocytes. Leukocyte depletion techniques, like cell washing, decrease the probability of febrile, nonhemolytic transfusion reaction.
A 55-year-old woman comes to her primary care physician with complaints of pain, itching and red streaks in her left arm. She feels that her arm is "simply not the same". She had a similar episode in her chest almost 2 weeks ago, but it went away on its own. She has no other complaints, except for heartburn and some upper abdominal pain that has been there for a few months. She takes antacids for it, but has never got over it completely. She thinks that she has lost some weight but she feels happy about it, as she always wanted to lose weight. Her vitals are stable and there is mild epigastric tenderness. She has tender, erythematous, cord like veins palpable over left arm and some over the chest. She smokes 1-2pack/day for 15years and drinks alcohol, only on parties. What is the next best step in this patient?


A. Upper GI endoscopy with barium swallow
B. Colonoscopy
C. Antibiotics and reassurance
D. CT scan abdomen
E. Spiral CT chest
F. Chest –X rayA 55-year-old woman comes to her primary care physician with complaints of pain, itching and red streaks in her left arm. She feels that her arm is "simply not the same". She had a similar episode in her chest almost 2 weeks ago, but it went away on its own. She has no other complaints, except for heartburn and some upper abdominal pain that has been there for a few months. She takes antacids for it, but has never got over it completely. She thinks that she has lost some weight but she feels happy about it, as she always wanted to lose weight. Her vitals are stable and there is mild epigastric tenderness. She has tender, erythematous, cord like veins palpable over left arm and some over the chest. She smokes 1-2pack/day for 15years and drinks alcohol, only on parties. What is the next best step in this patient?


A. Upper GI endoscopy with barium swallow
B. Colonoscopy
C. Antibiotics and reassurance
D. CT scan abdomen
E. Spiral CT chest
F. Chest –X ray
The patient has classical symptoms of Migratory thrombophlebitis, known as Trousseau's syndrome. Patients with Trousseau's syndrome usually have an occult tumor, which may not always be detectable at the time of presentation. The most common tumor is an adenocarcinoma. Based on the reviews, 24% of patients have pancreatic carcinoma, 20% lung carcinoma, 13% prostate cancer, 12% stomach cancer, 9% acute leukemia, and 5% colon cancer. The thrombophlebitis of the atypical sites such as arm and chest is an important clue for the underlying carcinoma. So, the presence of this along with symptoms of upper GI discomfort immediately leads to the possibility of pancreatic carcinoma that needs to be thoroughly investigated.

Pancreatic cancer is the 4th leading cause of death from cancer in the U.S. The disease is slightly more common in men than in women, and risk increases with age. The cause is unknown, but the incidence is greater in smokers. Abdominal CT scan is the test of first choice when the suspicion of Pancreatic cancer is so high.

Trousseau's syndrome that occurs in about 10% of patients with pancreatic adenocarcinoma is attributed to the elaboration of platelet-aggregating factors and procoagulants from the tumor or its necrotic products. Both venous and arterial thrombi can occur. So coagulation studies and Doppler studies may be needed later but not at this point.

These venous thrombi can embolize to the lungs. A large "saddle" pulmonary embolus may result in sudden severe dyspnea and demise of the patient. Smaller emboli, as in this case, may produce pulmonary infarctions with some dyspnea and chest pain. In such a case a V/Q scan or spiral CT chest will be needed.

Upper GI endoscopy is an invasive procedure, and is considered only when the CT abdomen is negative for pancreatic cancer. Chest x-ray and colonoscopy can be considered if the symptomatology is different.

Antibiotics and reassurance is certainly not the best step for a patient with weight loss, chronic upper abdominal pain, and evidence of recurrent migratory thrombophlebitis.

Educational Objective:
Recognize migratory thrombophlebitis and consider appropriate investigation to find out the possible occult malignancy.
A 55-year-old woman comes with complaints of episodes of night sweats and fever, for the last 6 weeks. She denies any cough or other respiratory complaints. She has lost 20lb(9kg) and does not feel like eating anything. She denies smoking or alcohol use. Her brother was recently diagnosed with polycythemia vera. Physical examination reveals pallor and splenomegaly. Chest-X ray is within normal limits. Blood work is ordered and results are:

WBC 66,100/cmm
Hemoglobin 8.70
Hematocrit 30%,
Platelets 610,000/cmm

The leukocyte distribution on differential smear was

Promyelocyte 1%
Myelocyte 8%
Metamyelocyte 15%
Bands 35%
Segmented Neutrophils 25%
Lymphocytes 14%
Monocytes 2%

Further tests are ordered. Which one of the following is expected in this patient?


A. Elevated leukocyte alkaline phosphatase
B. Increased bone marrow iron
C. Absence of measurable erythropoetin in urine
D. Presence of auer rods
E. Decreased leukocyte alkaline phosphatase
F. Presence of tartrate resistant acid phosphatase
Chronic Myelogenous Leukemia (CML) is characterized by leukocytosis, anemia, and increased number of mature granulocytic forms, such as segmented neutrophils and band forms on peripheral blood film. The disease is mostly seen after the age of 50 and presents with fatigue, malaise, low-grade fever, anorexia, weight loss and bone pains. Night sweats and fever associated with increased metabolism due to granulocytic cell turn over may occur. Examination of a bone marrow biopsy specimen reveals hypercellularity with prominent granulocytic hyperplasia. The leukocyte alkaline phosphate score is low. The only other diseases where this may happen are hypophosphatemia and paroxysmal nocturnal hemoglobinuria (PNH).

Elevated Leukocyte Alkaline Phosphatase is characteristic of Leukemoid Reaction. Presence of Philadelphia chromosome and the low leukocyte alkaline phosphatase makes the CML more likely than the leukemoid reaction.

Increased bone marrow iron is seen in Hemachromatosis, anemia of chronic disease, and sideroblastosis.

Absence of measurable erythropoietin in Urine is an important diagnostic feature of polycythemia vera.

Auer rods are seen in AML.

Tartrate resistant acid phosphatase is present in hairy cell leukemia.

Educational Objective:
Know that low leukocyte alkaline phosphatase with leukocytosis is classic for chronic myelogenous leukemia.
A 55-year-old woman comes with complaints of episodes of night sweats and fever, for the last 6 weeks. She denies any cough or other respiratory complaints. She has lost 20lb(9kg) and does not feel like eating anything. She denies smoking or alcohol use. Her brother was recently diagnosed with polycythemia vera. Physical examination reveals pallor and splenomegaly. Chest-X ray is within normal limits. Blood work is ordered and results are:

WBC 66,100/cmm
Hemoglobin 8.70
Hematocrit 30%,
Platelets 610,000/cmm

The leukocyte distribution on differential smear was

Promyelocyte 1%
Myelocyte 8%
Metamyelocyte 15%
Bands 35%
Segmented Neutrophils 25%
Lymphocytes 14%
Monocytes 2%

Further tests are ordered. Which one of the following is expected in this patient?

A. Elevated leukocyte alkaline phosphatase
B. Increased bone marrow iron
C. Absence of measurable erythropoetin in urine
D. Presence of auer rods
E. Decreased leukocyte alkaline phosphatase
F. Presence of tartrate resistant acid phosphatase
Chronic Myelogenous Leukemia (CML) is characterized by leukocytosis, anemia, and increased number of mature granulocytic forms, such as segmented neutrophils and band forms on peripheral blood film. The disease is mostly seen after the age of 50 and presents with fatigue, malaise, low-grade fever, anorexia, weight loss and bone pains. Night sweats and fever associated with increased metabolism due to granulocytic cell turn over may occur. Examination of a bone marrow biopsy specimen reveals hypercellularity with prominent granulocytic hyperplasia. The leukocyte alkaline phosphate score is low. The only other diseases where this may happen are Wilson’s disease and paroxysmal nocturnal hemoglobinuria (PNH).

Elevated Leukocyte Alkaline Phosphatase is characteristic of Leukemoid Reaction. Presence of Philadelphia chromosome and the low leukocyte alkaline phosphatase makes the CML more likely than the leukemoid reaction.

Increased bone marrow iron is seen in Hemachromatosis, anemia of chronic disease, and sideroblastosis.

Absence of measurable erythropoietin in Urine is an important diagnostic feature of polycythemia vera.

Auer rods are seen in AML.

Tartrate resistant acid phosphatase is present in hairy cell leukemia.

Educational Objective:
Know that low leukocyte alkaline phosphatase with leukocytosis is classic for chronic myelogenous leukemia.
A 56-year-old white male comes to the office for a periodic visit. He is feeling healthy and enjoys his life, as much as he can in his stressed environment. He smokes 2-3pack/day and drinks socially. His Temperature: 37C(98.6F); BP: 158/94 mm Hg; PR: 78/min; RR: 14/min. On examination, plethora of face is noted and there is moderate splenomegaly. Blood tests are ordered and result comes back as:

WBC 15,600/mm3
Hemoglobin 17 gm/dL
Hematocrit 54%,
Platelets 550,000mm3
RBC count 7 million
Sodium 141 mEq/L
Potassium 3.6 mEq/L
Bicarbonate 18 mEq/L
Blood urea nitrogen 16 mg/dL
Creatinine 0.6 mg/dL
Glucose 95 mg/dL

On peripheral blood smear an increase in neutrophilic bands and segmented neutrophils were observed. The erythrocyte indices were all within normal range.

What is the most probable diagnosis?


A. Idiopathic hypertension
B. Cushing disease
C. Polycythemia vera
D. Congestive heart failure
E. Primary hyperaldosteronism
Polycythemia Vera is a clonal Myeloproliferative Disease (MPD) of the pluripotent hematopoietic stem cell with an unknown etiology. It is characterized by increased RBC mass, mild granulocytosis, and low erythropoetin level. Exam usually shows plethoric face and splenomegaly. The patient may have normal erythrocyte indices. The bone marrow is virtually always hypercellular. A diagnosis of Polycythemia Vera requires an accurate assessment of plasma volume and RBC mass. Reversible moderate hypertension frequently occurs as a result of expanded volume. Polycythemia Vera may burn out to myelofibrosis and appearance of teardrop shaped RBCs should not be a surprise. The treatment of Polycythemia Vera is phlebotomy to keep HCT < 45%.

The patient has increased RBC mass, which need to be investigated first before labeling him with Idiopathic hypertension.

An increased total blood volume may occur in CHF but it is combined with other signs and symptoms like dyspnea, JVD, hepatomegaly etc. This patient clearly has no CHF on exam.

Cushing syndrome has its whole panorama that doesn’t fit well with this patient. It can have hypertension and plethoric face but they usually do not have high hematocrit, splenomegaly. They can have high white count with increased segments and decreased lymphocytes.

He has normal sodium and potassium so primary hyperaldosteronism is not the best answer. Again these patients do not have any splenomegaly and high hematocrit.

Educational Objective:
Polycythemia can present with hypertension. Know various presentations of polycythemia vera and try to identify it from other causes of hypervolemia.
A 56-year-old woman is complaining of cough for the past 4 months. The cough was dry initially, but only yesterday she was scared-to-death to see phlegm with blood. She is not feeling well and thinks that she has lost "a lot of weight" recently, despite no apparent change in her diet. She swears that she has never smoked a cigarette in her life. There is no history of night sweats nor contact with any person with similar complaints. She is a housewife who rarely drinks, and can't think of using recreational drugs. She is completely faithful to her husband. Her husband is also a "religious non-smoker". Her vitals are stable and lung examination reveals no significant findings. Her chest x-ray shows 1.5cm lesion in the right peripheral lung field and no evidence of any hilar or mediastinal lymph node enlargement. What is the most probable diagnosis?


A. Tuberculosis
B. Sarcoidosis
C. Small cell carcinoma of lung
D. Squamous cell carcinoma of lung
E. Adenocarcinoma of lung
Adenocarcinoma of lung has the least association with smoking. This is the most common type of lung cancer, even in smokers. It is usually located peripherally and consists of columnar cells growing along the septa. It is present as a solitary nodule and may be detected incidentally. There are few types of adenocarcinoma, out of which bronchoalveolar type is most important. It is related to pulmonary scars, such as in pulmonary fibrosis. They metastasize early to adrenals, bone, and CNS.

The patient's history, along with the solitary lesion is not consistent with Tuberculosis or Sarcoidosis. TB is very rare in USA unless the patient is immunosuppressed such as HIV patients. The classic presentation of sarcoidosis is a bilateral hilar lymphadenopathy. Sarcoidosis is unlikely to present with hemoptysis.

Small-cell and squamous-cell carcinoma of the lung are the other possible cancers in this patient, but not the most likely cancers. They are usually more prevalent in smokers, and often centrally located.

Educational Objective:
Adenocarcinoma of lung has the least association with smoking. It is usually located peripherally and consists of columnar cells growing along the septa. It presents as a solitary nodule and may be detected incidentally.
A 56-year-old, known alcoholic, male is admitted in ICU for the management of hepatic encephalopathy. He was started on oral neomycin, lactulose, low protein and high carbohydrate diet. His vital are: BP: 116/78 mm Hg; PR: 78/min; Temperature: 37.5C(99.5F); RR: 14/min.

His labs revealed

Hb 10.2%
WBC 7,400
Hct 27%
Platelets 42,000/mm3
PT 18 sec
INR 2.02
PTT 31 sec

Which of the following should be administered to this patient at this time?


A. Fresh frozen plasma
B. Cryoprecipitate
C. Vitamin-K
D. Platelets
E. Packed RBC
This patient is having elevated PT and INR. Fresh frozen plasma is usually indicated if the patient is actively bleeding or if the patient requires immediate correction like for surgery or for invasive procedures.

Platelet transfusion is indicated only if the patient is actively bleeding and the platelet count is less than 20,000 to 30,000/mm3.

Vitamin K for 3 days is usually sufficient to correct elevated PT in these types of patients.

Cryoprecipitate is required for the coagulation factor deficiencies like factor VIII deficiency.

Packed red cells are usually indicated if Hb is < 8 in asymptomatic patients (or) < 10 in symptomatic patients.

Educational Objective:
Patients with hepatic failure are given vitamin K for correction of their elevated PT and fresh frozen is indicated in this setting if patients are actively bleeding or if patients need immediate surgery or invasive procedure.
A 58-year-old nurse was diagnosed with node-positive metastatic breast cancer. Her whole body bone scan is positive for metastatic disease. Tumor is estrogen receptor (ER) and progesterone receptor (PR) positive. She is being treated with systemic chemotherapy and hormonal therapy (Tamoxifen). She comes to you for monthly examinations. She is also feeling weak with vague muscle, joint, and bone pains. On physical examination, there is a hard, well-defined dominant mass in the left breast. Laboratory examination shows the following results.

Sodium 145 mEq/dL
Potassium 3.9 mEq/dL
Chloride 103 mEq/dL
Bicarbonate 24 mEq/dL
Calcium 11.3 mg/dL
BUN 18 mg/dL
Creatinine 0.8 mg/dL
Glucose 146 mg/dL

Which of the following is the next best step in management?


A. Stop tamoxifen
B. Start pamidronate
C. IV fluids
D. Furosemide
E. IV mithramycin
The patient’s complaints are due to metastatic disease, not entirely from hypercalcemia. Hypercalcemia in patients with malignancy is due to increased bone resorption and release of calcium from bone. The pathology involves osteolytic metastases with local release of cytokines, tumor secretion of parathyroid hormone-related protein (PTHrP), and tumor production of calcitriol.

The bisphosphonates are the drugs of choice for mild to moderate hypercalcemia; they are relatively nontoxic and are more potent than intravenous saline. There is increasing evidence that bisphosphonates may prevent skeletal complications (reduce bone pain, fracture risk) and perhaps improve survival in patients with multiple myeloma or breast cancer. According to the clinical practice guidelines published by the ASCO, in all women with metastatic breast cancer and radiographic lytic bone disease who are receiving either hormone therapy or chemotherapy, intravenous pamidronate is recommended.

IV fluids and furosemide are the treatment of hypercalcemic crisis.

Here most important is the treatment of underlying tumor so stopping Tamoxifen is not the right choice. Tamoxifen has nothing to do with hypercalcemia.

Although it is highly effective, IV Mithramycin is rarely used because of its toxicity.

Educational Objective:
The bisphosphonates are the drugs of choice for mild to moderate hypercalcemia.
A 6-year-old African-American child is brought in by his father for complaints of easy fatigability and pallor. He reports the above symptoms in his son after he was treated with "some medication" for a recent diarrhea. Physical examination is normal except for pallor and multiple petechiae. Laboratory values are as follows:

Hb 8.0 g/dL
WBC 12,000/cmm
Platelets 50,000/cmm
Blood glucose 118 mg/dL
Serum Na 135 mEq/L
Serum K 5.3 mEq/L
Chloride 110 mEq/L
Bicarbonate 18 mEq/L
BUN 38 mg/dL
Serum creatinine 2.5 mg/dL
Total bilirubin 3 mg/dL
Direct bilirubin 0.5 mg/dL
PT 12 seconds
APTT 30 seconds.
LDH 400 IU/L
Reticulocyte count 3%.

A peripheral blood smear reveals giant platelets and multiple schistocytes. What is the most likely underlying pathophysiology for his pallor?


A. Sickle cell anemia
B. Thalassemia
C. Vitamin B12 deficiency
D. Folate deficiency
E. Microangiopathic hemolytic anemia
F. Lead poisoning
G. Disseminated intravascular coagulation
H. Idiopathic thrombocytopenic purpura
I. Renal failure
Hemolytic Uremic Syndrome is typically a disease of young children. It is usually preceded by an acute diarrheal illness (due to the pathogens Escherichia coli serotype 0157:H7 and Shigella, Salmonella, Yersinia, and Campylobacter species). It is less commonly preceded by an upper respiratory infection. The hallmark finding of this condition is a microangiopathic hemolytic anemia. Other features are acute renal failure, fever, oliguria (or anuria) and thrombocytopenia. GI bleeding is also a common symptom. Physical examination frequently reveals purpura and hypertension. Schistocytes are found in the peripheral smear, which represent fragmented RBCs. The peripheral smear also may contain giant platelets. Intravascular hemolysis results in elevated lactate dehydrogenase (LDH), indirect bilirubin and reticulocyte count. BUN and creatinine are markedly elevated. A moderate leukocytosis may be present. The urine contains hemoglobin, hemosiderin, albumin, RBCs, WBCs, and casts.

Choice A: Sickle cell anemia is another type of hemolytic anemia, which can be easily ruled out in this case, due to the absence of sickle cells on peripheral smear.

Choice B: Patients with Thalassemia present with hepatosplenomegaly (absent in this patient). Peripheral blood smear may reveal target cells, microcytosis, hypochromia, and anisopoikilocytosis, but not schistocytes. BUN and Cr will be normal.

Choice C: Vitamin B12 deficiency is characterized by macrocytosis, and hypersegmented neutrophils on peripheral smear.

Choice D: Folate deficiency also presents with macrocytosis and hypersegmented polymorphonuclear leucocytes (PMNs).

Choice F: Lead poisoning causes a microcytic anemia, with basophilic stippling of RBCs on peripheral smear. Besides, it also presents with some GI symptoms.

Choice G: Prothrombin time (PT) and activated partial thromboplastin time (aPTT) would be expected to be high in patients with DIC, unlike this patient, who has normal values for these tests.

Choice H: ITP is characterized by an isolated thrombocytopenia. On peripheral smear, the morphology of red cells is normal.

Choice I: Bleeding in renal failure patients is usually due to dysfunctional platelets; it is very unusual to see schistocytes and hemolytic picture from renal failure.

Educational objective:
Recognize HUS in a child who has recently recovered from a diarrheal illness and presents with acute renal failure, microangiopathic hemolytic anemia, fever, thrombocytopenia and characteristic peripheral smear finding of schistocytes.

*Extremely high yield question for USMLE!!!
A 6-year-old Caucasian boy is hospitalized for acute sinusitis that was accompanied with intensive nasal bleeding. Past medical history is significant for recurrent pulmonary infections and several hospitalizations for parenteral antibiotic therapy. The sweat chloride test is positive. The blood tests reveal a prothrombin time (PT) of 20 seconds. Which of the following coagulation factors is most likely to be deficient in this patient?


A. Fibrinogen
B. Hageman factor
C. Factor VIII
D. Factor VII
E. Factor V
Severe recurrent respiratory infections and positive sweat chloride test are characteristic for cystic fibrosis. Fat malabsorption is common in patients with cystic fibrosis and is due to exocrine deficiency of the pancreas; therefore, deficiency of fat-soluble vitamins, including vitamin K, is frequently present if no vitamin supplementation is provided. Besides that, antibiotic therapy may predispose this patient to vitamin K deficiency. Vitamin K is an important cofactor for the liver enzyme gamma-glutamyl carboxylase which adds carboxyl groups to the glutamate residues of several coagulation factors: factors II, VII, IX, and X as well as proteins C and S. These extra carboxyl groups increase the affinity of these proteins for the phospholipids on the surface of the platelets.

Fibrinogen (Choice A) and factor V (Choice E) does not require vitamin K-dependent posttranslational modification. Hageman factor (Choice B) and factor VIII (Choice C) are not vitamin K dependent; besides that, their deficiency does not prolong PT.

Educational Objective:
Vitamin K is an important cofactor in posttranslational modification of several coagulation factors: factors II, VII, IX, and X as well as proteins C and S.
A 6-year-old, African-American, boy presents with fever and pain in his extremities for the last several hours. Pain is unrelated to movement or posture. His vitals are, PR: 102/min; BP: 110/70mm of Hg; Temperature: 38C(100.4F); RR: 18/min. On examination pallor, jaundice and splenomegaly are noted. Both his lower legs are tender to palpation.
Lab tests show

Hematocrit 20%
WBC count 13,000/micro-L
Platelet count 180,000/miro-L
Bilirubin 7 mg/dL
Direct bilirubin 0.6 mg/dL.
Peripheral blood smear shows sickle shaped cells
Reticulocyte count 7%.

Hemoglobin electrophoresis is performed which confirms the diagnosis of sickle cell anemia. The child is treated with analgesia, hydration, and oxygen therapy and the painful episode subsides. Which of the following measures will be employed to prevent aplastic crisis in this child?


A. Supplementation with folic acid
B. Supplementation with iron
C. Vaccination against parvovirus
D. Vaccination against pneumococcus
E. Treatment with hydroxyurea
A patient with sickle cell anemia may present in many ways. Its clinical course is marked with recurrent painful episodes. The patient may also undergo hemolytic or aplastic crisis. Sickle cell anemia is a type of hemolytic anemia in which abnormal sickle shaped cells are being destroyed by spleen resulting in anemia. Bone marrow tends to compensate for this loss of RBCs by increasing red blood cell production. In those cases where folic acid is deficient, bone marrow fails to compensate for the increased destruction of RBCs and aplastic crisis occurs. Another cause of aplastic crisis in a patient of sickle cell anemia is infection with parvovirus that causes transient impairment of bone marrow function.

Supplementation with folic acid is recommended in all cases of sickle cell anemia to prevent the occurrence of aplastic crisis.

Though parvovirus is a cause of aplastic crisis in a patient with sickle cell anemia, vaccination against this virus is not done.

Repeated micro-infarction of spleen results in its functional impairment and ultimately autosplenectomy by the age of 20. As a result the patient is at an increased risk of infection with capsulated organism like pneumococci so vaccination against pneumococcus is recommended in a patient with sickle cell anemia. But infection with pneumococcus is not a cause of aplastic crisis and is not an appropriate choice here.

In sickle cell anemia, hemolysis is of extra-cellular variety and iron released during red cell destruction in the spleen is conserved in the body and there is usually no iron deficiency. Iron deficiency occurs in cases of intra-vascular hemolysis as a result of loss of hemoglobin in the urine.

Hydroxyurea is used in sickle cell anemia when painful episodes occur very frequently. It produces its beneficial effect by increasing HbF levels.

Educational Objective:
Supplementation with folic acid is recommended in all cases of sickle cell anemia to prevent the occurrence of aplastic crisis.
A 60-year-old man comes to office with persistent complaints of malaise and easy fatigability, for the past 8 months. On examination, he appears pale. PR: 93/min; BP: 127/84mm Hg; Temperature: 37C(98.6F); RR:16/min. Fecal occult blood test is negative. Further testing is ordered that include CBC, serum electrolytes and colonoscopy. The results are:

WBC 7,600mm3
Hemoglobin 8.8 gm/dL
Hematocrit 30%,
RBC cont 3.6 million
Platelets 211,000mm3

RBC Indices were:

MCV 65 fL
MCH 16.5 pg
MCHC 26%
Reticulocyte count 0.5%

Which one of the following is expected on iron studies in this patient?

Serum Iron, Ferritin, TIBC, Transferrin Saturation


A. Low,High,Low,Low to normal
B. Normal,Normal,Normal,Normal
C. Increased, Normal,Increased,Normal to increased
D. Low,Low,High,Low
E. Low,Low,Low,Low
The patient has microcytic hypochromic anemia due to chronic blood loss, which may not always be detected by FOBT. RBC indices reveal a decreased MCV, MCH, and a decreased reticulocyte count which correlates with Iron deficiency anemia. The peripheral blood picture of anisocytosis, microcytosis, hypochromia and poikilocytosis further confirms the diagnosis. Iron deficiency anemia results in decreased serum iron, ferritin, and percent saturation (Serum Iron / TIBC) along with an increased TIBC (Choice D).

Choice A is seen in anemia of chronic disease.

Choice B is the normal condition.

Choice C is seen in Sideroblastic anemia.

Choice E is just a distracter.

Educational Objective:
Know the iron studies in various types of microcytic anemias. Low serum iron, ferritin, and increased TIBC indicates iron deficiency anemia.
A 62-year-old male comes into your clinic with complaints of generalized malaise, bony pain, headaches, and a low-grade fever. He says that he has been feeling sick for a few months. He does have a history of hypertension. Blood work reveals anemia and hypercalcemia. The skull x-ray is shown below. What is the most likely diagnosis in this patient?

A. Lymphoma
B. Osteoporosis
C. Multiple myeloma
D. Paget’s disease
E. Ewing’s tumor
Multiple myeloma is a malignant proliferation of plasma cells chiefly in the bone marrow. Cells may form solitary tumor masses known as plasmacytomas. Most patients present with bone pain. Pathological fractures may occur and vertebral collapse may lead to spinal cord compression. Bone lesions are typically osteolytic. Recurrent infections, renal failure, and neuropathies may also occur. Serum and urine electrophoresis is used to detect and quantitate the monoclonal protein. Skeletal radiographs, typically of the skull, will show punched out lesions and are characteristic of multiple myeloma (Choice C).

(Choice A) Lymphomas are tumors of the immune system. Approximately 65% of lymphomas are of B cell origin, and 30-40% are of T cell origin. Lymphomas usually present with asymptomatic lymph node enlargement or may present with fever, night sweats, weight loss, and pruritus. Diagnosis is by CT scan, followed by biopsy. Staging is done to determine therapy, which consists of chemotherapy/radiation.

(Choice B) Osteoporosis is defined as a reduction of bone density below the level required for mechanical support. Remodeling of bone is continuous and density decreases whenever the rate of resorption exceeds formation. The vertebrae, wrist, hip, humerus, and tibia are particularly prone to fractures. Blood levels of calcium, phosphorous, and alkaline phosphatase are normal. Bone density scans (DEXA) are diagnostic. Treatment is directed towards prevention of further bone loss and calcium administration.

(Choice D) Paget’s disease of the bone is due to excessive resorption of bone mediated by osteoclasts, followed by replacement of normal marrow by vascular, fibrous connective tissue. Patients will present elevated alkaline phosphatase, deformity of bone, enlargement of skull, and pain. Hearing loss and cranial nerve palsies may also occur. Sarcoma is a long-term complication.

(Choice E) Ewing’s tumor is a malignant sarcoma composed of small round cells that occur more frequently in the first decade of life. Most tumors are located at diaphysis of the long bones, although any bone may be involved. Ewing’s sarcoma is a malignant lesion with a low incidence of cure by ablative surgery with or without radiation. However, aggressive chemotherapy and radiation have proven successful in improving survival of patients with Ewing’s sarcoma.

Educational Objective:
A skull x-ray showing punched out lesions is typical of multiple myeloma.
A 64-year-old male patient is being treated with unfractionated heparin for deep vein thrombosis. On the 4th day of treatment his platelet count dropped to 80,000. His previous platelet count on day 3 was in normal range. Patient denies any bleeding related complications and is completely asymptomatic. His complete examination doesn’t reveal any signs of bleeding. His vitals are: BP: 128/80mm of Hg; PR: 78/min; RR: 20/min and afebrile. What is the most appropriate next step in the management of this patient?


A. Switch to low molecular weight heparin immediately
B. Stop heparin and start warfarin
C. Stop heparin
D. Start plasmapheresis
E. Stop heparin and start platelet transfusion
Thrombocytopenia is a well-recognized serious complication of heparin therapy. There are two types of heparin-induced thrombocytopenia (HIT). HIT II usually occurs within 4 to 10 days after heparin treatment has started. HIT II is an autoimmune disorder characterized by the formation of antibodies against the heparin-platelet factor 4 complex. The other type, HIT I, occurs within the first two days after heparin therapy. The mechanism is non-immune and is possibly due to a direct affect of heparin on platelet activation.

The first step in a patient with HIT should be immediate cessation of all exposure to heparin including LMW heparin since it may cross react with the heparin-induced antibodies and may induce heparin-dependent IgG antibody formation. However, cessation of heparin alone is often not sufficient, since these patients remain at risk for thrombosis. There are two recommended alternatives to heparin in patients who developed HIT and requires ongoing anticoagulation (e.g mechanical heart valve): danaparoid and a direct thrombin inhibitor such as lepirudin or argatroban.

The use of warfarin without other anticoagulants should be avoided in patients with HIT until the platelet count rises above 100,000/µL. Without heparin, warfarin therapy alone may increase the risk of venous limb gangrene in patients with deep vein thrombosis.

Plasmapheresis is usually reserved for symptomatic and severe thrombocytopenia patients. Again, heparin should be stopped first.

Educational Objective:
Heparin must be stopped in patients who develop heparin-induced thrombocytopenia.
A 64-year-old woman comes to the hospital with the complaints of an irritating sore throat for 3 months. She has smoked 1pack/day of cigarettes for 29 years and continues to smoke. She had been jailed twice for DUI (driving under influence) and has gone to Alcohol Rehabilitation 1 year ago, though she denies any alcohol intake now. She has no fever or any other complaints. She had a complete hysterectomy several years ago for symptomatic fibroids. Her vitals are stable. On physical examination, a 1.5-cm right cervical lymph node was noted. Rest of the examination is completely normal. Biopsy of the lymph node revealed metastatic squamous cell carcinoma. CT scan of the chest was negative. What is the next best step in this patient?


A. Empirical chemotherapy
B. Panendoscopy
C. Repeat CT in 3 months
D. Complete breast examination and mammogram
E. Radical neck dissection
F. Radiotherapy
The patient has squamous cell carcinoma of mucosa of head and neck. These are very common in people who smoke and drink a lot of alcohol. The first manifestation of such a tumor may be a palpable cervical lymph node. The best test for such patients is panendoscopy (triple endoscopy= esophagoscopy, bronchoscopy, and laryngoscopy) to detect primary tumor, or in most case, tumors. After the primary tumor/s is detected, we should biopsy it to confirm diagnosis. CT scan may be repeated later to have a close look at its extent. Open dissection of the lymph node at this stage is always discouraged because it makes the task of surgeon difficult later as it obscures the surgical approach to the cancer. After the diagnosis is established then we can choose from the following options the most appropriate one for this patient.
1.Radical Neck Dissection
2.Tumor resection
3.Platinum based chemotherapy
4.Radiotherapy
5.Palliative management

Educational Objective:
Squamous cell carcinoma of mucosa of head and neck may present with palpable cervical lymph node. The best test for such patients is panendoscopy to detect the primary tumor.
A 64-year-old, African-American, male who rarely sees the doctor and doesn’t believe in screening procedures, comes to office with the complaint of severe back pain. He has not felt comfortable for the past 4 months and is having severe pain in the back. He has found it difficult to sleep. He has hypertension, for which he takes hydrochlorothiazides regularly. He thinks that he has lost some weight. On examination, his BP is 165/90mm of Hg, while rest of vitals are stable. On rectal examination you find a rock hard 2 cm nodule in prostate. Biopsy shows poorly differentiated adenocarcinoma. Bone scans shows wide spread metastasis all over his skeleton. What is the most appropriate treatment for his metastatic disease?


A. Leuprolide
B. Diethylstilbestrol (DES)
C. Flutamide
D. Radicle prostatectomy
E. Complete androgen blockade with Leuprolide and Flutamide
The patient has prostate cancer that has metastasized to bones. Prostate cancer is one of the leading causes of cancer deaths among men in USA. At the time when the cancer is in late stages, surgery is not performed unless needed for palliation.

Palliative radiation, along with anti-androgen therapy, is the treatment of choice. Anti-androgen therapy consists of Leuprolide (LHRH analogue). Flutamide is considered inferior to LHRH analogues, though some studies have shown added benefit of combining the two. In various studies the results with anti-androgen therapy are comparable to orchiectomy; however, long-term survival in orchiectomy is not yet completely studied.

Complete androgen blockade (CAB) using combination of LHRH analogue with an antiandrogen has been tested in various trials. Most studies reported a higher incidence of adverse events with CAB, because of the addition of an antiandrogen, and even the benefit is very modest. So far, there is no clear consensus on the advantage of CAB over monotherapy as first-line therapy for metastatic prostate cancer.

DES is associated with increased cardiovascular disease and is not used anymore.

Educational Objective:
Palliative radiation, along with anti-androgen therapy, is the treatment of choice for metastatic prostate cancer. Anti-androgen therapy consists of Leuprolide (LHRH analogue). Flutamide is considered inferior to LHRH analogues.
A 65-year-old Caucasian male had undergone cardiac catheterization followed by aortic valve replacement for severe aortic stenosis and coronary artery bypass grafting for three-vessel disease. His postoperative course was complicated by atrial fibrillation and an urinary tract infection. His other medical problems include hypertension, diabetes, and hypercholesterolemia. He is also receiving heparin, ciprofloxacin, and amiodarone. On postoperative day five, he developed prolonged bleeding from the venipuncture site. His labs shows:

Hb 11.5 g/dL
MCV 88 fl
Platelet count 50,000/cmm
Leukocyte count 7,500/cmm
Segmented neutrophils 68%
Bands 1%
Eosinophils 1%
Lymphocytes 24%
Monocytes 6%
Prothrombin time 12 sec (INR=1.0)
Partial thromboplastin time 65 sec

His preoperative labs were unremarkable. What is the most likely cause of these findings in this patient?


A. Thrombotic thrombocytopenic purpura
B. Hemolytic uremic syndrome
C. Idiopathic thrombocytopenia purpura
D. Vitamin deficiency
E. Medication affect
F. Disseminated intravascular coagulation
G. Factor VIII deficiency
H. Factor IX deficiency
I. Von Willebrand’s disease
The coagulation profile in this patient is suggestive of elevated PTT and a decrease in platelet count. While the elevated PTT is a therapeutic effect of heparin, the thrombocytopenia is an adverse effect of the same.

Heparin is a common cause of thrombocytopenia in hospitalized patients. HIT (Heparin-Induced Thrombocytopenia) is defined as a fall in the platelet count to < 150 × 109 /L or a decrease by 50% in patients with pre-existing thrombocytopenia. It is seen in about 5-15% of patients taking therapeutic heparin. It is more common with unfractionated heparin derived from bovine lung than porcine intestine. Onset of HIT is usually seen between 3-15 days after initiation of heparin, but it may be as short as hours in previously sensitized patients. Resolution of thrombocytopenia occurs within 4-5 days of discontinuation of heparin. Paradoxical thrombosis (white clot syndrome) may be seen in about 0.4% of patients taking heparin and may be life threatening. Co-administration of oral anticoagulants in patients who need long term anti-coagulation and use of LMW heparin can decrease the risk of HIT. Patients on heparin should be regularly followed up with platelet counts.

(Choice A) Thrombotic thrombocytopenia purpura (TTP) is a rare form of consumptive thrombocytopenia caused by widespread platelet thrombi that form in the microcirculation. Thrombocytopenia, microangiopathic hemolytic anemia, fever, and neurological signs characterize it.

(Choice B) Hemolytic uremic syndrome (HUS) often follows diarrhea. HUS and TTP belong to a similar spectrum. HUS is also manifested by thrombocytopenia, microangiopathic hemolytic anemia, and fever; but, instead of more neurologic signs, patients will have renal failure.

(Choice C) Idiopathic thrombocytopenia purpura (ITP) is a diagnosis of exclusion and is characterized by thrombocytopenia. All other cell lines, PT, and PTT will be normal.

(Choice D) Vitamin-K deficiency leads to a fall in plasma levels of all the prothrombin complex proteins (factor 2, 7, 9, 10, and protein C and S). This would lead to a prolonged PT followed by a prolonged PTT and PT >> PTT. Thrombocytopenia is not a feature.

(Choice F) Disseminated intravascular coagulation is a potentially life threatening coagulation disorder characterized by an initial thrombotic phase followed by a phase of procoagulant consumption and secondary fibrinolysis. Labs would show thrombocytopenia, prolonged PT, PTT, and thrombin time. A reduced fibrinogen level from depletion of coagulation proteins and elevated fibrin degradation products (FDP) from intense secondary fibrinolysis is also seen.

(Choices G and H) Factor VIII and IX deficiency are an X-linked inherited plasma coagulation disorders and would cause prolongation of PTT. Also, it would manifest early in life. It is impossible to clinically distinguish between factor VIII and IX deficiency, and a detailed laboratory work up is required.

(Choice I) Von Willebrand’s disease is the most frequently diagnosed inherited bleeding disorder in adults. Labs would show an increase in bleeding time with an increase in PTT. PT will be normal. Platelet count is normal.

Educational Objective:
HIT is seen in about 5-15% of patients taking therapeutic heparin with onset between 3-15 days after initiation of heparin and resolution within 4-5 days of discontinuation of heparin. While the elevated PTT is a therapeutic effect of heparin, the thrombocytopenia is an adverse effect of the same.
A 65-year-old female is being evaluated for "generalized depression". The patient is feeling weak and fatigued, since her husband died 4 months ago. She does not have any suicidal thoughts but is losing interest in daily activities. She quit smoking 24 years ago, and drinks 1-2 beers/wk. On examination, she is pale with both cervical and supra-clavicular lymphadenopathy. Blood work reveals:

WBC 28,600mm3
Hemoglobin 10.6
Hematocrit 40%,
Platelets 96,000mm3

The distribution of leukocytes was as under

Lymphocytes 77%
Bands 6%
Neutrophils 16%
Monocytes 1%

Some variants of lymphocytes and smudge type cells were also present. What is the most probable diagnosis?


A. Lymphoblastic lymphoma
B. Hodgkin's disease
C. Chronic lymphocytic leukemia
D. Chronic myeloid leukemia
E. Angioimmunoblastic lymphadenopathy
Chronic lymphocytic leukemia (CLL) is the most common type of leukemia in Western Countries making up approximately 30% of leukemias in USA. The patients are often asymptomatic and are diagnosed incidentally. Signs include lymphadenopathy and splenomegaly. Lymphocytosis is the classical signature mark of the disease. Most of these cells have an over-matured look because of the hypercondensed nuclear chromatin material. Smudge cells are highly characteristic. As the disease advances, anemia and thrombocytopenia are seen.

Lymphoblastic Lymphoma has a number of blast cells in its blood picture.

CML is one of the myeloproliferative syndromes and the whole picture consists of leukocytosis with left shift (more myelomonocytes, neutrophils etc.) basophilia and marked splenomegaly. Lymphocytosis is not a feature of CML.

Angioimmunoblastic lymphadenopathy is an idiopathic disease that causes a generalized lymphadenopathy, skin rash, hepatosplenomegaly and a Coomb's positive hemolytic anemia.

In early Hodgkin’s disease peripheral smear is usually normal and the diagnosis requires lymph node biopsy. Presence of Reid Sternberg cell is pathognomonic.

Educational Objective:
Chronic lymphocytic leukemia is almost always seen in elderly patients; presence of marked leukocytosis with predominant lymphocytosis and smudge cells are the diagnostic features.
A 65-year-old female, with a past medical history significant for coronary artery disease, presents with complaints of feeling weak and fatigued, all the time. She has lost interest in her daily activities and doesn't feel like eating anything. She has lost 30 lb(13.6kg) in the last 2 months. Her husband died last year in a road traffic accident. She was "depressed" initially, but with the help of her son she came out of it in 4 months. She thinks that same thing has come back again. She had hepatitis when she was 37 and did not remember whether it was A, B, or C. She was treated for it for a "few" months. She denies smoking or alcohol use. Her vitals are stable but there is hepatomegaly (increased liver span) on abdominal examination.

Her labs are

sodium 138
chloride 103
potassium 3.2
bicarbonate 24
BUN 18
creatinine 0.8
glucose 97
calcium 10.0
phosphorus 2.9
albumin 4.6
total bilirubin 0.8
ALT 32
AST 190
ALP 57
GGT 58.

CBC reveals

WBC 12.0
hemoglobin 11.6
hematocrit 39.9
platelets 168,000/microL.

What is the next best step in management?


A. ERCP
B. PTC
C. Refer her to psychiatrist
D. CT scan abdomen
E. Barium swallow followed by endoscopy
The patient has a history of hepatitis, along with hepatomegaly. Even with normal liver function tests, her symptoms and history makes her a candidate for hepatocellular carcinoma. Most of hepatocellular cancers are detected by just a mass, felt on palpation. Hepatocellular carcinoma accounts for 80% to 90% of all liver cancers. It occurs more often in men, than women, and occurs mostly in people 50-60yrs old. The cause of liver cancer is unknown, but contributing factors include chronic liver disease, viral hepatitis (especially hepatitis B and C), hemochromatosis, known hepatic (liver) carcinogens, and toxins (mycotoxins) found in foods. Abdominal CT scan should be done to check for the mass in liver.

ERCP and PTC are not indicated for hepatomegaly without jaundice and no apparent biliary tract disease. She had depression, but first her hepatomegaly has to be investigated. She has no signs and symptoms of upper GI disease; so Barium swallow with endoscopy is not the best choice.

Educational Objective:
Abdominal CT scan is the test of choice for suspected liver cancer.
A 65-year-old Hispanic male is brought to the Emergency Room after an episode of seizure. He was diagnosed with non-small cell lung carcinoma two years ago, and it was treated surgically. He had no residual disease at that time. A CT scan of the brain with contrast done in the emergency room, shows a solitary cortical mass in the right hemisphere of the brain at the gray-white matter junction. His chest x-ray is clear. He is admitted for further management and started on phenytoin. Which of the following is the most appropriate management for this patient?


A. Focal radiation to brain mass
B. Whole brain radiation
C. Surgical resection of the mass
D. Combination chemotherapy
E. Palliative pain therapy and seizure prophylaxis
The patient has a solitary metastatic brain lesion from non-small cell lung carcinoma (NSCLC). Metastatic brain tumors are the most common type of intracranial brain tumors. The metastasis may be solitary or multiple. Brain metastasis is frequently seen in patients with NSCLC, and it is most commonly located directly beneath the gray-white matter interface. Patients with solitary brain metastasis have a median survival period of one month in absence of any treatment. Glucocorticoids have an important role in palliating symptoms of intra-cranial hypertension; however, they have not been shown any long-term survival benefit.

(Choice C) Surgical resection is the preferred management of solitary brain metastasis. The role of surgery in patients with brain metastases is to provide immediate relief from the mass effect of the tumor, to establish a histologic diagnosis, and to improve local control of the tumor. Whole brain radiotherapy is given after surgical resection to destroy microscopic residual disease and to provide a better quality of life.

(Choices A and B) Radiotherapy for the treatment of brain metastases is meant to improve neurologic deficits caused by the tumor deposits. Studies have shown that survival rates following whole brain radiotherapy are not as good as those following surgical resection for solitary brain metastasis. Focal radiotherapy or brachytherapy is generally used in patients with unresectable brain metastasis and for those who cannot undergo whole brain irradiation.

(Choice D) Chemotherapy might be used in case of brain metastasis from chemosensitive tumors, like small cell lung cancer, lymphoma, or choriocarcinoma; however, it has not shown to be beneficial in the management of solitary brain metastasis from NSCLC.

Educational Objective:
Surgical resection followed by whole brain radiation is the standard of practice in the management of solitary brain metastasis with stable extracranial disease. Multiple brain metastases are best treated with palliative whole brain radiation.

*Extremely high yield question for USMLE!!!
A 65-year-old Hispanic male is brought to the Emergency Room after an episode of seizure. He was diagnosed with non-small cell lung carcinoma two years ago, and it was treated surgically. He had no residual disease at that time. A CT scan of the brain with contrast done in the emergency room, shows a solitary cortical mass in the right hemisphere of the brain at the gray-white matter junction. His chest x-ray is clear. He is admitted for further management and started on phenytoin. Which of the following is the most appropriate management for this patient?


A. Focal radiation to brain mass
B. Whole brain radiation
C. Surgical resection of the mass
D. Combination chemotherapy
E. Palliative pain therapy and seizure prophylaxis
The patient has a solitary metastatic brain lesion from non-small cell lung carcinoma (NSCLC). Metastatic brain tumors are the most common type of intracranial brain tumors. The metastasis may be solitary or multiple. Brain metastasis is frequently seen in patients with NSCLC, and it is most commonly located directly beneath the gray-white matter interface. Patients with solitary brain metastasis have a median survival period of one month in absence of any treatment. Glucocorticoids have an important role in palliating symptoms of intra-cranial hypertension; however, they have not been shown any long-term survival benefit.

(Choice C) Surgical resection is the preferred management of solitary brain metastasis. The role of surgery in patients with brain metastases is to provide immediate relief from the mass effect of the tumor, to establish a histologic diagnosis, and to improve local control of the tumor. Whole brain radiotherapy is given after surgical resection to destroy microscopic residual disease and to provide a better quality of life.

(Choices A and B) Radiotherapy for the treatment of brain metastases is meant to improve neurologic deficits caused by the tumor deposits. Studies have shown that survival rates following whole brain radiotherapy are not as good as those following surgical resection for solitary brain metastasis. Focal radiotherapy or brachytherapy is generally used in patients with unresectable brain metastasis and for those who cannot undergo whole brain irradiation.

(Choice D) Chemotherapy might be used in case of brain metastasis from chemosensitive tumors, like small cell lung cancer, lymphoma, or choriocarcinoma; however, it has not shown to be beneficial in the management of solitary brain metastasis from NSCLC.

Educational Objective:
Surgical resection followed by whole brain radiation is the standard of practice in the management of solitary brain metastasis with stable extracranial disease. Multiple brain metastases are best treated with palliative whole brain radiation.

*Extremely high yield question for USMLE!!!
A 67-year-old woman comes to her physician because she is feeling tired, all the time. She thinks that it is due to multiple surgeries she had over the past several years. She had two caesarian sections at the age of 22 and 26. She also had a thyroid surgery for Graves disease, 30 years ago. 12 years ago she was diagnosed with colon cancer and had undergone left hemicolectomy. She denies smoking or alcohol use. Her vitals are: Temperature: 36.7C(98.2F); BP: 138/86 mm Hg; PR: 77/min; RR: 12/min. She looks markedly pale and has weakness in all four extremities. There is some sensory loss in lower limbs. The test for occult blood was negative. Labs came back as:

Sodium 144 mEq/L
Potassium 4.2 mEq/L
Bicarbonate 24mEq/L
Blood urea nitrogen 18mg/dL
Creatinine 1.0 mg/dL
Glucose 82 mg/dL

WBC 8,600cmm
Hemoglobin 7.9
Hematocrit 25%,
Platelets 176,000cmm

The physician decides to further investigate anemia and order RBC indices and peripheral blood smear. The results are

MCV 120 fL
MCH 36 pg
MCHC 28%
Reticulocyte count 0.4%

Peripheral smear showed anisocytosis, poikilocytosis, 4+ macrocytes, polychromatophilia and basophilic stippling.

A whole new bunch of tests are ordered and the following report is seen on the computer.

Vitamin B12 106 pg/mL (N=210-911pg/mL)
Serum Folate 16.4 ng/mL (N=2.8-17.8 ng/mL)
Serum Bilirubin 1.8 mg/dL
Serum LDH 2500 U/L

Gastric analysis demonstrated an absence of hydrochloric acid. What is the most probable cause of her anemia?


A. Folate deficiency
B. Dietary B12 deficiency
C. Pernicious anemia
D. Hemicolectomy
E. Malabsorption syndrome
This is a complicated patient whose anemia was thoroughly investigated. The cause of megaloblastic anemia was found to be Pernicious anemia (PA). PA is the most common megaloblastic anemia and is the prototype of the group. In PA, Vitamin B12 deficiency is caused by reduced Intrinsic factor (IF) secondary to gastric atrophy. In majority of the cases antibodies to parietal cells have been reported. Vitamin B12 deficiency leads to megaloblastic anemia, which results in elevated MCV, elevated MCH, and normal MCHC. The typical findings on the peripheral blood smear are hypersegmented neutrophils, macro ovalocyte RBCs, anisocytosis, poikilocytosis, and basophilic stippling. The diagnosis of PA is confirmed by achlorhydria, decreased serum vitamin B12, positive IF antibodies, and extremely elevated LDH. Elevated bilirubin and serum iron are also seen but they are not specific.

The patient has normal folate level and also some neurological signs. This rules out folic acid deficiency.

Dietary vitamin B12 deficiency does not causes achlorhydria and elevated LDH.

Vitamin B12 is absorbed in ileum so left hemicolectomy has nothing to do with Vitamin B12 deficiency.

The patient has no signs and symptoms to suggest malabsorption as the cause of Vitamin B12 deficiency.

Educational Objective:
Know the typical lab findings of pernicious anemia and the way to differentiate it from other forms of Vitamin B12 deficiency.
A 67-year-old, veteran of Vietnam War, comes for his annual examination at a VA Hospital. He does not have any complaints, other than getting tired very quickly. Physical examination is unremarkable, other than pallor and enlarged spleen. CBC is ordered and reveals

WBC 20,100cmm with more than 65% lymphocytes
Hemoglobin 9.9
Hematocrit 34%,
Platelets 140,000mm3

The lymphocytes have fine irregular cytoplasmic projections. On cytochemical tests there is a strong acid phosphatase reaction, which is not inhibited by tartaric acid. What is the most probable diagnosis?


A. Lymphoblastic lymphoma
B. Hairy cell leukemia
C. Chronic lymphocytic leukemia
D. Chronic myeloid leukemia
E. Hodgkin's disease
Hairy cell leukemia is a type of B-Lymphocytic derived chronic leukemia. It is so named because of fine hair like irregular projections that are characteristics of lymphocytes in this disease. The bone marrow may become fibrotic; therefore, bone marrow aspirates are frequently unsuccessful (a dry tap). The cytochemical feature of hairy cell leukemia includes a tartrate resistant acid phosphatase (TRAP) stain. The CD11c marker is relatively specific for hairy cell leukemia. The clinical course of hairy cell leukemia is more benign than many forms of leukemia.

Lymphoblastic Lymphoma has a number of blast cells in its blood picture.

CLL is the most common type of leukemia in Western Countries but presence of hairy cells and TRAP + cells make the diagnosis as Hairy Cell leukemia.

CML is one of the myeloproliferative syndromes and the whole picture consists of leukocytosis with basophilia and marked splenomegaly.

Hodgkin's disease requires the Reed-Sternberg giant cells i.e. Owl's eye cells for diagnosis.

Educational Objective:
Recognize the importance of tartrate resistant acid phosphatase stain in the diagnostic of hairy cell leukemia.
A 68-year-old man is brought to the ER by hospice nurse because he had a seizures 2 hours ago. The man is unconscious and does not respond to painful stimulus. His vitals are, Temperature: 37.9C(100.2F); RR: 23/min; PR: 96/min; BP: 140/85mm of Hg. The hospice staff has secured his airway and he is on 2L oxygen and his pulse oximetry is 91%. He is in hospice for terminal care for stage IV esophageal carcinoma. A rectal exam reveals hard, irregular surface of an enlarged prostate. His recent prostate specific antigen level is 40. He also had a part of finger removed for skin cancer 1.5 years ago. An MRI is ordered and it shows an intracranial lesion with bleeding inside, consistent with metastasis. What is the most probable diagnosis?


A. Metastatic prostatic carcinoma
B. Glioblastoma multiforme
C. Metastatic melanoma
D. Metastatic squamous cell carcinoma of the skin
E. Metastatic esophageal carcinoma
F. Metastatic basal cell carcinoma
The patient has history of surgery for a skin cancer, which was most probably malignant melanoma. Malignant melanoma is called "the fascinating disease". It can go to most unimaginable places and can even lay dormant for 15-25 years, and come back in a big way even if the primary tumor was resected. This is the case with this patient. His primary tumor was removed and now he is having explosion of metastasis in brain. Malignant melanoma has spread in many cases to all sorts of places like muscle of heart or in bile duct. Malignant melanomas are notorious to cause to bleed in side the mass in brain.

There are four tumors in the body that are not reported (almost never metastasize) to have metastasized to brain
1. Non-melanomatous skin cancer
2. Oropharyngeal cancer
3. Esophageal carcinoma
4. Prostate cancer

The patient is having esophageal cancer and may have prostate cancer but they are rarely found to be going in brain.

Metastatic brain tumors are more common than primary brain tumors.

Educational Objective:
Melanoma should be on the top of the list if a metastatic mass bleeds inside brain.
A 72-year-old Hispanic man comes to the clinic because he is having a mild headache and lethargy for the past several days. He complains of coughing for the past 12 years, although the cough is bothering him more lately. The cough is mucoid, though he has noticed blood in it once in a while. He has been smoking 1 pack/day for 29 years. His Temperature: 37C (98.6F); BP: 120/84 mm Hg; PR: 78/min; RR: 24/min. On examination, adventitious sounds are heard in all lobes and scant basilar crackles in the lungs. Laboratory studies show:

WBC 7,600mm3
Hemoglobin 13.6
Hematocrit 40%,
Platelets 214,000mm3
Sodium 131 mEq/L
Potassium 3.6 mEq/L
Bicarbonate 18 mEq/L
Blood urea nitrogen 16 mg/dL
Creatinine 0.6 mg/dL
Glucose 95 mg/dL
Serum osmolality 260 mOsm/kg (normal 280)
Urine osmolality 110 mmol/L

A chest x-ray shows a 2-centimeter left upper lobe mass and mediastinal adenopathy. Your diagnosis is?


A. Chronic obstructive pulmonary disease
B. Squamous cell carcinoma
C. Large cell carcinoma
D. Small cell carcinoma
E. Adenocarcinoma
This patient has a classic syndrome of inappropriate antidiuretic hormone secretion (SIADH). He has hyponatremia, elevated urine osmolality in the presence of decreased serum osmolality. The clinical symptoms are often vague, until more serious seizures or coma present. SIADH is seen most commonly with small (Oat) cell carcinoma of lung (Choice D). Small cell lung carcinoma is distinctive from other kinds of lung cancer (metastases are already present at the time of discovery) and accounts for approximately 110,000 cancer diagnoses annually. Smoking is by far the major risk factor for small cell cancer.

SIADH is not usually associated with COPD (Choice A), or other forms of lung cancers (Choices B, C, E). It is important to distinguish Small Cell Carcinoma because the treatment is very different. Small cell cancers are primarily treated with Chemotherapy. Surgery does not play a significant role in this type of lung cancer.

Educational Objective:
Syndrome of inappropriate antidiuretic hormone secretion is often seen with small cell carcinoma of the lung.
A 72-year-old Hispanic man comes to the clinic because he is having a mild headache and lethargy for the past several days. He complains of coughing for the past 12 years, although the cough is bothering him more lately. The cough is mucoid, though he has noticed blood in it once in a while. He has been smoking 1 pack/day for 29 years. His Temperature: 37C (98.6F); BP: 120/84 mm Hg; PR: 78/min; RR: 24/min. On examination, adventitious sounds are heard in all lobes and scant basilar crackles in the lungs. Laboratory studies show:

WBC 7,600mm3
Hemoglobin 13.6
Hematocrit 40%,
Platelets 214,000mm3
Sodium 131 mEq/L
Potassium 3.6 mEq/L
Bicarbonate 18 mEq/L
Blood urea nitrogen 16 mg/dL
Creatinine 0.6 mg/dL
Glucose 95 mg/dL
Serum osmolality 260 mOsm/kg (normal 280)
Urine osmolality 110 mmol/L

A chest x-ray shows a 2-centimeter left upper lobe mass and mediastinal adenopathy. Your diagnosis is?


A. Chronic obstructive pulmonary disease
B. Squamous cell carcinoma
C. Large cell carcinoma
D. Small cell carcinoma
E. Adenocarcinoma
This patient has a classic syndrome of inappropriate antidiuretic hormone secretion (SIADH). He has hyponatremia, elevated urine osmolality in the presence of decreased serum osmolality. The clinical symptoms are often vague, until more serious seizures or coma present. SIADH is seen most commonly with small (Oat) cell carcinoma of lung (Choice D). Small cell lung carcinoma is distinctive from other kinds of lung cancer (metastases are already present at the time of discovery) and accounts for approximately 110,000 cancer diagnoses annually. Smoking is by far the major risk factor for small cell cancer.

SIADH is not usually associated with COPD (Choice A), or other forms of lung cancers (Choices B, C, E). It is important to distinguish Small Cell Carcinoma because the treatment is very different. Small cell cancers are primarily treated with Chemotherapy. Surgery does not play a significant role in this type of lung cancer.

Educational Objective:
Syndrome of inappropriate antidiuretic hormone secretion is often seen with small cell carcinoma of the lung.
A 72-year-old Hispanic man comes to the clinic because he is having a mild headache, and lethargy for the past several days. He complains of coughing for the past 12 years, although the cough is bothering him more lately. The cough is mucoid, though he has noticed blood in it once in a while. He has smoked 1pack/day for 29 years. His vitals are, Temperature: 37.0C (98.6F); BP: 120/84 mm Hg; PR: 78/min; RR: 24/min. On examination, adventitious sounds are heard in all lobes and scant basilar crackles in the lungs. Laboratory studies show:

WBC 7,600mm3
Hemoglobin 13.6
Hematocrit 40%,
Platelets 214,000mm3
Sodium 130 mEq/L
Potassium 3.6 mEq/L
Bicarbonate 18 mEq/L
Blood urea nitrogen 16 mg/dL
Creatinine 0.6 mg/dL
Glucose 95 mg/dL
Serum osmolality 260 mOsm/kg (normal 280)
Urine osmolality 110 mmol/L

A chest x-ray shows a 2-centimeter left upper lobe mass and mediastinal adenopathy. What will be the next best step in management of this patient?


A. Prescribe demeclocycline
B. Restrict the patient water intake
C. Start an infusion of hypertonic saline
D. Start furosemide
This patient has classic SIADH from lung carcinoma. Treatment usually consists of restriction of water intake to daily insensible losses (Choice C). If the serum sodium is below 120 or if the patient is seizing, emergency treatment is administration of 3% sodium chloride solution to raise the serum sodium to 125 (Choice D). If the patient has evidence of fluid overloading, a history of CHF, or is resistant to treatment, loop diuretics may be added as well (Choice E). Chronic treatment may involve administration of lithium or demeclocycline, which inhibit ADH action (Choice B).

Educational Objective:
Water restriction is the first step in managing hyponatremia in patients with syndrome of inappropriate antidiuretic hormone secretion.
A 74-year-old man is under treatment for terminal colon cancer. He was diagnosed with the disease 8 months ago and has metastasis in his liver. He has gone through chemotherapy with 5-Flourouracil + Oxaloplatin. During the past 4 months, his condition has gotten really worse. He has lost 50lb(27.6kg) and appears totally wasted. He does not feel like eating anything. His abdomen is distended with positive fluid thrill, and his liver has several palpable nodules. He has refused any surgery for his severe constipation, although he is willing to take any pill that will make him comfortable. The doctor wants to add a medicine for his anorexia. What is the pharmacological agent of choice for this patient?


A. Megestrol acetate
B. Dexamethasone
C. Dimenhydrinate
D. Ondansetron
E. Metoclopramide
Anorexia is one of the common problems in terminally ill patients. This provides a significant challenge to health-care provider. The rule-of- thumb is that, it is always better to have oral nutrition than parenteral route in palliative care. So, all efforts need to be made to encourage the patient to eat adequate meals. Non-pharmacological therapy is always the first step and that includes giving patient what he likes best, i.e. presenting food attractively, and giving him small and frequent meals. After this comes the drugs. The drug of first choice in cancer-associated anorexia is Megestrol acetate. It is a synthetic progestin with progestational effects similar to those of progesterone and is currently used as an appetite stimulant in advanced malignancy. Prednisone can also be added with it.

Dexamethasone is not used for malignancy-associated anorexia.

Dimenhydrinate, ondansetron, and metoclopramide are given for chemotherapy and cancer-associated nausea and vomiting.

Educational Objective:
The drug of first choice for cancer-associated anorexia is Megestrol acetate.
A 74-year-old man is under treatment for terminal colon cancer. He was diagnosed with the disease 8 months ago and has metastasis in his liver. He has gone through chemotherapy with 5-Flourouracil + Oxaloplatin. During the past 4 months, his condition has gotten really worse. He has lost 50lb(27.6kg) and appears totally wasted. He does not feel like eating anything. His abdomen is distended with positive fluid thrill, and his liver has several palpable nodules. He has refused any surgery for his severe constipation, although he is willing to take any pill that will make him comfortable. The doctor wants to add a medicine for his anorexia. What is the pharmacological agent of choice for this patient?


A. Megestrol acetate
B. Dexamethasone
C. Dimenhydrinate
D. Ondansetron
E. Metoclopramide
Anorexia is one of the common problems in terminally ill patients. This provides a significant challenge to health-care provider. The rule-of- thumb is that, it is always better to have oral nutrition than parenteral route in palliative care. So, all efforts need to be made to encourage the patient to eat adequate meals. Non-pharmacological therapy is always the first step and that includes giving patient what he likes best, i.e. presenting food attractively, and giving him small and frequent meals. After this comes the drugs. The drug of first choice in cancer-associated anorexia is Megestrol acetate. It is a synthetic progestin with progestational effects similar to those of progesterone and is currently used as an appetite stimulant in advanced malignancy. Prednisone can also be added with it.

Dexamethasone is not used for malignancy-associated anorexia.

Dimenhydrinate, ondansetron, and metoclopramide are given for chemotherapy and cancer-associated nausea and vomiting.

Educational Objective:
The drug of first choice for cancer-associated anorexia is Megestrol acetate.
A 75-year-old Caucasian male comes to your office for his routine medical check up. He complains of fatigue since last one month. His previous medical history is significant for calcified aortic valves and hypertension. His vitals are stable except a BP of 150/90 mm Hg. On examination, he has pallor and 4+ejection systolic murmur in the aortic area. Lab reports showed:

Hb 9g/dL
MCV 75 fl
Reticulocyte count Increased
Serum LDH Increased
Haptoglobulin Decreased
Peripheral smear Fragmented RBC

Which of the following is the most likely cause of anemia in this patient?


A. Bleeding peptic ulcer.
B. Diverticulosis.
C. Macro vascular traumatic hemolysis.
D. Warm antibody hemolysis.
E. G6PD deficiency anemia.
Most likely diagnosis in this patient is intravascular hemolytic anemia, as suggested by elevated reticulocyte count, increased LDH levels, and decreased haptoglobin. Microcytic anemia is due to iron loss from chronic hemolysis. With the given history, the most likely cause of hemolysis is mechanical trauma from calcified aortic valves, also called macro vascular traumatic hemolysis (Choice C). This is usually seen with artificial heart valves or markedly calcified valves. Hamlet cells or fragmented RBCs on peripheral smear is characteristic of traumatic hemolysis.

Choice A and B: Bleeding peptic ulcer and diverticulosis are common causes of microcytic anemia in elderly but they would not explain the signs of hemolysis in this patient.

Choice D: Warm antibody type autoimmune hemolytic anemia is most commonly seen in women with some underlying disease affecting immune system, like lymphoid neoplasm, collagen vascular disease, and congenital immunodeficiency diseases. It is characterized by positive Coomb’s test. However, the clinical presentation of the patient in this vignette is more suggestive of traumatic hemolytic anemia.

Choice E: G6PD deficiency generally causes acute hemolysis precipitated by drugs, infection or metabolic abnormality, rather than chronic hemolysis and would have manifested at an earlier age.

Educational objective:
Consider macro vascular hemolysis as a cause of microcytic anemia in patients with artificial heart valves or severely calcified aortic valves.
A father brings his 7-year-old boy to the office because “his eyes are getting yellow”. The boy has no other complaints. However, he is feeling tired and does not feel like going to the next Steelers game. There is a history of diabetes and lung cancer in the family. His vitals are stable, but he has pallor, jaundice, and splenomegaly. His labs revel:

WBC 10,000cmm
Hemoglobin 10 gm/dL
Hematocrit 28%
RBC count 3.5 million
Platelets 240,000cmm
Reticulocyte count 4%

RBC indices were as follows:

MCV 90 fL
MCHC 38%
MCH 28 pg

Peripheral smear revealed anisocytosis, some spherocytosis, and polychromatophilia. Coomb’s test was negative.

LFTs showed:

Albumin 5.2 g/dL
AST 47 U/L
ALT 52 U/L
Alkaline phosphatase 120 U/L
Total bilirubin 3.0 mg/dL
Direct bilirubin 0.2 mg/dL

What is the next step?


A. Bone marrow biopsy
B. Osmatic fragility test
C. Sugar water test
D. Hemoglobin electrophoresis
E. Serum B12 level
In a patient with increased reticulocyte count, increased bilirubin, negative Coombs test and spherocytes on peripheral blood films, the diagnosis of Hereditary Spherocytosis (HS) is top on the list. HS is an autosomal dominant trait and is the most common hereditary hemolytic anemia in white population. Anemia may manifest itself from early infancy to later age. There is congenital RBC membrane defect in HS resulting in extravascular hemolysis occurring only in the presence of spleen. The range of symptoms varies widely from asymptomatic to severe disease. The patient may be severely anemic with jaundice and splenomegaly on presentation. Peripheral blood smear demonstrate spherocytes. MCV is normal or slightly decreased while MCH is normal but the MCHC is generally greater than 36%. The osmotic fragility test is the next best test for the diagnosis of the patient.

Bone marrow biopsy is not required. The disease can be easily diagnosed by other hematological parameters.

Sugar water test is done for the diagnosis of Paroxysmal Nocturnal Hemoglobinuria (PNH).

Hemoglobin electrophoresis is not useful as HS is not one of the hemoglobinopathies like sickle cell disease or thalassemia.

Serum B12 levels are done for macrocytic anemias, which are characterized by elevated MCV (>100).

Educational Objective:
Know about the classical picture of chronic hemolytic states and hereditary spherocytosis.
An 80-year-old female is brought to your office, by her son, because of severe fatigue. She lives alone and is suffering from severe degenerative joint disease, which puts her in a house arrest-type state. Her son usually helps with getting grocery. Her only other medical problem is hypertension. She takes hydrochlorothiazide and acetaminophen. Her vitals are stable. On examination, she has pallor, and evidence of severe degenerative joint disease. Which of the following is the most likely cause of pallor in this patient?


A. Vitamin D deficiency
B. Vitamin C deficiency
C. Iron deficiency
D. Folate deficiency
E. Chronic hemolysis
F. Anemia of chronic disease
G. Vitamin B12 deficiency
H. Copper deficiency
The following points are to be noted in this clinical vignette. This is a geriatric patient with pallor and fatigue. Her medical history is significant for severe degenerative joint disease and HTN. With the given information, it is obvious that her fatigue is most likely due to anemia. Iron deficiency is the most common cause of anemia in elderly patients; the iron deficiency is most likely secondary to a nutritional deficiency, in this case. Other very common cause of IDA in elderly patients is chronic blood loss; however this patient has no complaints suggestive of chronic blood loss (such as gastritis, PU, diverticulitis, etc.).

Choice F: Anemia of chronic disease (ACD) is seen in patients with chronic illnesses. This is usually seen with infectious, inflammatory, or neoplastic diseases. Also, more recently, this has been noted in patients with severe trauma, heart disease, and diabetes mellitus. This patient has chronic degenerative joint disease, but it should be noted that inflammatory joint disease and not degenerative joint disease cause ACD. The pathophysiology involves defective utilization of the iron by the RBC precursors, secondary to inflammatory mediators.

Choice A: Vitamin D deficiency can occur in a patient who does not go out of the house, or is in a house arrest state; however, this would cause osteomalacia, and not anemia.

Choice B: Vitamin C deficiency is also common among elderly patients who consume a diet poor in vitamin C, and it would cause scurvy. This patient has no clinical features suggestive of scurvy.

Choices D and G: A diet poor in green leafy vegetables and meat can cause folate or vitamin B-12 deficiency, respectively, which could lead to megaloblastic anemia. No information on the patient's diet is given in this vignette. It is also important to note that IDA is the most common anemia in the elderly.

Choices E and H: This patient does not have any clinical features suggestive of copper deficiency or chronic hemolysis. Chronic hemolysis and copper deficiency can also cause IDA.

Educational Objective:
Iron deficiency is the most common cause of anemia in elderly patients. Inflammatory joint disease, and not degenerative joint disease, causes the anemia of chronic disease.

*Extremely high yield question for USMLE!!!
An 80-year-old female is brought to your office, by her son, because of severe fatigue. She lives alone and is suffering from severe degenerative joint disease, which puts her in a house arrest-type state. Her son usually helps with getting grocery. Her only other medical problem is hypertension. She takes hydrochlorothiazide and acetaminophen. Her vitals are stable. On examination, she has pallor, and evidence of severe degenerative joint disease. Which of the following is the most likely cause of pallor in this patient?

A. Vitamin D deficiency
B. Vitamin C deficiency
C. Iron deficiency
D. Folate deficiency
E. Chronic hemolysis
F. Anemia of chronic disease
G. Vitamin B12 deficiency
H. Copper deficiency
The following points are to be noted in this clinical vignette. This is a geriatric patient with pallor and fatigue. Her medical history is significant for severe degenerative joint disease and HTN. With the given information, it is obvious that her fatigue is most likely due to anemia. Iron deficiency is the most common cause of anemia in elderly patients; the iron deficiency is most likely secondary to a nutritional deficiency, in this case. Other very common cause of IDA in elderly patients is chronic blood loss; however this patient has no complaints suggestive of chronic blood loss (such as gastritis, PU, diverticulitis, etc.).

Choice F: Anemia of chronic disease (ACD) is seen in patients with chronic illnesses. This is usually seen with infectious, inflammatory, or neoplastic diseases. Also, more recently, this has been noted in patients with severe trauma, heart disease, and diabetes mellitus. This patient has chronic degenerative joint disease, but it should be noted that inflammatory joint disease and not degenerative joint disease cause ACD. The pathophysiology involves defective utilization of the iron by the RBC precursors, secondary to inflammatory mediators.

Choice A: Vitamin D deficiency can occur in a patient who does not go out of the house, or is in a house arrest state; however, this would cause osteomalacia, and not anemia.

Choice B: Vitamin C deficiency is also common among elderly patients who consume a diet poor in vitamin C, and it would cause scurvy. This patient has no clinical features suggestive of scurvy.

Choices D and G: A diet poor in green leafy vegetables and meat can cause folate or vitamin B-12 deficiency, respectively, which could lead to megaloblastic anemia. No information on the patient's diet is given in this vignette. It is also important to note that IDA is the most common anemia in the elderly.

Choices E and H: This patient does not have any clinical features suggestive of copper deficiency or chronic hemolysis. Chronic hemolysis and copper deficiency can also cause IDA.

Educational Objective:
Iron deficiency is the most common cause of anemia in elderly patients. Inflammatory joint disease, and not degenerative joint disease, causes the anemia of chronic disease.

*Extremely high yield question for USMLE!!!
An 80-year-old man has advanced prostate cancer with bony metastasis. Now, he presents with a complaint of severe back pain that has been progressively worsening for the past two weeks. His pain is so severe that it restricts him from playing golf. Eight months ago, he underwent orchiectomy, after which he was free of bone pain until now. Physical examination reveals tenderness at two sites in the lumbar region. Radionuclide bone scan shows increased uptake in these areas. Which of the following is the most appropriate next step in the management of this patient?


A. Cervical cordotomy
B. Etidronate disodium therapy
C. Flutamide therapy
D. Hypophysectomy
E. Radiation therapy
Radiation therapy would be most appropriate for management of progressive pain in a patient with prostate cancer metastatic to bone after androgen ablation (orchiectomy). Focal external beam therapy will be an excellent choice in this patient, as metastasis is localized to few sites.

(Choice A) Cervical cordotomy is not effective for controlling axial pain or pain in the upper extremities.

(Choice B) Etidronate disodium and other bisphosphonates may be useful in patients with bony metastasis as it reduces bone resorption. It is useful in controlling chronic pain from bony metastasis, but, its onset of action is not as rapid as radiation therapy in alleviating acute bone pain.

(Choice C) Flutamide is a non-steroidal anti-androgen and it provides androgen blockade by competitively binding to dihydrotestosterone receptors. This drug, when combined with a LHRH agonist, prolongs the survival of patients with limited disease. However, there were no demonstrable benefits in patients who underwent orchiectomy, as in this patient. Estramustine, a combination of estrogen and nitrogen mustard, has a 40% response rate in castrated men, but is not well tolerated in individuals with cardiovascular or hematopoietic disorders.

(Choice D) Hypophysectomy is not effective in delaying progression or controlling pain.

Educational Objective:
Radiation therapy is useful in managing bone pain in patients with prostate cancer who have undergone orchiectomy.
An 81-year-old man comes from a nursing home, suffering from stage IV prostate cancer. He was diagnosed with the disease 7 months ago, and cancer was found to reach the spine. He had palliative radiotherapy of his spine and is taking leuprolide. He now complains of very severe pain in his back and thighs. The nursing home staff gave him some ibuprofen, but he still is restless and very irritable. He has never received any narcotics. What is best next step in the management of pain in this patient?


A. Give high dose NSAIDs
B. Transdermal fentanyl patch
C. Start with short acting morphine
D. Long acting opiates
E. Repeat radiotherapy
The patient is in terminal stages of his cancer. The three rules for prescribing pain medications to such patients are:

1. Try non-narcotic measures first UNLESS your clinical judgment is sure that the patient is in severe pain.
2. Don’t be afraid of narcotic analgesic
3. Prescribe adequate amount of medication

Most of the surveys indicate that more than 50% of cancer patients die with significant pain. This is due to fear among doctors of using narcotics because of making patient addicted, or doing "too much" or even at times just to save themselves. Now in this case patient is in severe pain so he should be given dose of short acting morphine every 4-6 hours to attain complete pain control. Once the dose is established, then switch to long acting narcotics and keep the option of short acting morphine for break through pain.

Repeating radiotherapy may slightly help him in the long run if determined so by the oncologist but that is not the best step to take at this point of time.

Transdermal fentanyl patch is an option too but it takes at least 8-12 hours to give the full affect. So patient should receive short acting morphine.

High dose of NSAIDs in setting of severe cancer pain is not very useful. Terminal cancer pain needs narcotics not NSAIDs.

Educational Objective:
Cancer patient should not suffer from pain; know how to manage these patients.
Elizabeth, a 13-year-old girl, comes with complaints of new onset seizures. She has morning headaches that goes away after vomiting. She also has history of colonic polyps, for which she is undergoing work-up. Her family history is significant for her mother having problems with polyps. (She underwent proctocolectomy.) Initial non-contrast T1-weighted MRI shows a large, heterogeneous, hypointense mass in the white matter of the right temporal lobe and extending into the parietal and occipital lobes. The lower signal area within the mass suggests tissue necrosis. What is the most likely diagnosis in this patient?


A. Multiple hamartoma syndrome
B. Gardner's Syndrome
C. Turcot's Syndrome
D. Peutz-Jeghers syndrome
E. Cronkhite-Canada syndrome
The patient has colonic polyps and a brain glioma which most probably is Glioblastoma multiforme. Turcot's syndrome refers to an association between brain tumors (primarily medulloblastomas and gliomas) and FAP (Familial Adenomatous polyposis) or HNPCC (Hereditary nonpolyposis colorectal cancer). The majority of FAP-associated brain tumors are medulloblastomas, but gliomas have also been described. Patients with HNPCC are prone to high-grade gliomas. It is autosomal recessive and mainly occurs in teens.

Gardner's syndrome is autosomal dominant where colonic polyps are seen with prominent extraintestinal lesions. These include desmoid tumors, sebaceous or epidermoid cysts, lipomas, osteomas (especially of the mandible), supernumerary teeth, gastric polyps, and juvenile nasopharyngeal angiofibromas.

Peutz-Jeghers syndrome is an autosomal dominant inherited disorder characterized by intestinal hamartomatous polyps in association with mucocutaneous melanocytic macules.

Multiple hamartoma syndrome (Cowden syndrome) is association of GI-tract hamartomas with breast Ca, thyroid Ca, and nodular gingival hyperplasia.

Cronkhite-Canada syndrome is association of juvenile-type polyps and ectodermal abnormalities like alopecia, hyperpigmentation, and nail loss (onycholysis).

Educational Objective:
Turcot's syndrome refers to an association between brain tumors (primarily medulloblastomas and gliomas) and FAP (Familial Adenomatous polyposis) or HNPCC (Hereditary nonpolyposis colorectal cancer).
Elizabeth, a 13-year-old girl, comes with complaints of new onset seizures. She has morning headaches that goes away after vomiting. She also has history of colonic polyps, for which she is undergoing work-up. Her family history is significant for her mother having problems with polyps. (She underwent proctocolectomy.) Initial non-contrast T1-weighted MRI shows a large, heterogeneous, hypointense mass in the white matter of the right temporal lobe and extending into the parietal and occipital lobes. The lower signal area within the mass suggests tissue necrosis. What is the most likely diagnosis in this patient?


A. Multiple hamartoma syndrome
B. Gardner's Syndrome
C. Turcot's Syndrome
D. Peutz-Jeghers syndrome
E. Cronkhite-Canada syndrome
The patient has colonic polyps and a brain glioma which most probably is Glioblastoma multiforme. Turcot's syndrome refers to an association between brain tumors (primarily medulloblastomas and gliomas) and FAP (Familial Adenomatous polyposis) or HNPCC (Hereditary nonpolyposis colorectal cancer). The majority of FAP-associated brain tumors are medulloblastomas, but gliomas have also been described. Patients with HNPCC are prone to high-grade gliomas. It is autosomal recessive and mainly occurs in teens.

Gardner's syndrome is autosomal dominant where colonic polyps are seen with prominent extraintestinal lesions. These include desmoid tumors, sebaceous or epidermoid cysts, lipomas, osteomas (especially of the mandible), supernumerary teeth, gastric polyps, and juvenile nasopharyngeal angiofibromas.

Peutz-Jeghers syndrome is an autosomal dominant inherited disorder characterized by intestinal hamartomatous polyps in association with mucocutaneous melanocytic macules.

Multiple hamartoma syndrome (Cowden syndrome) is association of GI-tract hamartomas with breast Ca, thyroid Ca, and nodular gingival hyperplasia.

Cronkhite-Canada syndrome is association of juvenile-type polyps and ectodermal abnormalities like alopecia, hyperpigmentation, and nail loss (onycholysis).

Educational Objective:
Turcot's syndrome refers to an association between brain tumors (primarily medulloblastomas and gliomas) and FAP (Familial Adenomatous polyposis) or HNPCC (Hereditary nonpolyposis colorectal cancer).
Her son brings a 50-year-old female to the emergency room because she is confused and sweating a lot. She does not have any medical problems, except hypertension for which she takes hydrochlorothiazide. She is not taking any other medication. He tells you that his mother complained of profound weakness, and inability to stand, because of lightheadedness 3 hours ago. On examination, the patient is drowsy, confused, and appears sick. Her Temperature: 36.8C(98.2 F); BP: 153/83mm Hg; PR: 128/min. Her clothes are damp from perspiration. No other abnormalities are noted on physical examination. Initial laboratory studies show:

Sodium: 144 mEq/L
Potassium: 3.6 mEq/L
Bicarbonate: 26 mEq/L
Blood urea nitrogen: 12 mg/dL
Creatinine: 0.6 mg/dL

You order a few more tests and on the basis of which you diagnose insulinoma. What set of following results leads to this diagnosis?

Serum Glucose, Insulin, C-Protein



A. Increased,Decreased,Increased
B. Increased,Increased,Increased
C. Decreased,Increased,Increased
D. Decreased,Increased,Decreased
E. Normal,Normal,Normal
The patient has classical symptoms of hypoglycemia that includes weakness, dizziness, diaphoresis, nausea, tachycardia, pallor, headache, visual disturbances, lethargy, agitation, confusion, and inappropriate behavior (autonomic activation, and neuroglycopenic symptoms). Insulinoma presents as hypoglycemia. They are the most common pancreatic endocrine tumor (Approximately 50% of islet cell tumors are Insulinomas). An increase in serum levels of both insulin and C-peptide is diagnostic of insulinoma. (Choice C)
The symptoms include:
1. About 85% of patients present with diplopia, blurred vision, palpitations, or weakness.
2. Other symptoms include confusion, abnormal behavior, unconsciousness, or amnesia.
3. About 12% of patients have grand mal seizures.
4. Adrenergic symptoms (hypoglycemia causes adrenalin release) include weakness, sweating, tachycardia, palpitations, and hunger.
5. Hypoglycemia usually occurs several hours after a meal.

The Whipple triad characterizes Insulinomas clinically.
1. Episodic hypoglycemia
2. Central nervous system (CNS) dysfunction temporally related to hypoglycemia (confusion, anxiety, stupor, convulsions, coma)
3. Dramatic reversal of CNS abnormalities by glucose administration

(Choice A) is seen in Insulin dependant Diabetes Mellitus. (Choice D) is a case of factitious hypoglycemia while (Choice B) is the picture seen in severe insulin resistance. (Choice E) is the normal state, which is not consistent with insulinoma.

Educational Objective:
An increase in serum levels of both insulin and C-peptide and hypoglycemia is diagnostic of insulinoma.
Her son brings a 50-year-old female to the emergency room because she is confused and sweating a lot. She does not have any medical problems, except hypertension for which she takes hydrochlorothiazide. She is not taking any other medication. He tells you that his mother complained of profound weakness, and inability to stand, because of lightheadedness 3 hours ago. On examination, the patient is drowsy, confused, and appears sick. Her Temperature: 36.8C(98.2 F); BP: 153/83mm Hg; PR: 128/min. Her clothes are damp from perspiration. No other abnormalities are noted on physical examination. Initial laboratory studies show:

Sodium: 144 mEq/L
Potassium: 3.6 mEq/L
Bicarbonate: 26 mEq/L
Blood urea nitrogen: 12 mg/dL
Creatinine: 0.6 mg/dL

You order a few more tests and on the basis of which you diagnose insulinoma. What set of following results leads to this diagnosis?

Serum Glucose, Insulin, C-Protein



A. Increased,Decreased,Increased
B. Increased,Increased,Increased
C. Decreased,Increased,Increased
D. Decreased,Increased,Decreased
E. Normal,Normal,Normal
The patient has classical symptoms of hypoglycemia that includes weakness, dizziness, diaphoresis, nausea, tachycardia, pallor, headache, visual disturbances, lethargy, agitation, confusion, and inappropriate behavior (autonomic activation, and neuroglycopenic symptoms). Insulinoma presents as hypoglycemia. They are the most common pancreatic endocrine tumor (Approximately 50% of islet cell tumors are Insulinomas). An increase in serum levels of both insulin and C-peptide is diagnostic of insulinoma. (Choice C)
The symptoms include:
1. About 85% of patients present with diplopia, blurred vision, palpitations, or weakness.
2. Other symptoms include confusion, abnormal behavior, unconsciousness, or amnesia.
3. About 12% of patients have grand mal seizures.
4. Adrenergic symptoms (hypoglycemia causes adrenalin release) include weakness, sweating, tachycardia, palpitations, and hunger.
5. Hypoglycemia usually occurs several hours after a meal.

The Whipple triad characterizes Insulinomas clinically.
1. Episodic hypoglycemia
2. Central nervous system (CNS) dysfunction temporally related to hypoglycemia (confusion, anxiety, stupor, convulsions, coma)
3. Dramatic reversal of CNS abnormalities by glucose administration

(Choice A) is seen in Insulin dependant Diabetes Mellitus. (Choice D) is a case of factitious hypoglycemia while (Choice B) is the picture seen in severe insulin resistance. (Choice E) is the normal state, which is not consistent with insulinoma.

Educational Objective:
An increase in serum levels of both insulin and C-peptide and hypoglycemia is diagnostic of insulinoma.
Joe is a 2-year-old boy, brought by his mother into the ER because of a high-grade fever. She has tried tylenol (acetaminophen), but the fever is "just not going away". The child has been very irritable for the last 4 days and is crying a lot. He is also pulling his ear and not eating well. The child has been generally well other than occasional sore throat, this season. His vitals are, Temperature: 38.8C(102.2F); BP: 90/60 mm Hg; PR: 119/min; RR: 24/min. Physical examination revealed a well nourished, but irritable child. He had enlarged cervical lymph nodes and splenomegaly. The tympanic membranes were inflammed. A stat CBC is ordered which shows

WBC 81,100mm3
Hemoglobin 8.0
Hematocrit 27%,
Platelets 16,000mm3

The leukocyte distribution on differential smear was

Blast forms 80%
Prolymphocytes 10%
Lymphocytes 10%

The blast cells had one or two nuclei and no Auer rods were seen. Subsequent cytochemical reaction reveals strongly positive periodic acid schiff reaction. What is the most probable diagnosis?


A. Burkitt’s lymphoma
B. Acute myelocytic leukemia
C. Prolymphocytic leukemia
D. Acute lymphoblastic leukemia
E. Myelodysplastic syndrome
The predominant type of leukemia in children from ages 2-10 years is acute lymphoblastic leukemia (ALL). It presents acutely with child feeling ill for just a couple of weeks. About 30-50% of them present with infections. Lymphadenopathy and splenomegaly occur in about half. ALL presents with lymphoblasts in the peripheral smear. The patients have varying degree of anemia, neutropenia, and thrombocytopenia. Presence of more than 25% lymphoblasts in bone marrow and the positive periodic acid schiff reaction (PAS) staining makes the diagnosis of ALL. Good prognostic indications are youth and hyperploidy while poor prognosis is indicated by increasing age.

Lymphoma is the preferred term when the process is confined to a mass lesion with minimal or no blood and marrow involvement. Burkitt’s lymphoma is a diffuse, small, undifferentiated high-grade malignancy and is associated with Epstein-Barr virus. A starry sky appearance of the tumor cells is described with it.

In acute myelocytic leukemias the predominant cells are of myeloid origin. Auer rods are specific for AML.

Prolymphocytic leukemia is a CLL variant with massive splenomegaly, high lymphocytosis with little lymphadenopathy.

Myelodysplastic syndromes (MDS) are clonal stem cell disorders, which may progress to acute leukemias. They are usually seen in the adults and characterized by pancytopenia.

Educational Objective:
Acute lymphoblastic leukemia is predominantly a disease of children and recognize the importance of periodic acid schiff positive staining in diagnosis.
John, a 14-year-old boy, is brought by his mother because she has noticed a change in his voice. He is having problem of frequent nosebleeds for the last 1 month and also feeling his "left nose" to be congested all the time. Pointing towards his left nose he says "its difficult to breathe from here". He is otherwise, doing fine and has participated in the school basketball tournament. There is no history of trauma. He is not taking any medicine and admits to using marijuana, in the absence of her mother. On examination, his nasal septum is intact but there is a visible mass at the back of left nose. A CT scan is ordered that confirms the erosion of the adjacent bone. What is the reason of the nasal bleeds?


A. Cocaine abuse
B. Angiofibroma
C. Bleeding disorder
D. Reactive nasal polyps
E. Chondroma of nasal cartilage
The patient has juvenile angiofibroma (JNA). It is a benign growth of the back of the nose, or upper throat (nasopharynx) that contains many blood vessels. It typically is discovered in adolescent boys and is associated with nasal obstruction, a visible nasal mass and nosebleeds (epistaxis). The tumor is locally invasive and capable of eroding. It can cause big problems because JNAs bleed readily and occur in an area where surgical access is difficult. This tumor should only be touched by a specialist because of the high potential of causing bleeding. Treatment is required if the angiofibroma is enlarging, obstructing the airway, or causing chronic nosebleeds. In some cases, no treatment is necessary. Surgical treatment includes removal of the tumor. Removal is often difficult because the tumor is unencapsulated and may be deeply invasive. Recurrence of the tumor after surgical resection is common.

Cocaine abuse can be a cause of nasal bleeding, but does not present with nasal mass and bone erosion as in this case.

Bleeding disorder should always be kept in mind, whenever a young adult comes with epistaxis, but the nasal mass has to be investigated first.

There is no reason to believe to consider these as reactive nasal polyps. They normally do not cause bony erosions. They are usually associated with chronic infections or allergies. The main complaint with polyp is obstruction rather than a bleeding.

Chondroma of nasal cartilage is very rare and is not seen in young adults.

Educational Objective:
Any adolescent boy who presents with epistaxis and has localized mass with bony erosion on the back of the nose has an angiofibroma until proven otherwise.
Mr. Benson, 65-year-old male is brought to ER by his wife because of very high fever. He has been coughing for the past 2 days. The cough is with greenish sputum, but there is no blood in it. The patient told that he had pneumonia 3 months ago. The patient is diabetic and smokes 1pack/day for 27 years. His vitals are, Temperature: 38.4C(101.2F); BP: 118/74 mm Hg; PR: 98/min; RR: 24/min. On examination, he has cervical lymphadenopathy. Some scattered rales are heard in right lung base. Chest-X ray shows right lower lobe infiltrate. Blood work is ordered and results are:

Hb 13gm/dL
WBC 40,000/cmm
Platelets 190,000/cmm
Peripheral smear
Segmented neutrophils 10%
Bands 4%
Lymphocytes 85%
Monocytes 1%

Some variant lymphocytes, and smudge cells are seen on peripheral smear. What is the most appropriate way to confirm the diagnosis in the condition?


A. Serology for epstein barr virus
B. Bone marrow biopsy
C. Lymph node biopsy
D. Cytogenetic analysis
E. Lung biopsy
The patient has chronic lymhpocytic leukemia (CLL), which is the most common type of leukemia in Western Countries. It is seen mostly in older patients. The patients are often asymptomatic when diagnosed after lymphocytosis is picked up incidentally. When symptomatic it is usually due to lymphadenopathy. The patient may develop hypogammaglobulinemia and become more susceptible to infections. On peripheral blood film it is characterized by lymphocytosis of small mature appearing lymphocytes. Smudge cells are highly characteristic. In general, lymph node biopsy is not routinely required to make the diagnosis of CLL. However, a highly specific lymph node biopsy can be performed to confirm the diagnosis. It may show tumor cells, which are over matured lymphocytes because of hyper condensed nuclear chromatin pattern.

The clinical scenario of the patient does not fit well with Infectious mononucleosis. So there is no need to test serology for the disease.

Bone marrow biopsy is not a useful tool in CLL. They are more useful in myeloid disorders.

Cytogenetic analysis may be useful because some CLL are associated with trisomy 12 but this is not a highly specific test.

The patient is increased risk of lung cancer because of his smoking habits. However, his blood disorder needs to be investigated first.

Educational Objective:
Try to identify chronic lymhpocytic leukemia from clinical picture and hematological findings. Lymph node biopsy not bone marrow biopsy confirms the diagnosis.
Mr. Booze, a 55-year-old male, presented with several week history of fatigue. He denies any other symptoms. His vitals are stable. Examination shows pallor, massive splenomegaly, and mild hepatomegaly. CBC revealed pancytopenia with striking monocytopenia. Peripheral blood smear is shown below. Bone marrow biopsy showed dry tap. What is the most appropriate treatment for his condition?




A. Bone marrow transplantation
B. Cladribine
C. Cyclophosphamide
D. CHOP regimen
E. Chlorambucil and prednisone
The patient has hairy cell leukemia (leukemic reticuloendotheliosis) that is a type of B-Lymphocyte derived chronic leukemia. It is so named because of fine hair like irregular projections that are characteristics of lymphocytes in this disease. The bone marrow may become fibrotic; therefore, bone marrow aspirates are frequently unsuccessful (a dry tap). The cytochemical feature of hairy cell leukemia includes a tartrate resistant acid phosphatase (TRAP) stain. The drug of choice in this disease is a purine analog, Cladribine. Cladribine is toxic to bone marrow. It can cause neurological and kidney damage.

Bone marrow transplant, Cyclophosphamide, 5-Flourauracil and Prednisone have not been found to be very useful in improving the out come of this disease.

Chlorambucil and Prednisone are used for chronic lymphocytic leukemia.

CHOP regimen is used for Non-Hodgkin’s lymphoma.

Educational Objective:
Know the diagnostic features of Hairy Cell Leukemia and that Cladribine is the drug of choice for it.
Mr. Claude, a 35-year-old man, comes to his primary care physician for his gastrointestinal complaints for the last 4 months. Right now he has fever, bloody diarrhea, nausea, and severe abdominal cramps. He has lost 20 lb(9kg) and is also feeling fatigued and anorexic. His vitals are, Temperature: 37.3C(99.1F); BP: 110/74mm of Hg; PR: 98/min; RR: 22/min. On examination, remarkable findings are tenderness in right lower quadrant and skin rash. Digital Rectal examination is positive for occult blood. Blood work returns as:

WBC: 11,600mm3
Hemoglobin: 9.6
Hematocrit: 31%,
Platelets: 214,000mm3

The flexible sigmoidoscopy reveals larger areas of ulceration within the colon. What is the next best step in the management?


A. Refer him for procto-colectomy
B. Tell him to avoid highly seasoned foods and send him home
C. Give him sulfasalazine
D. Perform a biopsy of the colon lesion
E. Give him a bolus of corticosteroids
F. Start 6-mercapto-purine (6-MP)
If a questionable ulcer is visualized by the flexible sigmoidoscopy, a biopsy to rule out possible cancerous lesions is recommended (Choice D). Another purpose of a biopsy is to distinguish ulcerative colitis from Crohn's disease, which is treated much differently.

Sulfasalazine (Choice C) is a drug that contains 5-aminosalicyclic acid (5-ASA), and it helps in controlling inflammation. But this is not the next best step in this case.

Some people whose symptoms are triggered by certain foods are able to control the symptoms by avoiding foods that upset their intestines, like highly seasoned foods, raw fruits and vegetables, or milk sugar (lactose). So this is an area that needs to be addressed but after the biopsy rules out colon cancer (Choice B).

Corticosteroids and Immunomodulators such as azathioprine and 6-mercapto-purine (6-MP) are also part of the treatment regimen for inflammatory bowel disease but not at this stage (Choices E, F).

About 25 percent to 40 percent of ulcerative colitis patients must eventually have their colons removed because of massive bleeding, severe illness, rupture of the colon, or risk of cancer. Sometimes the physician will recommend removing the colon if medical treatment fails or if the side effects of corticosteroids or other drugs threaten the patient's health (Choice A).

Educational Objective:
If a questionable ulcer is visualized by the flexible sigmoidoscopy, a biopsy to rule out possible cancerous lesions is recommended .
Mr. Claude, a 35-year-old man, comes to his primary care physician for his gastrointestinal complaints for the last 4 months. Right now he has fever, bloody diarrhea, nausea, and severe abdominal cramps. He has lost 20 lb(9kg) and is also feeling fatigued and anorexic. His vitals are, Temperature: 37.3C(99.1F); BP: 110/74mm of Hg; PR: 98/min; RR: 22/min. On examination, remarkable findings are tenderness in right lower quadrant and skin rash. Digital Rectal examination is positive for occult blood. Blood work returns as:

WBC: 11,600mm3
Hemoglobin: 9.6
Hematocrit: 31%,
Platelets: 214,000mm3

The flexible sigmoidoscopy reveals larger areas of ulceration within the colon. What is the next best step in the management?

A. Refer him for procto-colectomy
B. Tell him to avoid highly seasoned foods and send him home
C. Give him sulfasalazine
D. Perform a biopsy of the colon lesion
E. Give him a bolus of corticosteroids
F. Start 6-mercapto-purine (6-MP)
If a questionable ulcer is visualized by the flexible sigmoidoscopy, a biopsy to rule out possible cancerous lesions is recommended (Choice D). Another purpose of a biopsy is to distinguish ulcerative colitis from Crohn's disease, which is treated much differently.

Sulfasalazine (Choice C) is a drug that contains 5-aminosalicyclic acid (5-ASA), and it helps in controlling inflammation. But this is not the next best step in this case.

Some people whose symptoms are triggered by certain foods are able to control the symptoms by avoiding foods that upset their intestines, like highly seasoned foods, raw fruits and vegetables, or milk sugar (lactose). So this is an area that needs to be addressed but after the biopsy rules out colon cancer (Choice B).

Corticosteroids and Immunomodulators such as azathioprine and 6-mercapto-purine (6-MP) are also part of the treatment regimen for inflammatory bowel disease but not at this stage (Choices E, F).

About 25 percent to 40 percent of ulcerative colitis patients must eventually have their colons removed because of massive bleeding, severe illness, rupture of the colon, or risk of cancer. Sometimes the physician will recommend removing the colon if medical treatment fails or if the side effects of corticosteroids or other drugs threaten the patient's health (Choice A).

Educational Objective:
If a questionable ulcer is visualized by the flexible sigmoidoscopy, a biopsy to rule out possible cancerous lesions is recommended .
Mr. Frazier, a 41-year-old African-American firefighter, has been your patient for the past 6 years. He is concerned because his elder brother is diagnosed with prostate cancer. He has read in the paper "cancers run in families". He thinks now is the time to do some screening for it, as he cannot even think of life with such a deadly disease. He has no complaints and no other medical history. He is having a very balanced diet, takes " a lot of garlic so that his cholesterol remains in check", and takes a low-dose aspirin daily. He does not smoke and is only social drinker. He admits to using marijuana, once in a while. What is the next best step to address the patient's concerns?


A. Order prostate specific antigen
B. Bone scan
C. Transrectal ultrasonogram
D. Perform digital rectal examination
E. Needle biopsy
Prostate cancer is a very common cancer and a leading cause of cancer deaths in USA. Screening per American Cancer Society:
1. Men age 40-50, do yearly rectal exam
2. Men age > 50, do yearly digital rectal plus PSA (Prostate Specific Antigen)

If either the rectal examination is suggestive or PSA is greater than 4 ng/dl (n= 0-4) or the age specific normal values, a transrectal ultrasonogram is usually the next best step, followed by needle biopsy and then staging. For staging of prostate cancer do the bone scan to see the bone metastasis.

Educational Objective:
Screening for prostate cancer is done as follows:
1. Men age 40-50, do yearly rectal exam
Mr. Glenn, a 26-year-old man, comes with his girlfriend to ER with a very high fever. He finished his second cycle of BEP (bleomycin, etoposide, and cisplatin) chemotherapy for metastatic seminoma 4 days ago. Other than his fever, the patient has no complaints. He denies any chest pain, cough, diarrhea or any rash. He has stopped smoking since he was diagnosed with the "deadly disease", but drinks alcohol "off and on". His Temperature: 38.9C[102F); BP: 118/70 mm Hg; PR: 102/min; RR: 19/min. On examination he appears pale. He is wearing a wig and does not have eyebrows or eyelashes. Blood tests were ordered and the results are

WBC 690/mm3 with 9% neutrophils
Hemoglobin 8.6
Hematocrit 25%,
Platelets 74,000/mm3

What is the next best step in the management of this patient?


A. Give acetaminophen
B. Admit the patient; obtain blood cultures and give IV cefepime
C. Admit the patient; obtain blood cultures and give vancomycin
D. Order blood cultures and then decide treatment according to the results
E. Give blood, platelet, and G-CSF transfusion
F. Give gentamicin
Fever in a neutropenic patient is a medical emergency. Febrile neutropenia is defined as a single temperature of >38.3ºC (101.3ºF), or a sustained temperature >38ºC (100.4ºF) for more than one hour. However, patients who are on corticosteroids may not develop fever. Neutropenia is defined as an absolute neutrophil count (ANC) <500 cells/mm3. Disruption of the skin and mucosal barrier resulting from the chemotherapy often results in seeding of bacteria into the blood stream (bacteremia) and the most common site of mucositis is in gastrointestinal tract. Chemotherapy also results in impaired immunologic function.

The frequently identified organisms in febrile neutropenic patients are Gram negatives, particularly P. aeruginosa. But there has been a recent increase in Gram positive infections for several reasons. Although GI tract has abundant anaerobic organisms, anaerobic coverage is not indicated in the initial empirical antibiotic regimen. Anaerobic coverage is indicated if there is any evidence of necrotizing mucositis, periodontal abscess, perirectal abscess/cellulitis, intraabdominal or pelvic infection, typhilitis (necrotizing neutropenic colitis), or anaerobic bacteremia.

The empirical antibiotic choice should cover the Gram-negative organisms especially Pseudomonas. Empiric treatment with ceftazidime/cefepime is the first step in these patients. The patient should be admitted and his response monitored. Third-generation cephalosporins are active against Pseudomonas aeruginosa, the Enterobacteriaceae, and many gram-positive organisms so they are used as first choice drugs. Always blood cultures should be obtained prior to initiating antibiotics.

The benefit of colony stimulating factors as an adjuvant to antibiotic therapy in an uncomplicated febrile neutropenic patents is not proven and is not indicated. However, it is considered in patients with predictive poor outcome such as patients with ANC <100/µL, uncontrolled primary disease, pneumonia, hypotension, multiorgan dysfunction, or invasive fungal infection.

Just giving acetaminophen will not be useful in this patient because of serious drop of neutrophils and very high fever.

It is always useful to try to find the cause of fever by ordering blood cultures but the cause is not identified in most of the post chemotherapy neutropenic patients. Many of the organisms responsible for infection are in fact endogenous normal flora.

The patient is anemic and may need some sort of effort to have his hemoglobin restored but this is not the issue right now.

Aminoglycosides are contraindicated in patients receiving cisplatin chemotherapy because of the potential for significant nephrotoxicity. Gentamycin or vancomycin alone doesn’t cover the pseudomonas.

Educational Objective:
Febrile neutropenia is a medical emergency; patient should be admitted in hospital and started on broad-spectrum antibiotics preferably 3rd generation cephalosporins after obtaining blood cultures
Mr. James is a 42-year-old white man who has been very worried recently because of a dark patch in his armpits that is "just not going away". The darker skin is also slightly elevated with the appearance of dirt that won't wash off. He is otherwise “healthy as a horse” and is not taking any medicines. On further inquiry, he says that he is occasionally having pain in his upper abdomen and also had two incidents of blood in vomitus in the last couple of months. He takes one baby aspirin daily, for prevention of heart attack. On examination, his vitals are stable but there is tenderness in the epigastric region. There is a light-brown-to-black, velvety, rough area under the right arm. What is your most probable underlying diagnosis?


A. Psoriasis
B. Gastric carcinoma
C. Irritative dermatitis
D. Malignant melanoma
E. Aspirin induced drug reaction
The patient has Acanthosis nigricans. Acanthosis nigricans is a disease that causes dark, thick areas on the skin. The areas affected are spread out, and the skin is velvety. It is most common in the armpits and other body folds. It may begin at any age. Acanthosis nigricans is most often associated with being overweight. It may also be associated with various tumors like gastric carcinoma; so with the signs and symptoms as in this patient he needs complete work up for gastric carcinoma like endoscopy and biopsy.

Psoriasis is a chronic scaling disease of the skin that normally affects elbow, knees or scalp (Choice A). Aspirin does not cause acanthosis nigricans (Choice E). Melanoma (Choice D) and dermatitis (Choice C) should not be high on differential diagnosis in a case like this.

Educational Objective:
Acanthosis nigricans might be a manifestation of occult gastrointestinal malignancy.
Mr. Jason, a 62-year-old man, comes to the office because of progressive fatigue and malaise, for the last 2 months. He is also having difficulty walking and feels some tingling in his feet. He has been quite sick for the last 2-3 years, and had a gastric resection because of a gastric carcinoma 1 year ago. He has stopped smoking and drinking for the last 2 years. He lives in a nursing home and watches TV, all the time. On examination, vitals are stable and there is marked pallor. Labs are as under:

WBC 8,600/cmm
Hemoglobin 8.9
Hematocrit 31%
Platelets 191,000/cmm

Peripheral smear showed hypersegmented neutrophils. Which of the following RBC indices are expected in this patient?

MCV (fl), MCH (pg), MCHC (%)


A. 88, 28, 32
B. 97.6, 29.2, 31
C. 100.4, 29.3, 38
D. 110.4, 38.6, 30
E. 68.4, 16, 23
The patient has Vitamin B12 deficiency secondary to absence of Intrinsic factor (IF), due to gastric resection. Intrinsic factor is a glycoprotein, produced by parietal cells of the stomach. They are essential for proper absorption of Vitamin B12 from diet. Vitamin B12 results in megaloblastic anemia. Neurological signs such as peripheral neuropathies, motor, sensory or even autonomic dysfunction is part of the syndrome. Megaloblastic anemias are characterized by elevated MCV, 90% of the times elevated MCH, and normal MCHC (Choice D). On the peripheral film hypersegmented neutrophils are seen with mean lobe count greater than four. The reticulocyte count is decreased although the bone marrow is hypercellular. The most specific test is low B12 level. Schilling test may be done to differentiate between pernicious anemia and other causes of vitamin B12 deficiency.

Choices A and B are seen in normal conditions or is the case in acute blood loss anemia.

Choice E is the picture of Iron deficiency anemia.

Educational Objective:
Know the blood indices in various types of anemias.Megaloblastic anemias are characterized by elevated MCV, 90% of the times elevated MCH, and normal MCHC.
Mr. Jason, a 62-year-old man, comes to the office because of progressive fatigue and malaise, for the last 2 months. He is also having difficulty walking and feels some tingling in his feet. He has been quite sick for the last 2-3 years, and had a gastric resection because of a gastric carcinoma 1 year ago. He has stopped smoking and drinking for the last 2 years. He lives in a nursing home and watches TV, all the time. On examination, vitals are stable and there is marked pallor. Labs are as under:

WBC 8,600/cmm
Hemoglobin 8.9
Hematocrit 31%
Platelets 191,000/cmm

Peripheral smear showed hypersegmented neutrophils. Which of the following RBC indices are expected in this patient?

MCV (fl), MCH (pg), MCHC (%)


A. 88, 28, 32
B. 97.6, 29.2, 31
C. 100.4, 29.3, 38
D. 110.4, 38.6, 30
E. 68.4, 16, 23
The patient has Vitamin B12 deficiency secondary to absence of Intrinsic factor (IF), due to gastric resection. Intrinsic factor is a glycoprotein, produced by parietal cells of the stomach. They are essential for proper absorption of Vitamin B12 from diet. Vitamin B12 results in megaloblastic anemia. Neurological signs such as peripheral neuropathies, motor, sensory or even autonomic dysfunction is part of the syndrome. Megaloblastic anemias are characterized by elevated MCV, 90% of the times elevated MCH, and normal MCHC (Choice D). On the peripheral film hypersegmented neutrophils are seen with mean lobe count greater than four. The reticulocyte count is decreased although the bone marrow is hypercellular. The most specific test is low B12 level. Schilling test may be done to differentiate between pernicious anemia and other causes of vitamin B12 deficiency.

Choices A and B are seen in normal conditions or is the case in acute blood loss anemia.

Choice E is the picture of Iron deficiency anemia.

Educational Objective:
Know the blood indices in various types of anemias.Megaloblastic anemias are characterized by elevated MCV, 90% of the times elevated MCH, and normal MCHC.
Mr. Javier, a 74-year-old immigrant from Columbia, comes with having "all sorts of problems of old age". He has pain all over his body. He's been feeling weak and numb in his feet, for the last several weeks. He is also having visual problems and finds it difficult to walk. He has multiple bruises on his body. Quite often, he has headaches and feels dizzy. He is living with his son and is not happy with the way his son treats him, though he denies all sort of physical abuse. His past history is unremarkable but his mother had some "blood disease". His vitals are stable. Physical examination shows enlargement of the lymph nodes, spleen, and liver. There are also sensory deficits in feet. Ophthalmoscopy shows retinal veins that are bleeding. Laboratory studies show:

WBC 10,200mm3
With Hemoglobin 9.6
Hematocrit 29%,
Platelets 14,000mm3
Sodium 141 mEq/L
Potassium 3.6 mEq/L
Blood urea nitrogen 18 mg/dL
Creatinine 0.8 mg/dL
Glucose 115 mg/dL

You get an IgM protein spike on electrophoresis. What is the most likely diagnosis?


A. Multiple myeloma
B. Waldenström’s macroglobulinemia
C. Monoclonal gammopathy of undetermined significance
D. Elderly abuse
E. Heavy chain disease
Waldenström’s Macroglobulinemia is a rare chronic plasma cell neoplasm. In Waldenström’s Macroglobulinemia, abnormal plasma cells multiply out-of-control. They invade the bone marrow, lymph nodes, and spleen. They produce excessive amounts of an antibody called, IgM. Excess IgM in the blood causes hyperviscosity (thickening) of the blood. Following are the major signs and symptoms of the disease:
1. Increased size of the spleen, liver, and some lymph nodes
2. Tiredness, usually the result of anemia (too few red blood cells)
3. A tendency to bleed easily, and to bruise easily (too few platelets)
4. Night sweats
5. Headache and dizziness
6. Various visual problems
7. Pain and numbness in the extremities are due to a predominantly demyelinating sensorimotor neuropathy.

Multiple Myeloma may present with similar symptoms but immunoglobulins are usually either IgG or IgA. They do not have hyperviscocity syndromes like retinal vein engorgement.

Monoclonal gammopathy of undetermined significance is mostly asymptomatic or has mild symptoms.

Heavy chain disease presents like abdominal lymphoma. IgA is the main issue in it.

This case does not fit with a scenario of elderly abuse. You cannot interpret this as abuse when there are clear abnormalities noted on evaluation.

Educational Objective:
In Waldenström’s Macroglobulinemia, IgM is produced in excess amounts which causes hyperviscosity of the blood. IgM spike on electrophoresis and hyperviscosity are two important diagnostic clues for this disorder.
Mr. Javier, a 74-year-old immigrant from Columbia, comes with having "all sorts of problems of old age". He has pain all over his body. He's been feeling weak and numb in his feet, for the last several weeks. He is also having visual problems and finds it difficult to walk. He has multiple bruises on his body. Quite often, he has headaches and feels dizzy. He is living with his son and is not happy with the way his son treats him, though he denies all sort of physical abuse. His past history is unremarkable but his mother had some "blood disease". His vitals are stable. Physical examination shows enlargement of the lymph nodes, spleen, and liver. There are also sensory deficits in feet. Ophthalmoscopy shows retinal veins that are bleeding. Laboratory studies show:

WBC 10,200mm3
With Hemoglobin 9.6
Hematocrit 29%,
Platelets 14,000mm3
Sodium 141 mEq/L
Potassium 3.6 mEq/L
Blood urea nitrogen 18 mg/dL
Creatinine 0.8 mg/dL
Glucose 115 mg/dL

You get an IgM protein spike on electrophoresis. What is the most likely diagnosis?


A. Multiple myeloma
B. Waldenström’s macroglobulinemia
C. Monoclonal gammopathy of undetermined significance
D. Elderly abuse
E. Heavy chain disease
Waldenström’s Macroglobulinemia is a rare chronic plasma cell neoplasm. In Waldenström’s Macroglobulinemia, abnormal plasma cells multiply out-of-control. They invade the bone marrow, lymph nodes, and spleen. They produce excessive amounts of an antibody called, IgM. Excess IgM in the blood causes hyperviscosity (thickening) of the blood. Following are the major signs and symptoms of the disease:
1. Increased size of the spleen, liver, and some lymph nodes
2. Tiredness, usually the result of anemia (too few red blood cells)
3. A tendency to bleed easily, and to bruise easily (too few platelets)
4. Night sweats
5. Headache and dizziness
6. Various visual problems
7. Pain and numbness in the extremities are due to a predominantly demyelinating sensorimotor neuropathy.

Multiple Myeloma may present with similar symptoms but immunoglobulins are usually either IgG or IgA. They do not have hyperviscocity syndromes like retinal vein engorgement.

Monoclonal gammopathy of undetermined significance is mostly asymptomatic or has mild symptoms.

Heavy chain disease presents like abdominal lymphoma. IgA is the main issue in it.

This case does not fit with a scenario of elderly abuse. You cannot interpret this as abuse when there are clear abnormalities noted on evaluation.

Educational Objective:
In Waldenström’s Macroglobulinemia, IgM is produced in excess amounts which causes hyperviscosity of the blood. IgM spike on electrophoresis and hyperviscosity are two important diagnostic clues for this disorder.
Mr. Kaiser is a 28-year-old male who is very worried after having several episodes of dark brown urine in the morning, during the past 2 months. He is also fatigued easily. On examination, pallor and mild hepatosplenomegaly are present. CBC and urinalysis of first morning urine are ordered and results are:

WBC 5,600mm3 with 55 % lymphocytes
Hemoglobin 8.0 gm/dL
Hematocrit 23%
RBC count 2.6million
Platelets 210,000mm3
Reticulocyte count is 10%

Urinalysis:

Urine Color Brown
Specific Gravity 1.023
PH 6.7
Urine ketone none
Urine Protein none
Urine Glucose none
Urine Nitrite negative
Leucocyte esterase negative
RBCs none
WBCs 1-2/HPF
Squamous cells none

Urine was also reported positive for hemosiderin.

What is the underlying pathology in this disease?


A. RBC membrane defect
B. RBC enzyme deficiency
C. Mutations in hemoglobin chain
D. Iron deficiency
E. Peripheral destruction of RBC
The patient has Paroxysmal Nocturnal Hemoglobinuria (PNH). A red cell membrane defect causes increased binding of complement to the red cell leading to increased intravascular hemolysis resulting in marked anemia. These cells are more susceptible to lysis in an acidic environment. Due to relative hypoventilation at night, there is mild acidosis. So the hemolysis is more at night and hemosiderinuria is common in the first morning urine. Thrombosis of major venous structures especially the hepatic vein is the common cause of death in these patients. They have usual findings on hemolysis on the lab like increased LDH, bilirubin, and reticulocyte count. Test specific to PNH are sugar-water test and the acidified-hemolysis (Ham) test, which determines the increased susceptibility of cells to lysis by complement. Decay Activating Factor (DAF) is diminished in PNH.

RBC enzyme deficiency like Glucose-6-Phosphate dehydrogenase deficiency results in sudden onset of hemolysis on oxidant stress like infections or drugs.

Mutations in hemoglobin chain are seen in Thalassemia and Sickle Cell Disease.

In Iron deficiency anemia there is no hyper bilirubinemia and microcytic hypochromic picture is seen on peripheral blood smear.

Peripheral destruction of RBC is usually seen in hereditary spherocytosis.

Educational Objective:
In Paroxysmal Nocturnal Hemoglobinuria , a red cell membrane defect causes increased binding of complement to the red cell leading to increased intravascular hemolysis.
Mr. Kaiser is a 28-year-old male who is very worried after having several episodes of dark brown urine in the morning, during the past 2 months. He is also fatigued easily. On examination, pallor and mild hepatosplenomegaly are present. CBC and urinalysis of first morning urine are ordered and results are:

WBC 5,600mm3 with 55 % lymphocytes
Hemoglobin 8.0 gm/dL
Hematocrit 23%
RBC count 2.6million
Platelets 210,000mm3
Reticulocyte count is 10%

Urinalysis:

Urine Color Brown
Specific Gravity 1.023
PH 6.7
Urine ketone none
Urine Protein none
Urine Glucose none
Urine Nitrite negative
Leucocyte esterase negative
RBCs none
WBCs 1-2/HPF
Squamous cells none

Urine was also reported positive for hemosiderin.

What is the underlying pathology in this disease?


A. RBC membrane defect
B. RBC enzyme deficiency
C. Mutations in hemoglobin chain
D. Iron deficiency
E. Peripheral destruction of RBC
The patient has Paroxysmal Nocturnal Hemoglobinuria (PNH). A red cell membrane defect causes increased binding of complement to the red cell leading to increased intravascular hemolysis resulting in marked anemia. These cells are more susceptible to lysis in an acidic environment. Due to relative hypoventilation at night, there is mild acidosis. So the hemolysis is more at night and hemosiderinuria is common in the first morning urine. Thrombosis of major venous structures especially the hepatic vein is the common cause of death in these patients. They have usual findings on hemolysis on the lab like increased LDH, bilirubin, and reticulocyte count. Test specific to PNH are sugar-water test and the acidified-hemolysis (Ham) test, which determines the increased susceptibility of cells to lysis by complement. Decay Activating Factor (DAF) is diminished in PNH.

RBC enzyme deficiency like Glucose-6-Phosphate dehydrogenase deficiency results in sudden onset of hemolysis on oxidant stress like infections or drugs.

Mutations in hemoglobin chain are seen in Thalassemia and Sickle Cell Disease.

In Iron deficiency anemia there is no hyper bilirubinemia and microcytic hypochromic picture is seen on peripheral blood smear.

Peripheral destruction of RBC is usually seen in hereditary spherocytosis.

Educational Objective:
In Paroxysmal Nocturnal Hemoglobinuria , a red cell membrane defect causes increased binding of complement to the red cell leading to increased intravascular hemolysis.
Mr. Mathews is a 44-year-old man who came to you with several episodes of hemoptysis 2 months ago. He has been smoking 2-3 packs for the last 24 years. After complete examination and chest X-ray you suspected a lung malignancy. You ordered CT chest and bronchoscopy with biopsy. The patient went on a vacation to Aruba. In the mean time you get back the results, which shows that tumor, has spread to hilar and mediastinal lymph nodes with bony extension. He has an appointment with you to discuss the results of the findings. What is the most appropriate statement to start the discussion?


A. Mr. Mathews, How much would you like to know about your condition?
B. Mr. Mathews, I am sorry to tell you that you have lung cancer and it is fairly advanced.
C. Mr. Mathews, What do you think about your symptoms?
D. Mr. Mathews, Unfortunately, the situation is more serious than what I earlier thought.
E. Mr. Mathews, Would you like to have some one else with you as I don't have good news for you?
Breaking bad news is a very sensitive issue. It is important that proper protocol has to be followed so that the patient can absorb the reality as smoothly as possible. Following are the steps that should be followed.

1. Patient should be in a comfortable environment.
2. Ask patient how much he knows or what he thinks of his condition to have an idea about his expectations. (Choice C)
3. Ask the patient how much he wants to know. (Choice A)
4. Give him a warning shot. (Choice D, E)
5. Break the news if he wants that. (Choice B)
6. Tell his prognosis but always keep him aware of all the options available to make his life as comfortable as possible.
7. Try to explain every thing clearly and as simple as possible.

Educational Objective:
Breaking bad news is a very sensitive issue and a big time topic for board exams. They love these questions.
Mr. Miranda, a 50-yr-old African-American man is feeling weak, for the past year or so. He gets tired early and has lost his “humor”. He is also finding it difficult to remember things. He is jobless and is living on social security benefits. He denies smoking or alcohol use but admits being charged with driving under intoxication three times. His father and mother died of old age. He shares his room with four of his friends. His vitals are normal, but there is pallor on examination. CBC is ordered and the results are:

WBC 5,500mm3
Hemoglobin 7 mg/dL
Hematocrit 24%,
Platelets 196,000mm3
RBC count 1.7 million
MCV 119
MCH 36
MCHC 28%
Reticulocyte count 0.4%

Peripheral smear showed anisocytosis, poikilocytosis, and basophilic stippling. What is the next step?


A. Iron studies
B. Osmotic fragility test
C. Serum B12 and folate levels
D. Sugar water test
E. Bone marrow biopsy with prussian blue staining
F. Blood lead levels
The patient has macrocytic anemia which is characterized by elevated MCV, 90% of the times elevated MCH, and normal MCHC. Macrocytic anemias are classified in 3 major categories:
1.Vitamin B12 deficiency
2.Folic acid deficiency
3.Miscellaneous like hypothyroidism, severe liver disease, and antimetabolites

The typical findings on the peripheral blood smear are hypersegmented neutrophils with a mean lobe count greater than 4, and macro ovalocyte RBCs. Anisocytosis, poikilocytosis, and basophilic stippling is also seen. The reticulocyte count is decreased although the bone marrow is hypercellular. Neurological deficits are mainly seen with pernicious anemia, which is the most important cause of vitamin B12 deficiency. Supporting laboratory assays of folic acid and vitamin B12 can differentiate between pernicious anemia and folic acid deficiency. This is important because the treatment regimen is different for the two types.

The difference between the two types is tabulated below

Laboratory Tests Pernicious Anemia, Folic Acid Def.
1. Serum B12: Decreased, Normal
2. Serum folic acid: Normal, Decreased
3. Serum LDH: Increased, Normal
4. Achlorhydria: Present, Absent

5. Schilling test: Positive, Negative
6. Methyl malonyl CoA in urine: Present, Absent
7. Neurological signs: Present, Absent

Iron studies are the tests done in microcytic anemia, where the MCV is <80.

The osmotic fragility test is the done to determine the diagnosis of hereditary spherocytosis.

Sugar water test is done for the diagnosis of Paroxysmal Nocturnal Hemoglobinuria (PNH).

Bone marrow biopsy is not of diagnostic significance in this condition. Prussian blue staining is used for sideroblastic anemia.

In lead poisoning you see iron deficiency anemia picture (MCV<80). Basophilic stippling is a distractor here.

Educational Objective:
Patients with high MCV should be tested for vitamin B12 and folic acid levels. Know how to differentiate between various causes of megaloblastic anemia.
Mr. Ruth, a 33-year-old computer programmer, was diagnosed with ulcerative colitis last year. He had a colonoscopy and biopsy that showed pancolitis but after that there has been no work-up. This is his first visit to a physician in 6 months. He is taking aminosalicylates and is in remission, without any symptoms. On examination, vitals are stable but he has mild tenderness on deep palpation in lower abdomen. He is a member of Ulcerative Colitis Support Group that has sent him a letter recently that says he should be screened for colon cancer. What is the best advice for him regarding his concerns?


A. He has increased risk of colon cancer and he should have prophylactic colectomy.
B. He has increased risk of colon cancer and he should have colonoscopy every year.
C. He has increased risk of colon cancer and he should have periodic colonoscopy eight years later.
D. Since he does not have any signs and symptoms he has no increased risk and doesn’t need any surveillance.
E. He has increased risk of colon cancer and he should have biopsy.
The American Society for Gastrointestinal Endoscopy (ASGE) recommends that patients with ulcerative colitis who have pancolitis should begin surveillance colonoscopy after eight years of disease (Choice C). Four biopsies should be obtained every 10cm from the cecum to the rectum. In addition, any suspicious lesions or masses should be biopsied. Colonoscopy should be repeated every one to three years. Such screening has not been proven to reduce the risk of colon cancer, but it may help identify cancer early should it develop.

There is no need for biopsy or colonoscopy at this time according to present recommendations (Choice B, E). Prophylactic colectomy is not useful at all in ulcerative colitis (Choice A).

About 5 percent of people with ulcerative colitis develop colon cancer (Choice D). The risk of cancer increases with the duration and the extent of involvement of the colon. For example, if only the lower colon and rectum are involved, the risk of cancer is no higher than normal. However, if the entire colon is involved, the risk of cancer may be as much as 32 times the normal rate.

Educational Objective:
The American Society for Gastrointestinal Endoscopy (ASGE) recommends that patients with ulcerative colitis who have pancolitis should begin surveillance colonoscopy after eight years of disease.
Mr. Scott, a 19-year-old man, comes to you saying, "Doctor! I have been having a peculiar problem lately. My stools are of funny color". First they were black and almost sticky and yesterday they became maroon. In fact, today I saw some bright red blood". He denies any associated pain or fever. He is adopted and his family history is unknown. On examination the vitals are stable, abdomen non-tender but stools are positive for occult blood. Laboratory tests show a hematocrit of 29% and hemoglobin of 9.6. On flexible sigmoidoscopy, you see hundreds of colonic polyps. On biopsy they turn out to be adenomatous polyp. What is the appropiate recommendation for this patient?


A. Reassure the patient as the polyps are most probably benign and have no long-term complications.
B. Perform regular colonoscopy and biopsy every three years from now on to see for any malignant change.
C. Start regular colonoscopy and biopsy eight years from now on.
D. The patient needs elective procto-colectomy.
E. The patient needs close surveillance with regular FOBT and CEA levels.
Patient has hundreds of adenomas in the colon and has Familial colonic polyposis. They have 100% risk of cancer, if not treated. These patients require a proctocolectomy, before the age of 20. Familial Colonic Polyposis is autosomal dominant disease caused by mutations in the adenomatous polyposis coli (APC) gene. FAP occurs in approximately 1/10,000 to 1/30,000 live births, and accounts for less than 1 % of the total colon cancer risk in the United States. It affects both sexes equally and has a worldwide distribution. There is no consensus on the regime for surveillance in such patients. All the different methods, as in other choices are tried in various trials, but have not proved to be very encouraging.

Educational Objective:
Patients with familial colonic polyposis have 100% risk of cancer, if not treated. These patients require a proctocolectomy, before the age of 20.
Mr. Woods, a 28-year-old Caucasian male, comes to office for complete physical examination for a new job he is starting next week. He is healthy and not having any complaints. He smokes 1 pack/day cigarettes for the last 10 years, and is only a social drinker. His mother has hypertension and father has heart problems. Physical examination is within normal limits except for a painless, hard mass in left testicle. Testicular ultrasound result puts the patient on high suspicion of carcinoma. What is the next best step in management?


A. Observe and recheck the mass after one month
B. Platinum based chemotherapy
C. Trans-scrotal biopsy
D. Fine needle aspiration cytology
E. Radical orchiectomy
After the diagnosis of a solid testicular mass has been made, (a painless hard mass in testicle + suggestive ultrasound), the initial management is removal of the testis and its associated cord, orchiectomy. This is done through a small inguinal incision. The procedure is called high inguinal orchiectomy. The testis and abnormal tissue present is then examined under the microscope to determine the type of cancer. Depending on the cell type of the cancer present other therapies, i.e. additional surgery, radiation therapy, or possibly chemotherapy may be indicated.

Using a combination of these therapies, testicular cancer has one of the highest cure rates of all cancers. This has been achieved using a combination of surgery and chemotherapy, as well as radiation in certain instances. Cure from testicular cancer can even be achieved in individuals who have had the cancer spread into other parts of the body. This is the tumor that we kill first and investigate later. So observation, chemotherapy, and biopsies are not the next best step in this case.

FNAC, or transscrotal biopsy, is contraindicated because of the risk of spillage of cancer cells, which can potentially spread through lymphatics and blood vessels.

Educational Objective:
After the diagnosis of a solid testicular mass has been made, (a painless hard mass in testicle + suggestive ultrasound), the initial management is removal of the testis and its associated cord.
Mr. Woods, a 28-year-old Caucasian male, comes to office for complete physical examination for a new job he is starting next week. He is healthy and not having any complaints. He smokes 1 pack/day cigarettes for the last 10 years, and is only a social drinker. His mother has hypertension and father has heart problems. Physical examination is within normal limits except for a painless, hard mass in left testicle. Testicular ultrasound result puts the patient on high suspicion of carcinoma. What is the next best step in management?


A. Observe and recheck the mass after one month
B. Platinum based chemotherapy
C. Trans-scrotal biopsy
D. Fine needle aspiration cytology
E. Radical orchiectomy
After the diagnosis of a solid testicular mass has been made, (a painless hard mass in testicle + suggestive ultrasound), the initial management is removal of the testis and its associated cord, orchiectomy. This is done through a small inguinal incision. The procedure is called high inguinal orchiectomy. The testis and abnormal tissue present is then examined under the microscope to determine the type of cancer. Depending on the cell type of the cancer present other therapies, i.e. additional surgery, radiation therapy, or possibly chemotherapy may be indicated.

Using a combination of these therapies, testicular cancer has one of the highest cure rates of all cancers. This has been achieved using a combination of surgery and chemotherapy, as well as radiation in certain instances. Cure from testicular cancer can even be achieved in individuals who have had the cancer spread into other parts of the body. This is the tumor that we kill first and investigate later. So observation, chemotherapy, and biopsies are not the next best step in this case.

FNAC, or transscrotal biopsy, is contraindicated because of the risk of spillage of cancer cells, which can potentially spread through lymphatics and blood vessels.

Educational Objective:
After the diagnosis of a solid testicular mass has been made, (a painless hard mass in testicle + suggestive ultrasound), the initial management is removal of the testis and its associated cord.
Mr.Scott is a 47-year-old man who works in postal service (USPS). He comes to you because he has a sore over the right ear. He explains that it bleeds and oozes and has remained open for 3 weeks. He thinks that it is growing slowly over the past 1year. He has to be out all day delivering mail and smokes almost 2packs/day of cigarettes for the last 13 years. On examination, you find a punched out 1.5cm ulcer on his right auricle. You don't find any enlarged lymph nodes. Full thickness biopsy reveals proliferation of malignant cells, arising from the epidermis and extending into the dermis in lobules or strands. What is the next best step in management?


A. Surgical excision with 1-2 mm of clear margins
B. Wide excision with at least 5 cm margins
C. Scraping
D. Observe for 3 weeks and then examine
E. Local chemotherapy with anticancer creams.
The patient has basal cell carcinoma. According to Society for Prevention of Skin Cancers following are the five warning signs of basal cell carcinoma:
1. Open Sore that bleeds, oozes, or crusts and remains open for three or more weeks.
2. A Reddish Patch or irritated area
3. A Shiny Bump or nodule, that is pearly or translucent and is often pink, red, or white.
4. A Pink Growth with a slightly elevated rolled border and a crusted indentation in the center.
5. A Scar-like Area, which is white, yellow or waxy, and often has poorly defined borders.

Basal cell carcinoma is the most common form of skin cancer in the US. It accounts for about 75% of all skin cancers. This cancer usually remains local and almost never spreads to distant parts of the body, but it may continue to grow and invade nearby tissues and structures, including the nerves, bones, and brain. The tumor may begin very small, growing to 1-2cm in diameter, after several years of growth. Treatment varies depending on the size, depth, and location of the cancer.

The carcinoma is removed using one of these procedures:
1. Cauterization (burning).
2. Surgical removal, including microscopic shaving (Mohs' surgery) i.e. excision with 1-2mm of clear margins.
3. Cryosurgery (freezing).
4. Radiation.

Early treatment by a dermatologist may result in a cure rate of more than 95%. New sites of basal cell cancer can occur, so monitoring should be diligent. Regular examination by a health care provider is required.

Educational Objective:
One of the recommended treatment option for basal cell carcinoma is surgical removal, including microscopic shaving (Mohs' surgery) i.e. excision with 1-2 mm of clear margins.
Mr.Tyson, a 57-year-old man, comes with long-standing heartburn and chest pain. The pain is burning in nature and is unrelated to eating. The patient, for the past week or so, had decreased interest in things and has been more tired than usual. He thinks that he has lost almost 40 lb(18kg) in the last 6 months. He used to smoke 6-10 cigarettes/day but quitted 4 years ago. He drinks alcohol, once in a while. His father died at the age of 67 with lung cancer. He takes ranitidine for his heartburn. His vitals are stable. No other abnormality is noted. Portable chest x-ray is within normal limits. What is the next best step?


A. Bronchoscopy
B. CT scan chest with and without contrast
C. Barium swallow followed by endoscopy
D. Give omeprazole and return visit in 2 months
E. Test and eradicate helicobacter pylori infection
Patient’s symptom of heartburn, along with so much weight loss, has to be investigated thoroughly. He is probably having esophageal cancer. Esophageal cancer is relatively uncommon in the United States, and occurs most often in men over 50 years old. There are two main types of esophageal cancer, distinguished by the way they look under the microscope: squamous cell carcinoma and adenocarcinoma. Squamous cell cancer is associated with smoking and alcohol consumption. The incidence of this disease in the United States has remained relatively constant, while the incidence of adenocarcinoma of the esophagus has risen dramatically. Barrett's esophagus, a complication of gastroesophageal reflux disease (GERD), is a risk factor for the development of adenocarcinoma of the esophagus. Symptoms included are difficulty swallowing, regurgitation of food, heartburn, weight loss, vomiting blood and chest pain unrelated to eating.

The investigations include Barium swallow, EGD (esophagogastroduodenoscopy) and biopsy, and PET scan (sometimes useful to determine the stage of disease and whether surgery is possible). When esophageal cancer is localized to the esophagus and has not spread elsewhere (metastasized), surgery is the treatment of choice. The goal of surgery, in most cases, is to cure the patient. In some circumstances chemotherapy, radiation, or a combination of the two will be used to make surgery easier to perform. In patients who cannot tolerate surgery, or in situations where the cancer has spread to other organs (metastatic disease), chemotherapy or radiation may be used to help alleviate symptoms (palliative therapy). In such circumstances, however, the disease is usually not curable.

CT chest is considered if the upper GI endoscopy is negative. Bronchoscopy is indicted if there is anything on CT or the patient have high index of suspicion (hemoptysis) for endobronchial lesion. CT is not the first step in a patient with normal CXR, and other system symptoms.

H. Pylori eradication should be considered if this patient is proven to have mucosa associated lymphoid tissue lymphoma on endoscopy and biopsy.

Even though switching to omeprazole is an option, to give him some symptomatic benefit, his weight loss should be addressed seriously and return visit in 2 months, is not an acceptable choice here.

Educational Objective:
Barium swallow followed by endoscopy is usually done when a patient is suspected of having an esophageal cancer.
Ms .Eva, a 25-year-old white female, presents to the clinic with persisting pain in her wrists and ankles for the last 3 months. The pain is 3/10 and is partially relived by ibuprofen. She also has a rash on her face. She denies smoking and drinks alcohol occasionally. Her Temperature: 37.3C(99.2F); BP: 120/80 mm Hg; PR: 79/min; RR: 18/min. On exam, swollen joints of the hands and ankle are present. There is also an erythema over the bridge of the nose and the upper cheeks. There is no muscle weakness.

Labs showed:

WBC 13,600/mm3
Hemoglobin 13.0
Hematocrit 40%,
Platelets 240,000/mm3

The distribution of leukocytes was as under

Segmented neutrophils 76%
Lymphocytes 20%
Bands 2%
Monocytes: 2%

RF, ANA, and antibodies to double stranded-DNA are positive in high titers.

What is the most likely diagnosis?


A. Scleroderma
B. Systemic lupus erythematosus
C. Dermatomyositis
D. Polymyositis
E. Mixed connective tissue disease
Systemic Lupus Erythematosus (SLE) is a prototype autoimmune disease that affects almost every organ of the body. It is commonly seen in young adult females. Common symptoms are fever, arthralgia, arthritis, weight loss, and classic butterfly rash over the bridge of the nose. Renal involvement is very common and leads to acute nephritis and later chronic glomerulonephritis. The disease follows a chronic and irregular course with periods of remission. The lab findings of SLE include determination of ANA antibodies, dsDNA antibodies, and hematological tests.

Disturbance of the lymphocyte subsets is one of the important features of SLE. There is lack of or reduced suppressor T-cell function and hyper production of Helper T cells, B-cell hyperactivity leads to increased serum antibodies and IgG auto antibodies production, which later forms circulating immune complexes which are the hallmark of the disease.

Scleroderma is associative with anti –Centromere antibodies.

In Dermatomyositis and Polymyositis Anti Jo-1 antibodies are detected.

Anti- RNP antibodies are associative with the mixed connective tissue disease.

Educational Objective:
Presence of antibodies to ds-DNA is specific for the diagnosis of systemic lupus erythematosus.
Ms .Eva, a 25-year-old white female, presents to the clinic with persisting pain in her wrists and ankles for the last 3 months. The pain is 3/10 and is partially relived by ibuprofen. She also has a rash on her face. She denies smoking and drinks alcohol occasionally. Her Temperature: 37.3C(99.2F); BP: 120/80 mm Hg; PR: 79/min; RR: 18/min. On exam, swollen joints of the hands and ankle are present. There is also an erythema over the bridge of the nose and the upper cheeks. There is no muscle weakness.

Labs showed:

WBC 13,600/mm3
Hemoglobin 13.0
Hematocrit 40%,
Platelets 240,000/mm3

The distribution of leukocytes was as under

Segmented neutrophils 76%
Lymphocytes 20%
Bands 2%
Monocytes: 2%

RF, ANA, and antibodies to double stranded-DNA are positive in high titers.

What is the most likely diagnosis?

A. Scleroderma
B. Systemic lupus erythematosus
C. Dermatomyositis
D. Polymyositis
E. Mixed connective tissue disease
Systemic Lupus Erythematosus (SLE) is a prototype autoimmune disease that affects almost every organ of the body. It is commonly seen in young adult females. Common symptoms are fever, arthralgia, arthritis, weight loss, and classic butterfly rash over the bridge of the nose. Renal involvement is very common and leads to acute nephritis and later chronic glomerulonephritis. The disease follows a chronic and irregular course with periods of remission. The lab findings of SLE include determination of ANA antibodies, dsDNA antibodies, and hematological tests.

Disturbance of the lymphocyte subsets is one of the important features of SLE. There is lack of or reduced suppressor T-cell function and hyper production of Helper T cells, B-cell hyperactivity leads to increased serum antibodies and IgG auto antibodies production, which later forms circulating immune complexes which are the hallmark of the disease.

Scleroderma is associative with anti –Centromere antibodies.

In Dermatomyositis and Polymyositis Anti Jo-1 antibodies are detected.

Anti- RNP antibodies are associative with the mixed connective tissue disease.

Educational Objective:
Presence of antibodies to ds-DNA is specific for the diagnosis of systemic lupus erythematosus.
Ms. Andrews, a 30-year-old female, was admitted because of community acquired pneumonia last night and was on Zithromax (Azithromycin). You find the lady in an altered mental status with fluctuating neurological signs. Her vitalis are, Temperature: 37.9C(100.3F); BP: 108/72 mm Hg; PR: 87/min; RR: 21/min. There are marked pallor and petechial rash present on examination. The labs are:

Sodium 136 mEq/L
Potassium 3.7 mEq/L
Bicarbonate 29mEq/L
Blood urea nitrogen 24mg/dL
Creatinine 1.6 mg/dL
Glucose 82 mg/dL
WBC 8,600mm3
Hemoglobin 8.9
Hematocrit 31%,
Platelets 34,000mm3
Bleeding Time 6 min
PT 12.1s
APTT 30s

Peripheral smear shows severely fragmented red blood cells. What is the most probable diagnosis?


A. Idiopathic thrombocytopenic purpura
B. Disseminated intravascular coagulation
C. Hemolytic uremic syndrome
D. Thrombotic thrombocytopenic purpura
E. Glazmann’s thrombasthenia
Thrombotic thrombocytopenic purpura (TTP) is a serious disorder, which presents with the following classical pentad:
1.Severe thrombocytopenia
2.Microangiopathic hemolytic anemia (RBC fragments)
3.Fluctuating neurological signs
4.Renal failure
5.Fever

The patients with TTP generally presents with fever, pallor, petechial, and confusion. The peripheral smear shows RBC fragments. PT and PTT are usually normal. LDH is elevated due to hemolysis. Hemolytic uremic syndrome (HUS) and TTP come under a spectrum of diseases. If the patient has more neurologic symptoms and less of renal failure, it is considered TTP. On the other hand if the patient has significant renal failure and less neurologic symptoms it is considered HUS. However, both conditions are very serious and require emergent plasmapheresis.

Disseminated intravascular coagulation (DIC) has a similar presentation and may show RBC fragments and thrombocytopenia, but PT, PTT, Bleeding time are all elevated. In TTP the problem is mainly thrombocytopenia, so you see prolonged bleeding time. Where as in DIC the problem is mainly consumption of all the clotting factors in the early coagulation and you will have elevated PT, PTT as well as elevated BT (from thrombocytopenia). DIC usually have bleeding manifestations; all the IV lines will be oozing blood in advanced DIC. Renal failure is not necessarily a future of DIC.

Idiopathic thrombotic thrombocytopenic purpura (ITP) is a diagnosis of exclusion. They typically presents with isolated decrease in platelet count. The Hb and WBC will be normal. Although they usually manifest petechial rash, spontaneous bleeding is extremely rare. There is no gold standard test to diagnose this condition. PT, PTT will be normal. Peripheral pseudoclumping should be excluded before making thrombocytopenia in these individuals.

Glazmann’s Thrombasthenia is a qualitative platelet defect with normal platelet count but prolonged bleeding time and abnormal aggregation test. They have normal platelet count. This patient has thrombocytopenia.

Educational Objective:
Know the presentation of thrombotic thrombocytopenic purpura and its differentiating features from disseminated intravascular coagulation, idiopathic thrombocytopenic purpura and hemolytic uremic syndrome.
Ms. Andrews, a 30-year-old female, was admitted because of community acquired pneumonia last night and was on Zithromax (Azithromycin). You find the lady in an altered mental status with fluctuating neurological signs. Her vitalis are, Temperature: 37.9C(100.3F); BP: 108/72 mm Hg; PR: 87/min; RR: 21/min. There are marked pallor and petechial rash present on examination. The labs are:

Sodium 136 mEq/L
Potassium 3.7 mEq/L
Bicarbonate 29mEq/L
Blood urea nitrogen 24mg/dL
Creatinine 1.6 mg/dL
Glucose 82 mg/dL
WBC 8,600mm3
Hemoglobin 8.9
Hematocrit 31%,
Platelets 34,000mm3
Bleeding Time 6 min
PT 12.1s
APTT 30s

Peripheral smear shows severely fragmented red blood cells. What is the most probable diagnosis?


A. Idiopathic thrombocytopenic purpura
B. Disseminated intravascular coagulation
C. Hemolytic uremic syndrome
D. Thrombotic thrombocytopenic purpura
E. Glazmann’s thrombasthenia
Thrombotic thrombocytopenic purpura (TTP) is a serious disorder, which presents with the following classical pentad:
1.Severe thrombocytopenia
2.Microangiopathic hemolytic anemia (RBC fragments)
3.Fluctuating neurological signs
4.Renal failure
5.Fever

The patients with TTP generally presents with fever, pallor, petechial, and confusion. The peripheral smear shows RBC fragments. PT and PTT are usually normal. LDH is elevated due to hemolysis. Hemolytic uremic syndrome (HUS) and TTP come under a spectrum of diseases. If the patient has more neurologic symptoms and less of renal failure, it is considered TTP. On the other hand if the patient has significant renal failure and less neurologic symptoms it is considered HUS. However, both conditions are very serious and require emergent plasmapheresis.

Disseminated intravascular coagulation (DIC) has a similar presentation and may show RBC fragments and thrombocytopenia, but PT, PTT, Bleeding time are all elevated. In TTP the problem is mainly thrombocytopenia, so you see prolonged bleeding time. Where as in DIC the problem is mainly consumption of all the clotting factors in the early coagulation and you will have elevated PT, PTT as well as elevated BT (from thrombocytopenia). DIC usually have bleeding manifestations; all the IV lines will be oozing blood in advanced DIC. Renal failure is not necessarily a future of DIC.

Idiopathic thrombotic thrombocytopenic purpura (ITP) is a diagnosis of exclusion. They typically presents with isolated decrease in platelet count. The Hb and WBC will be normal. Although they usually manifest petechial rash, spontaneous bleeding is extremely rare. There is no gold standard test to diagnose this condition. PT, PTT will be normal. Peripheral pseudoclumping should be excluded before making thrombocytopenia in these individuals.

Glazmann’s Thrombasthenia is a qualitative platelet defect with normal platelet count but prolonged bleeding time and abnormal aggregation test. They have normal platelet count. This patient has thrombocytopenia.

Educational Objective:
Know the presentation of thrombotic thrombocytopenic purpura and its differentiating features from disseminated intravascular coagulation, idiopathic thrombocytopenic purpura and hemolytic uremic syndrome.
Ms. Brown, a 66-year-old woman, is your patient for the last 8 years. She was diagnosed with intramucosal colon carcinoma 2 years ago. She had an endoscopic resection. She is healthy since then and is coming to you regularly for the follow up. She is very grateful to you and has stated many times that she owes her life to you. You are a co-investigator of a retrospective observational study related to colon cancer and you want to use some of her data in it. What is the best course of action for you?


A. Use the data, as she is your patient for so many years.
B. Use the data and inform her whenever she comes next time.
C. Call her and take a verbal consent to use her data.
D. Get the data de-identified by a colleague and use it.
E. Use data only after taking informed consent.
You can only use the data of any patient in research after taking Informed consent. This includes explaining to the patient:
1. Purpose of the Research.
2. Foreseeable risks or discomforts.
3. Refusal or withdrawal would not affect current or future care that the patient is receiving.
4. Patient can withdraw from study anytime she wants to.

The consent has to be written and verbal consent is not recommended. A physician cannot use the data of the patient in a research project without an informed consent first. So, all other choices are incorrect.

Educational Objective:
Use the data of any patient in research only after taking informed consent.
Ms. Brown, a 66-year-old woman, is your patient for the last 8 years. She was diagnosed with intramucosal colon carcinoma 2 years ago. She had an endoscopic resection. She is healthy since then and is coming to you regularly for the follow up. She is very grateful to you and has stated many times that she owes her life to you. You are a co-investigator of a retrospective observational study related to colon cancer and you want to use some of her data in it. What is the best course of action for you?


A. Use the data, as she is your patient for so many years.
B. Use the data and inform her whenever she comes next time.
C. Call her and take a verbal consent to use her data.
D. Get the data de-identified by a colleague and use it.
E. Use data only after taking informed consent.
You can only use the data of any patient in research after taking Informed consent. This includes explaining to the patient:
1. Purpose of the Research.
2. Foreseeable risks or discomforts.
3. Refusal or withdrawal would not affect current or future care that the patient is receiving.
4. Patient can withdraw from study anytime she wants to.

The consent has to be written and verbal consent is not recommended. A physician cannot use the data of the patient in a research project without an informed consent first. So, all other choices are incorrect.

Educational Objective:
Use the data of any patient in research only after taking informed consent.
Ms. Donna is a 65-year-old woman who has been your patient for the last 15 years. She is very health-conscious and reads all the health magazines regularly. She doesn’t smoke, and drinks alcohol only socially. She drinks cranberry juice to have her kidneys stay strong and takes a lot of garlic to control her cholesterol. She exercises regularly for 30 min, in the morning. She is having regular colonoscopy, mammograms, and pap smears. While you are appreciating her healthy life style she tells you of her mother, who died of an ovarian cancer. She wants you to perform not only her abdominal ultrasound every 6 months, but also “everything” so that her cancers are detected early. What are your recommendations to her?


A. There is no evidence that ultrasound surveillance has any role in decreasing mortality from ovarian cancer.
B. CXR, EGD, and Abdominal ultrasound do help in detecting cancers early.
C. Abdomen ultrasound is not effective for detecting ovarian cancer early but CXR surveillance has helped in decreasing mortality in lung cancer.
D. Perform Ultrasound six monthly as it is non-invasive procedure and save you from any risk of being sued for malpractice.
E. Reassure her that with healthy life style she can’t have cancers.
There is a lot of work going on to devise proper and effective screening for various cancers, as this is the best way to prevent mortality from cancer. “Catch them early before they catch you”, is the buzzword these days. But despite all the euphoria surrounding the policy, there is no evidence that ultrasound of abdomen can help in decreasing mortality from ovarian cancer. Nor is there any data supporting that regular CXR can prevent Lung Cancer deaths. A study in Sweden showed that CXR screening for lung cancer has a very low sensitivity and specificity and leads to false alarms. There is a lot of revenue wastage without any improvement in overall mortality or morbidity.

Patient is having a healthy lifestyle that should be encouraged but no false reassurance about the cancer should be given.

Educational Objective:
There is no evidence that ultrasound of abdomen can help in decreasing mortality from ovarian cancer.
Ms. Epps, a 66-year-old female, comes for removal of a lipoma from her elbow. She wants the swelling out because it looks ugly when she wears sleeveless tops. She is not having any complaints, other than general malaise and fatigue for the past 8 months. However, she suspects that it is related to her being alone all the time. Her vitals are within normal limits but examination reveals mild pallor and both cervical and supraclavicular lymphadenopathy. Her preoperative blood count revealed:

WBC 21,600/cmm
Hemoglobin 10.0
Hematocrit 32%,
Platelets 66,000/cmm

The distribution of leukocytes was as under

Lymphocytes 77%
Bands 4%
Segmented Neutrophils 18%
Monocytes 1%

The pathologist reports presence of “leukocytes that have undergone partial breakdown during preparation of a stained smear or tissue section, because of their greater fragility”. Lymph node biopsy confirms the diagnosis. What is the correct statement about the above patient?


A. Prognosis is extremely poor
B. This is a form of plasma cell leukemia
C. Presence of thrombocytopenia is a poor prognostic factor
D. This is a classic ‘T’ cell disease
E. Most common cause of death is from renal failure
This is a classical description of “smudge cells” which are characteristic of chronic lymphocytic leukemia (CLL). The disease is seen mostly in older patients. The patients are often asymptomatic when diagnosed after lymphocytosis is picked up incidentally. When symptomatic it is usually due to lymphadenopathy. On peripheral blood film it is characterized by lymphocytosis of small mature appearing lymphocytes. The patient may also have splenomegaly, anemia and thrombocytopenia. The staging system is directly related to prognosis.

Stage Clinical Feature Prognosis
0 Lymphocytosis only Good
I Lymphocytosis + adenopathy Fair
II Splenomegaly present Fair
III Anemia present Intermediate
IV Thrombocytopenia Poor

So presence of thrombocytopenia is a poor prognostic factor.

Patients with CLL especially in early stages have almost the same median survival (8-10 years) as the rest of population. Even in later stages with Chlorambucil and Prednisone the prognosis is not as bad as with other forms of leukemias.

Multiple Myeloma is a form of plasma cell leukemia.

CLL is a ‘B’ Cell disease not a ‘T’ cell.

Infections, not renal failure is the principle cause of death if the patient dies of CLL.

Educational Objective:
Know the staging of chronic lymhpocytic leukemia as it is directly related to prognosis of the disease.
Ms. Epps, a 66-year-old female, comes for removal of a lipoma from her elbow. She wants the swelling out because it looks ugly when she wears sleeveless tops. She is not having any complaints, other than general malaise and fatigue for the past 8 months. However, she suspects that it is related to her being alone all the time. Her vitals are within normal limits but examination reveals mild pallor and both cervical and supraclavicular lymphadenopathy. Her preoperative blood count revealed:

WBC 21,600/cmm
Hemoglobin 10.0
Hematocrit 32%,
Platelets 66,000/cmm

The distribution of leukocytes was as under

Lymphocytes 77%
Bands 4%
Segmented Neutrophils 18%
Monocytes 1%

The pathologist reports presence of “leukocytes that have undergone partial breakdown during preparation of a stained smear or tissue section, because of their greater fragility”. Lymph node biopsy confirms the diagnosis. What is the correct statement about the above patient?


A. Prognosis is extremely poor
B. This is a form of plasma cell leukemia
C. Presence of thrombocytopenia is a poor prognostic factor
D. This is a classic ‘T’ cell disease
E. Most common cause of death is from renal failure
This is a classical description of “smudge cells” which are characteristic of chronic lymphocytic leukemia (CLL). The disease is seen mostly in older patients. The patients are often asymptomatic when diagnosed after lymphocytosis is picked up incidentally. When symptomatic it is usually due to lymphadenopathy. On peripheral blood film it is characterized by lymphocytosis of small mature appearing lymphocytes. The patient may also have splenomegaly, anemia and thrombocytopenia. The staging system is directly related to prognosis.

Stage Clinical Feature Prognosis
0 Lymphocytosis only Good
I Lymphocytosis + adenopathy Fair
II Splenomegaly present Fair
III Anemia present Intermediate
IV Thrombocytopenia Poor

So presence of thrombocytopenia is a poor prognostic factor.

Patients with CLL especially in early stages have almost the same median survival (8-10 years) as the rest of population. Even in later stages with Chlorambucil and Prednisone the prognosis is not as bad as with other forms of leukemias.

Multiple Myeloma is a form of plasma cell leukemia.

CLL is a ‘B’ Cell disease not a ‘T’ cell.

Infections, not renal failure is the principle cause of death if the patient dies of CLL.

Educational Objective:
Know the staging of chronic lymhpocytic leukemia as it is directly related to prognosis of the disease.
Ms. Ginning, a 47-year-old woman, comes to her primary care physician for burning pain in her abdomen for the past 3 months. The pain is 6/10, continuous and gets better when she takes antacids. Her father has ulcers in his belly and she thinks she is having the same. She also complains of some constipation. She works as a floor secretary in a surgical ward and is not happy with her new boss. Her review of systems is negative for weight loss or decreased appetite. Her vitals are, Temperature: 37.1C(98.8F); BP: 130/85 mm Hg; HR: 78/min; RR: 14/min. On examination, she is awake, alert, and oriented. There is abdominal tenderness in epigastric region but there is no abdominal mass; bowel sounds are heard. Her stools are occult blood positive. ECG is ordered and it reveals increase in PR interval and short QT. Laboratory results are as under:

Sodium 137 mEq/dL
Potassium 4.2 mEq/dL
Chloride 101 mEq/dL
Bicarbonate 27 mEq/dL
Calcium 13.5 mg/dL
Phosphorus 2.2 mg/dl
BUN 67 mg/dL
Creatinine 2.8 mg/dL

What is the most probable diagnosis?


A. Glucagonoma
B. Stress ulcer
C. Parathyroid adenoma
D. Vitamin-D toxicity
E. Gastric carcinoma with bony metastasis
The patient has classical combination of hypercalcemia and gastric ulcers. It is parathyroid adenoma, which is manifested by increased calcium and decreased phosphorus. Hypercalcemia results in increased gastrin release and there are peptic ulcers as seen in this patient. In multiple endocrine neoplasia (MEN) Type 1 it can be the combination of parathyroid adenoma and Zollinger-Ellison Syndrome (ZES). ZES is caused by a gastrin-producing tumor called gastrinoma causing severe and refractory peptic ulcer disease (PUD).

Glucagonoma is characterized by hyperglycemia, dermatitis, weight loss, and anemia.

The patient has stress at work and her stomach complaints may be related to her stress but that does not explain the hypercalcemia seen in this patient. Usually stress ulcers do not occur with the above presentation. They are usually seen in severely ill patients who are being managed in ICU settings.

Vitamin-D toxicity may lead to hypercalcemia but the history is nowhere suggestive of such medications. It will also cause elevated phosphate rather than decreased.

Every gastric ulcer has to be investigated for malignancy. But here we have a reason (hypercalcemia) for gastric ulcer, which puts this diagnosis lower in the list, but still this is no excuse for not doing the biopsy of the lesion. Advanced gastric cancer is unlikely without any constitutional symptoms. However, measurement of PTH will resolve this situation easily.

Educational Objective:
Hypercalcemia and intractable ulceration can be due to MEN syndromes.
Ms. Gunner, a 46-year-old female, is brought to ER because of visual blurring and altered mental status. Patient was recently treated for gout. Exam showed moderate to severe splenomegaly and positive sternal tenderness. Her labs showed WBC of 120,000/uL with left shifted myeloid series and basophilia. Bone marrow exam showed 3% blasts. A new set of CBC is ordered and a hematologist is called and he starts leukopharesis emergently. What other drug should be given immediately to this patient?


A. Cladribine
B. Hydroxyurea
C. Cyclophosphamide
D. Interferon alpha
E. Imatinib mesylate
F. Busulfan
The patient has Chronic Myelogenous Leukemia (CML). There is increased number of mature granulocytic forms, such as segmented neutrophils and band forms, and smaller number of immature forms on the peripheral film. Her history is also consistent with it. Examination of a bone marrow is expected to reveal hypercellularity with prominent granulocytic hyperplasia.

The clinical course of CML has been characterized by 3 progressive phases. She is right now in the crisis phase, which needs immediate attention. The FDA has approved a new drug for CML called imatinib (Gleevec). Imatinib is a tyrosine kinase inhibitor and works by blocking signals within cancer cells and preventing a series of chemical reactions that cause the cancer cells to grow and divide. Introduction of this medication has dramatically changed the prognosis of CML. Oral preparation is available and trials are on to see its effect on other forms of cancers. The major side effects are mild nausea, diarrhea, leg aches/cramps, swelling of the face and around the eyes, itchy rash, and temporary reduction in the production of blood cells by the bone marrow.

Interferons are no longer considered as first line medication for CML. Hydroxyurea and Busulfan are also not considered for first line for CML. The use of cyclophosphamide, interferons and prednisone in the management of CML is not recommended before Imatinib.

Cladribine is the drug of choice for hairy cell leukemia.

A bone marrow transplant preceded by high-dose chemotherapy and radiation therapy remains the standard treatment, however, although not all people with CML are suitable candidates for transplantation or have a suitable donor. It is not known at the present time which patients should receive a bone marrow transplant as the first treatment and who should receive imatinib.

Educational Objective:
Imatinib has changed the prognosis in patients with chronic myelogenous leukemia.
Ms. Janet, a 65-year-old woman, comes to you because she is much worried after seeing a few of the people who are very close to her dying, one after the other. First, her 85-year-old mother died of an ovarian cancer that was diagnosed 3 years ago. Then last week, she lost her best friend because of lung cancer that has gone to her liver and brain. Today, her 58-year-old sister has learned that she has breast cancer. She is regular with her pap smears and breast examinations. She has read in paper that there are a few ways that have proved to be very successful in preventing breast cancer. Which of the following has the best evidence in preventing breast cancer in high-risk women?


A. Prophylactic oophorectomy
B. BRCA screening
C. Low fat diet
D. Tamoxifen
E. Having mammograms every month
Tamoxifen is an antiestrogen drug, used to treat breast cancer. Tamoxifen blocks the effects of the hormone, estrogen, in the body. Research has shown that when tamoxifen is used as adjuvant therapy for early stage breast cancer, it reduces the risk of recurrence of the original cancer. It also reduces the risk of developing new cancers in the other breast. Based on these findings, the National Cancer Institute (NCI) funded a large research study to determine the usefulness of tamoxifen in preventing breast cancer in women who have an increased risk of developing the disease. This study was known as the "Breast Cancer Prevention Trial" (BCPT). This study found a 49 % reduction in diagnoses of invasive breast cancer, among women who took tamoxifen. Women who took tamoxifen also had 50% fewer diagnoses of non-invasive breast tumors, such as ductal or lobular carcinoma in situ. However, there are risks associated with tamoxifen. Some are even life threatening. The decision to take tamoxifen is an individual one. The woman, and her doctor, must carefully consider the benefits and risks of therapy. Women with an increased risk of developing breast cancer have the option to consider taking tamoxifen to reduce their chance of developing this disease. At this time, there is no evidence that tamoxifen is beneficial for women who do not have an increased risk of developing breast cancer.

Educational Objective:
Tamoxifen reduces the risk of breast cancer in patients who are at an increased risk of developing breast cancer.
Ms. Karceski, a 50-year-old woman, comes with complaints of having pain in her right shoulder. She has had rheumatoid arthritis for the past 10 years and is "used to" having pain in her joints. Her fingers are always swollen and she is finding it difficult to walk now, with her knees giving way. She states that this pain is different, as it is moving to her hand. So, she thought she better come see a doctor. Also, she has had cough for many weeks and is feeling more tired. She takes celecoxib for rheumatoid arthritis. She smokes 1 pack/day cigarettes for 25 years, and drinks "one odd" beer every night. All her family members have crippling rheumatoid arthritis. On examination, her vitals are stable, with no fever. However, there is drooping of the right eyelid and miosis. What is the next best step in management?


A. CT scan of head and neck
B. X-ray shoulder
C. Start steroids
D. Chest X-ray
E. Nerve conduction study
F. EMG
The patient’s shoulder pain is due to direct extension of lung cancer and involving the nerves of brachial plexus and sympathetic trunk. She has few important clinical clues. Presence of partial ptosis, and miosis (Horner’s syndrome) in a known smoker should alert you think of lung cancer until proven otherwise. So simple chest x-ray should be the first test in this patient. Her joint problems might be a part of hypertrophic osteoarthropathy.
Most lung cancers are caused by cigarette smoking. The more cigarettes you smoke/day and the earlier you started smoking equals the greater the risk of lung cancer. Second-hand smoke has also been shown to increase risk. High levels of pollution, radiation, and asbestos exposure may also increase risk.
Lung cancer can present with either of the following:
1. Cough with Bloody sputum
2. Shortness of breath
3. Chest pain
4. Loss of appetite
5. Weight loss
Additional symptoms that may be associated with this disease:
6. Weakness
7. Swallowing difficulty
8. Hoarseness or changing voice
9. Facial swelling
10. Facial paralysis
11. Eyelid drooping

Though the patient has a history of RA the pain does not appear to be related to it, so starting steroids is not really the best option.

This is not a nerve disease so nerve conduction and EMG are not the part of management here.

There is no history of trauma, so shoulder x-ray is not the test of first choice.

Educational Objective:
Presence of partial ptosis, and miosis (Horner’s syndrome) in a known smoker should alert you to think of lung cancer until proven otherwise. Simple chest x-ray should be the first test in patients with suspected lung cancer.
Ms. Kerstin, a 51-year-old woman, comes to you for annual examination. She is healthy and is not having any complaints. Her diet is normal and she exercises regularly. Her vitals are stable and but some pallor is noted on clinical examination. You perform fecal occult blood test and you get a positive sample. You also order a complete blood count and serum electrolytes. The following results are obtained

WBC 7,600/mm3
Hemoglobin 9.8
Hematocrit 32%,
RBC cont 3.1 million
Platelets 116,000/mm3

You also find iron studies done on the patient last month when she volunteered for a study on a new ‘novel’ diet plan. The results were

Serum iron 44µg/dl
TIBC 461µg/dl
Percent saturation of transferrin 9.5%

Which one of the following is expected on blood indices?
MCV(fl), MCH(pg), MCHC(%)


A. 82, 28, 32
B. 97.6, 29.2, 31
C. 96.4, 29.3, 38
D. 120.4, 38.6, 30
E. 68.4, 16, 23
The patient has microcytic hypochromic anemia due to chronic blood loss from gastrointestinal tract. In such cases we have chronic iron loss leading to Iron deficiency anemia. RBC indices reveal a decreased MCV and MCH (Choice E). On serum Iron studies, a decreased serum iron and percent saturation (Serum Iron / TIBC) along with an increased TIBC is seen, which is consistent with Iron deficiency anemia. The peripheral blood smear shows marked anisocytosis, microcytosis, hypochromia and poikilocytosis. The reticulocyte count is low. On bone marrow examination, though rarely done, decreased body iron stores are detected. Clinical manifestations include pallor, fatigue or dyspnea on exertion.

Choices A and B are seen in normal conditions or is the case in acute blood loss.

Choice C is seen in hereditary spherocytosis (Elevated MCHC).

Choice D is seen in megaloblastic anemias.

Educational Objective:
Know the blood indices in various types of anemias. Iron deficiency anemia is characterized by decreased MCV and MCH.
Ms. Lambert is a 34-year-old woman who has been your patent for the last 4 year. She comes to you for her annual examination. She has read in “US Health News” that breast cancer is the leading culprit for cancer death among non-smoking women. She eats a lot of meat but does not like fruits and vegetables. Her menstrual period began at age 12 and she has two boys ages 13 and 11. She is in a monogamous relationship with her husband and uses oral contraceptive pills. She smokes 1/2 pack/day and drinks alcohol socially. Her mother had some breast disease and she was operated on. However, she died 1month after the surgery because of a heart attack. She wants to know what is the most important risk factor for breast cancer?


A. Parity of woman
B. Age of menarche
C. Age of woman
D. Use of oral contraceptive pills
E. Family history of breast cancer
F. BRCA status
G. Sexual orientation
H. Alcohol ingestion
The most significant risk factor for breast carcinoma is increased age. Breast cancer is relatively rare before the age of 30 and then risk increases with age. The incidence rises sharply after 45-50yr. Family history of breast cancer is an important risk factor, but only 10% of women diagnosed with breast cancer have a positive family history.

First-degree relatives of breast cancer patients have 3 times increased risk of developing breast cancer after the age of 35yr. Especially, if the involved member was diagnosed premenopausal and had bilateral disease.

In 1994 BRCA1 gene was identified. It is present in 5-10% of women and these women are at 50% increased risk of developing breast cancer.

Early menarche, nulliparity, conjugated estrogens, and alcohol ingestion are all associated with slight increased risk of breast cancer.

Sexual orientation has no association with breast cancer.

Educational Objective:
The most significant risk factor for breast carcinoma is increased age.
Ms. Linda, a 19-year-old lady, presents for evaluation of resent change in her menstrual pattern. She is soaking 5 pads/day for the last 4 periods. This time menses continued up to 12 days that made her exceedingly worried. She remembers her younger brother having several nose bleeds that needed cauterization to arrest. Physical examination is unremarkable except for pallor. CBC was ordered and results are

WBC 6,000cmm
Hemoglobin 9.5gm/dL
Hematocrit 27%,
Platelets 196,000cmm

Coagulation profile reveals:

Bleeding Time 6 min
PT 12.1s
APTT 30s
Clot retraction Decreased

The physician decides to go for some qualitative platelets tests, which shows:

Platelet aggregation studies with ADP Abnormal
Platelet aggregation studies with ristocetin Normal
Von Willebrand factor (VWF) Level Normal

What is the most probable diagnosis?


A. Thrombotic thrombocytopenic purpura
B. Von Willebrand disease
C. Idiopathic thrombocytopenic purpura
D. Glazmann’s thrombasthenia
E. Chédiak - Higashi Syndrome
F. Bernard-Soulier syndrome
This is a classical presentation of Glazmann’s Thrombasthenia. It is an autosomal recessive disorder that results in deficient glycoproteins IIb-IIIa complex so fibrinogen will not cross-connect. The patient presents with increased bleeding episodes for some time. Platelet counts may be normal but on the peripheral blood stream, platelets remain isolated and do not exhibit clumping that is normally seen. The bleeding time (BT) is a measure of the interaction of platelets with the blood vessel wall. Bleeding time is markedly increased and clot retraction is decreased. The diagnosis lies in qualitative platelet tests. Epinephrine, collagen and thrombin fail to induce aggregation. There is absence of primary wave of aggregation in response to ADP however platelet aggregation studies with ristocetin are normal. Von Willebrand factor is also normal.

Thrombotic thrombocytopenic purpura is a fulminant disorder and results in marked decrease in platelet count.

VWF Level is decreased in Von Willebrand disease.

Idiopathic thrombocytopenic purpura is an autoimmune disease when antibodies are formed against platelets resulting in decreased platelet count.

Chédiak - Higashi Syndrome is a storage granulocyte abnormality resulting in hepatosplenomegaly, lymphadenopathy, anemia, thrombocytopenia, roentgenological changes of bones, lungs and heart, skin and psychomotor abnormalities, and susceptibility to infection, usually resulting in death in childhood.

Bernard-Soulier syndrome is a bleeding disorder characterized by thrombocytopenia, giant platelets, and a bleeding tendency, which is typically greater than expected bleeding for the degree of thrombocytopenia. This patient does not have thrombocytopenia. Platelets from these patients do not aggregate in the presence of normal vWF and ristocetin because of the decrease or abnormality in the GP Ib.

Educational Objective:
Know rare bleeding disorders. They are not common in real life but frequently appear in the exams.
Ms. Parker, a 46-year-old bank executive, is referred by her dentist. She had complaints of swollen and bleeding gums for 6 weeks. She appears pale and complains of feeling weak. She smokes half a pack of cigarettes and drinks socially. Her family history is not significant. Her vitals are stable and she is afebrile. CBC reveals:

WBC 44,100mm3
Hemoglobin 10.0
Hematocrit 32%,
Platelets 16,000mm3

The leukocyte distribution is as follows

Blast forms 79%
Promonocytes 12%
Monocytes 8%
Lymphocytes 1%

Cytochemical tests are ordered and they are as under:

Sudan Black Slightly positive
Alpha-naphthyl esterase Positive
PAS reaction Negative

What is the most probable diagnosis?


A. Acute myeloblastic leukemia with maturation
B. Acute promyelocytic leukemia
C. Acute lymphoblastic leukemia
D. Acute erythro leukemia
E. Acute monocytic leukemia
The patient has acute monocytic (FAB M5) leukemia. M5 comprises less than 15% of all leukemias. Two forms are recognized. FAB M5A, which is common in young adults (mean age 16), while FAB M5B is seen in middle age (median age 49yr). The onset of this form of leukemia is dramatic, headaches and fever being the chief complaints. Other symptoms include fatigue, weight loss, and bleeding from mouth or nose. There is gingival hyperplasia and some times skin lesions. Lymphadenopathy and splenomegaly are uncommon. There is leukocytosis with high proportion of blast forms. Chemically they are ‘alpha-naphthyl esterase’ positive.

In acute myeloblastic leukemia with maturation (M2) myeloblast predominate on the peripheral blood film.

Acute promyelocytic leukemia (M3) has a lot of promyelocytes, which may be hyergranular, microgranular or hypogranular. DIC is also some times seen with this type.

In acute lymphoblastic leukemia the predominating cell in peripheral blood film are lymphoblasts. They are mostly PAS positive.

Acute erythro leukemia (M6) is characterized by the erythroblasts, which have an irregular outline with a high nuclear-cytoplasmic ratio.

Educational Objective:
Recognize acute monocytic (FAB M5) leukemia based on the smear and positive ‘alpha-naphthyl esterase’.
Ms. Sally, a 54-year-old woman, comes to the office with complaints of increased weight in the last 4 months. She denies any change in appetite or eating habits, but has gained 50 lb(27.6kg). She is on no medications and is only social drinker. She is very diet conscious and follows the exact instructions in "Dr. Atkin’s Diet revolution". She smokes 1pack/day cigarettes for the past 16 years. Physical examination shows an obese abdomen and thin legs, but is otherwise unremarkable. Further work-up confirms the diagnosis of small cell carcinoma of lungs. Which of the following set of tests will be most related to patient's presenting symptoms?

What will be the levels of:
Cortisol, ACTH levels, Low Dose Dexamethasone Suppression, High Dose Dexamethasone Suppression


A. Increased, Increased, No Suppression,Suppression
B. Increased, Decreased,No Suppression,No Suppression
C. Increased, Increased, Suppression, No Suppression
D. Increased, Increased, No Suppression,No Suppression
E. Normal,Normal,Suppression,Suppression
Some benign or malignant (cancerous) tumors that arise outside the pituitary can produce ACTH. This condition is known as Ectopic ACTH syndrome. Lung tumors cause over 50 % of these cases. Men are affected 3 times more frequently than women. The most common forms of ACTH-producing tumors are oat cell, or small cell lung cancer, which accounts for about 25 % of all lung cancer cases, and carcinoid tumors. Other less common types of tumors that can produce ACTH are thymomas, pancreatic islet cell tumors, and medullary carcinomas of the thyroid. Cushing’s syndrome work up shows the results as shown in Choice D. Choice A is seen in pituitary adenoma while Choice B is seen in Adrenal neoplasia. Choice C is a distracter while Choice E is normal.

Since, it is an ectopic ACTH syndrome, ACTH and cortisol will be elevated. Because the amount of ACTH is very high, it is difficult to suppress even with high dose dexamethasone.

Educational Objective:
Patients with ectopic producation of ACTH have very high ACTH levels and their cortisol level is not suppressed with high dose dexamethasone suppression test.
Ms. Shelf, a 22-year-old woman, comes to you because she has noticed something in her breast, while taking shower, 2 months ago. There is severe pain in this mass, during menses. She looks very anxious because her 45 year old friend was diagnosed with breast cancer last year. Now, she is having all sorts of medicines that have made her lose her hair. She has no other problems and has never been pregnant. She is occasional smoker and drinks 3-4 beers/wk. There is no family history of breast cancer. Her vitals are stable. On examination there is a 4 x 5 x 6cm firm, moveable, rubbery mass in her left breast. Ultrasound shows it is cystic and when put a needle in, you get a clear fluid. The mass has disappeared. What is the best approach in this patient?


A. Send the fluid for cytology
B. Perform a core biopsy
C. Observe for 4 weeks
D. Order a mammogram to look for other lesions
The patient has classical fibrocystic disease. Aspiration of the cyst was performed appropriately, and we got a clear fluid. The best approach in this patient is to wait for 4-6wk (Choice C). In fibrocystic disease the mass goes away and does not come back. If the mass recurs, or it doesn’t go away, then a biopsy is indicated. She is too young to have a useful mammogram (Choice D). If the fluid was bloody or foul smelling, cytology or further examination was needed at this stage. (Choice A)

Educational Objective:
For a breast mass that disappears completely with FNA and fluid aspirated is clear, next step is to observe for 4 weeks.
Ms. Shelf, a 22-year-old woman, comes to you because she has noticed something in her breast, while taking shower, 2 months ago. There is severe pain in this mass, during menses. She looks very anxious because her 45 year old friend was diagnosed with breast cancer last year. Now, she is having all sorts of medicines that have made her lose her hair. She has no other problems and has never been pregnant. She is occasional smoker and drinks 3-4 beers/wk. There is no family history of breast cancer. Her vitals are stable. On examination there is a 4 x 5 x 6cm firm, moveable, rubbery mass in her left breast. Ultrasound shows it is cystic and when put a needle in, you get a clear fluid. The mass has disappeared. What is the best approach in this patient?


A. Send the fluid for cytology
B. Perform a core biopsy
C. Observe for 4 weeks
D. Order a mammogram to look for other lesions
The patient has classical fibrocystic disease. Aspiration of the cyst was performed appropriately, and we got a clear fluid. The best approach in this patient is to wait for 4-6wk (Choice C). In fibrocystic disease the mass goes away and does not come back. If the mass recurs, or it doesn’t go away, then a biopsy is indicated. She is too young to have a useful mammogram (Choice D). If the fluid was bloody or foul smelling, cytology or further examination was needed at this stage. (Choice A)

Educational Objective:
For a breast mass that disappears completely with FNA and fluid aspirated is clear, next step is to observe for 4 weeks.
Ms. Smith is a 39-year-old woman complains of double vision. She is feeling "weak all over", especially at the end of the day. She had the same complaints 8 months ago that persisted for several weeks. However, she didn’t see the doctor, as she had no insurance then. She has no past medical history. Her mother has rheumatoid arthritis, and her brother has type 1 diabetes mellitus. Her vital signs are normal. She has diplopia when extra ocular muscles are tested and also has mild ptosis. The blood profile, CBC and thyroid tests are within normal limits. On electromyogram you see diminished amplitude of compound muscle action potential to repetitive stimulation. Acetylcholine receptor antibody test is positive. Which of the following test should be ordered next?


A. Muscle biopsy
B. Edrophonium (Tensilon) test
C. Anti-Jo antibodies
D. Anti-RNP antibodies
E. CT scan of chest
The diagnosis is Myasthenia Gravis (MG). The most common symptoms of MG relate to weakness of the muscles that lift up the lid (ptosis) or move the eyes (double vision). MG can affect muscles anywhere in the body including those of swallowing or even breathing. Shortness of breath or difficulty swallowing may be very serious symptoms of MG. MG do not produce pain or numbness. If pain is present there must be something else going on. The diagnosis is confirmed by EMG and acetylcholine receptor antibody test, which are confirmatory along with the suggestive signs and symptoms. After the diagnosis is confirmed, it is important to look for evidence of thymoma, especially in patients younger than age 60 (Choice E). Thymectomy is mostly useful in such cases though the results are seen 3-4 yeas after the surgery.

There is no use of doing edrophonium (Tensilon) test (Choice B), as it is difficult to interpret if there are only mild findings and is far less sensitive. We already have a diagnosis of MG.

Anti-Jo antibodies (Choice C) and Anti-RNP antibodies (Choice D) are done for Polymyositis and Mixed Connective Tissue Disease respectively.

Muscle biopsy (Choice A) is not the next best test but can be done if myositis is suspected or diagnosis of MG is difficult.

Educational Objective:
CT chest should be done to look for thymoma in all newly diagnosed myasthenia gravis patients.
Ms. Stern is a 57-year-old female diagnosed with stage II Hodgkin’s lymphoma 4 weeks ago. She is put on combination chemotherapy of ABVD regimen (Doxorubicin, Bleomycin, Prednisone, and Procarbazine). She complains of severe nausea and vomiting. She does not want to continue with her medicine, as they are making her life even more miserable. She is also having some general malaise, and then had a bit of a sore throat. She has lost 30 lb(12.6kg) in a month. Her vitals are, Temperature: 38.5C(101.3F); PR: 88/min; RR: 18/min; BP: 109/68mm of Hg. The patient is on 92% pulse oximetry on room air. On examination the mouth is dry and she looks significantly cachectic with enlarged cervical lymph nodes and hepatomegaly. How can her vomiting be best managed?


A. Stop the chemotherapy drugs
B. Ondansetron
C. Metoclopramide
D. Tell patient that her nausea will get better with time
E. Prochlorperazine
Nausea and vomiting is one of the major undesirable effects of chemotherapy. It is the major reason for non-compliance and failure of chemotherapy regimes. Patients should be thoroughly educated about nausea and vomiting before the chemotherapy begins. As far as pharmacological treatment is concerned, a 5-HT3 receptor antagonist Ondansetron is very promising in reducing chemotherapy-induced emesis and is available in a tablet form also. The other steps that should be taken in such patients include small frequent meals, avoiding bland foods, appropriate food presentation, and giving patient what he wants to eat.

Nausea and vomiting is a major hindrance to patient's compliance to chemotherapy so telling him to bear it is not the best option.

Metoclopramide and prochlorperazine are the classical agents that can be give to such patients either alone or in combination with ondansetron though they are not the best drugs for chemotherapy-induced emesis.

Expected side effects like nausea and vomiting are not the indications to stop chemotherapy.

Educational Objective:
Ondansetron is the drug of choice for chemotherapy-induced emesis.
Stupor Junior is an 8-month-old African-American child referred by a nurse practitioner because she observed that mucous membranes are pale. She also noted that the child was irritable and listless. Stool examination is negative for occult blood, ova and parasites. She also ordered some basic tests and the results are:

WBC 6,600/cmm
Hemoglobin 5.8
Hematocrit 26%,
RBC cont 3.5 million
Platelets 116,000/cmm

The child appears to be pale which is consistent with his blood work. Further blood tests are done which reveal:

MCV 68 fL
MCH 16.5 pg
MCHC 25%
Reticulocyte count 0.6%
Serum Iron 40µg/dl
TIBC 460µg/dl
Percent saturation of transferrin 8.7%
Total serum bilirubin 0.9mg/dl

The peripheral blood smear shows marked anisocytosis, microcytosis, hypochromia, and poikilocytosis. What is the diagnosis?


A. Iron deficiency anemia
B. Sideroblastic anemia
C. Dimorphic anemia
D. Megaloblastic anemia
E. Pernicious anemia
The child has hypochromic, microcytic erythrocytes in a peripheral blood film suggestive of an iron deficiency anemia. RBC indices reveal a decreased MCV and MCH. On serum iron studies, a decreased serum iron and percent saturation (Serum Iron /TIBC) along with an increased TIBC is seen, which is consistent with iron deficiency anemia. The serum bilirubin level is normal ruling out hemolysis. However, no evidence of bleeding and parasitic infections was detected. So, the most probable cause of this iron deficiency is inadequate diet. Small children are most frequent victim of this unfortunate disorder. With a diet consisting largely of milk, and unsupplemented with fortified food products during the early years of development, iron deficiency anemia is a frequent by product.

Serum iron is increased while TIBC may be normal in sideroblastic anemia.

In dimorphic anemia two distinct forms of red cells are circulating.

Megaloblastic anemia results in elevated MCV, elevated MCH, and normal MCHC.

Pernicious anemia is a cause of megaloblastic anemia due to absence of intrinsic factor secondary to gastric atrophy.

Educational Objective:
Know the laboratory findings in various types of microcytic anemias.
You receive a page from a Hospice nurse that wants you to see a patient. You find an 80-year-old woman with stage IV breast cancer. It has metastasized to liver and she has refused chemotherapy because of the side effects. Her abdomen is distended and on examination you find the fluid thrill positive with some irregular masses in liver. The woman does not want any invasive procedure done on her but is willing to take pills to "have her belly settle down." What is the drug of first choice in this patient?


A. Dexamethasone
B. Acetazolamide
C. Furosemide
D. Hydrochlorothiazide
E. Spironolactone
The patient has malignant ascites and the method of choice for acute relief is paracentesis, which the lady has refused. The drug of first choice is the aldosterone antagonist spironolactone. Spironolactone is a potassium-sparing diuretic also used for ascites associated with liver cirrhosis. Malignant ascites is seen most commonly in patients with ovarian, endometrial, breast, colon, gastric, and pancreatic cancer. The management of malignant ascites may include systemic chemotherapy, instillation of radioisotopes or chemotherapy drugs into the peritoneal fluid, and peritoneal-venous shunting procedures.

Other diuretics can be added to increase the effect of spironolactone (Choices B, C, D).

Dexamethasone has no role in the management of malignant ascites. (Choice A)

Educational Objective:
Spironolactone is the diuretic of choice for managements of ascites.
A 6-year-old African-American child is brought in by his father for complaints of easy fatigability and pallor. He reports the above symptoms in his son after he was treated with "some medication" for a recent diarrhea. Physical examination is normal except for pallor and multiple petechiae. Laboratory values are as follows:

Hb 8.0 g/dL
WBC 12,000/cmm
Platelets 50,000/cmm
Blood glucose 118 mg/dL
Serum Na 135 mEq/L
Serum K 5.3 mEq/L
Chloride 110 mEq/L
Bicarbonate 18 mEq/L
BUN 38 mg/dL
Serum creatinine 2.5 mg/dL
Total bilirubin 3 mg/dL
Direct bilirubin 0.5 mg/dL
PT 12 seconds
APTT 30 seconds.
LDH 400 IU/L
Reticulocyte count 3%.

A peripheral blood smear reveals giant platelets and multiple schistocytes. What is the most likely underlying pathophysiology for his pallor?


A. Sickle cell anemia
B. Thalassemia
C. Vitamin B12 deficiency
D. Folate deficiency
E. Microangiopathic hemolytic anemia
F. Lead poisoning
G. Disseminated intravascular coagulation
H. Idiopathic thrombocytopenic purpura
I. Renal failure
Hemolytic Uremic Syndrome is typically a disease of young children. It is usually preceded by an acute diarrheal illness (due to the pathogens Escherichia coli serotype 0157:H7 and Shigella, Salmonella, Yersinia, and Campylobacter species). It is less commonly preceded by an upper respiratory infection. The hallmark finding of this condition is a microangiopathic hemolytic anemia. Other features are acute renal failure, fever, oliguria (or anuria) and thrombocytopenia. GI bleeding is also a common symptom. Physical examination frequently reveals purpura and hypertension. Schistocytes are found in the peripheral smear, which represent fragmented RBCs. The peripheral smear also may contain giant platelets. Intravascular hemolysis results in elevated lactate dehydrogenase (LDH), indirect bilirubin and reticulocyte count. BUN and creatinine are markedly elevated. A moderate leukocytosis may be present. The urine contains hemoglobin, hemosiderin, albumin, RBCs, WBCs, and casts.

Choice A: Sickle cell anemia is another type of hemolytic anemia, which can be easily ruled out in this case, due to the absence of sickle cells on peripheral smear.

Choice B: Patients with Thalassemia present with hepatosplenomegaly (absent in this patient). Peripheral blood smear may reveal target cells, microcytosis, hypochromia, and anisopoikilocytosis, but not schistocytes. BUN and Cr will be normal.

Choice C: Vitamin B12 deficiency is characterized by macrocytosis, and hypersegmented neutrophils on peripheral smear.

Choice D: Folate deficiency also presents with macrocytosis and hypersegmented polymorphonuclear leucocytes (PMNs).

Choice F: Lead poisoning causes a microcytic anemia, with basophilic stippling of RBCs on peripheral smear. Besides, it also presents with some GI symptoms.

Choice G: Prothrombin time (PT) and activated partial thromboplastin time (aPTT) would be expected to be high in patients with DIC, unlike this patient, who has normal values for these tests.

Choice H: ITP is characterized by an isolated thrombocytopenia. On peripheral smear, the morphology of red cells is normal.

Choice I: Bleeding in renal failure patients is usually due to dysfunctional platelets; it is very unusual to see schistocytes and hemolytic picture from renal failure.

Educational objective:
Recognize HUS in a child who has recently recovered from a diarrheal illness and presents with acute renal failure, microangiopathic hemolytic anemia, fever, thrombocytopenia and characteristic peripheral smear finding of schistocytes.

*Extremely high yield question for USMLE!!!